You are on page 1of 142

SOLUTIONS MANUAL FOR

Stresses in Beams,
Plates, and Shells,
Third Edition

by
Ansel C. Ugural
SOLUTIONS MANUAL FOR
Stresses in Beams,
Plates, and Shells,
Third Edition

by
Ansel C. Ugural

Boca Raton London New York

CRC Press is an imprint of the


Taylor & Francis Group, an informa business
CRC Press
Taylor & Francis Group
6000 Broken Sound Parkway NW, Suite 300
Boca Raton, FL 33487-2742
© 2010 by Taylor and Francis Group, LLC
CRC Press is an imprint of Taylor & Francis Group, an Informa business

No claim to original U.S. Government works

Printed in the United States of America on acid-free paper


10 9 8 7 6 5 4 3 2 1

International Standard Book Number: 978-1-4398-1544-1 (Paperback)

This book contains information obtained from authentic and highly regarded sources. Reasonable efforts have been made to publish reliable data and
information, but the author and publisher cannot assume responsibility for the validity of all materials or the consequences of their use. The authors and
publishers have attempted to trace the copyright holders of all material reproduced in this publication and apologize to copyright holders if permission
to publish in this form has not been obtained. If any copyright material has not been acknowledged please write and let us know so we may rectify in any
future reprint.

Except as permitted under U.S. Copyright Law, no part of this book may be reprinted, reproduced, transmitted, or utilized in any form by any electronic,
mechanical, or other means, now known or hereafter invented, including photocopying, microfilming, and recording, or in any information storage or
retrieval system, without written permission from the publishers.

For permission to photocopy or use material electronically from this work, please access www.copyright.com (http://www.copyright.com/) or contact
the Copyright Clearance Center, Inc. (CCC), 222 Rosewood Drive, Danvers, MA 01923, 978-750-8400. CCC is a not-for-profit organization that provides
licenses and registration for a variety of users. For organizations that have been granted a photocopy license by the CCC, a separate system of payment
has been arranged.

Trademark Notice: Product or corporate names may be trademarks or registered trademarks, and are used only for identification and explanation
without intent to infringe.
Visit the Taylor & Francis Web site at
http://www.taylorandfrancis.com
and the CRC Press Web site at
http://www.crcpress.com
CONTENTS

PART I FUNDAMENTALS

Chapter 1 BASIC CONCEPTS 1

Chapter 2 STRESSES IN SIMPLE STRUCTURAL MEMBERS 8

PART II PLATES

Chapter 3 ELEMENTS OF PLATE - BENDING THEORY 16

Chapter 4 CIRCULAR PLATES 25

Chapter 5 RECTANGULAR PLATES 38

Chapter 6 PLATES OF VARIOUS GEOMETRICAL FORMS 57

Chapter 7 NUMERICAL METHODS 63

Chapter 8 ANISOTROPIC PLATES 79

Chapter 9 PLATES UNDER COMBINED LOADINGS 87

Chapter 10 LARGE DEFLECTIONS OF PLATES 94

Chapter 11 THERMAL STRESSES IN PLATES 97

PART III SHELLS

Chapter 12 MEMBRANE STRESSES IN SHELLS 104

Chapter 13 BENDING STRESSES IN SHELLS 116

Chapter 14 APPLICATIONS TO PIPES, TANKS, AND PRESSURE VESSELS 122

Chapter 15 CYLINDRICAL SHELLS UNDER GENERAL LOADS 131

iii
NOTES TO THE INSTRUCTOR

The Solutions Manual to accompany the text Stresses in Beams, Plates and Shells
supplements the study of stress and deformation analyses developed in the book. The
main objective of the manual is to provide efficient solutions for problems dealing with
variously loaded structural members. This manual can also serve to guide the instructor
in the assignments of problems, in grading these problems, and in preparing lecture
materials as well as examination questions. Every effort has been made to have a
solutions manual that can cut through the clutter and is self-explanatory as possible thus
reducing the work on the instructor. It is written and class tested by the author.

As indicated in its preface, the text is designed for the senior and/or first year
graduate level courses in the analysis of beams, pates and shells, stress analysis, pressure
vessels, advanced statics, or special topics in solid and structural mechanics. In order to
accommodate courses of varying emphasis, considerably more material has been
presented in the book than can be covered effectively in a single three-credit course.
The instructor has the choice of assigning a variety of problems in each chapter. Answers
to selected problems are given at the end of the text. A description of the topics covered
is given in the introduction of each chapter throughout the text. It is hoped that the
foregoing materials will help instructor in organizing his course to best fit the needs of
his students.

Ansel C. Ugural
Holmdel, N.J.

iv
CHAPTER 1

SOLUTION (1.1)

(a) RAy 7.5 kN


1m Entire Structure
RAx
A C D ∑M A = 0: 7.5(1) + 3(1) = 15
. RBx
1.5 m or RBx = 7 kN
RBx
B
3 kN
∑F x = 0: RAx = −7 kN
RBy = 0 ∑F y = 0: RAy = 10.5 kN
(b)
7.5 kN Member AD

7 kN
1m 1m Cx
∑ M = 0: C (2) = 7.5(1); C = 3.75 kN
A y y
A C
∑ F = 0: A = 7.5 − 3.75 = 3.75 kN
y y
Ay Cy
∑ F = 0: C = 7 kN
x x
(c)
5/6 Segment AE
10/3
5/3 10/3
M ∑ F = 0: P = 7 kN
x

7 kN
A E P
∑ F = 0: V = 3.75 − −
y
5
6
10
3 = 0.417 kN
3.75 kN 1/3
1/2 1/2
V
∑ M = 0: M = 3.75(1) −
E
5
6 ( 23 ) − 103 ( 12 )
= 1528
. kN ⋅ m

SOLUTION (1.2)
Refer to Fig. P1.2: ∑M A = 0: RB = 2
3 pa
(a)
p
∑M 0 = 0: M= 2
3 pax − 12 px 2 (a)
Thus
M B
V 0
dM
dx = 0:
pa − px = 0; 2
3 x = 23 a
x 2
pa
3
Equation (a), for x = 23 a; M max = 2
9 pa 2
( b ) Equation (a), for x = a: 3
2

M = 23 pa( 23 a ) − 12 p( 23 a ) 2 = − 18 pa 2 = 1
8 pa 2
Shear force at x = 23 a:
V = − 23 pa + 23 pa = 56 pa

SOLUTION (1.3)
(a)
40 kN 20 kN Member AB

2m 1m 1m
∑ M = 0: R = 35 5 kN
B A

A
1 40 kN ⋅m B RBx ∑ F = 0: R = 70 kN
x Bx

2 RA RBy ∑ F = 0: R = 25 kN
y By (CONT.)

1
(1.3 CONT.)
Member AC
35 kN ∑ M = 0: R = 2 R
A Cx Cy

∑ F = 0: R = 35 kN
A 70 kN
1m y Cy

∑ F = 0: R = 70 kN
RCx
2m C x Cx
RCy

(b)
Segment AD
35
M ∑ F = 0: P = 70 kN
x

∑ F = 0: V = 35 kN
A 1m
P y
70 D
V ∑ M = 0: M = 35 kN ⋅ m
D

SOLUTION (1.4)

Link BD is a two-force member and hence the direction of FBD is known.

(a) Free body-Member ADE


E ∑M A = 0:
D
P − P(4a) + FBD (2a ) = 0, FBD = 2 P ↑
A
RAx
FBD ∑F x = 0 : RAx = 0
2a 2a ∑F y = 0:
RAy − RAy + FBD − P = 0, RAy = P ↓

y VO (b) Free Body-Part AO


A
FO ∑x F x = 0 : FO = 0

P 1.5a
O
MO ∑F = 0: V = P ↑
y O

∑ M = 0 : M = 1.5Pa
O O

SOLUTION (1.5)

We have θ = 115o . Apply Eqs. ( 1.11 ):

σ x n = 12 (−50 + 40) + 12 (−50 − 40) cos 230o − 20sin 230o


= −5 + 28.92 + 15.32 = 39.2 MPa

τ x ' y ' = − 12 (−50 − 40) sin 230o − 20 cos 230o = −21.6 MPa

2
SOLUTION (1.6)

(a) τ (25, 50) θ p " = 12 tan −1 3750.5 = 26.6 o


( MPa )
r = (50 2 + 37.52 ) 2 = 62.5
1

2θ p "
r Thus,
0 C
σ σ1 = 62.5 + 62.5 = 125 MPa
σ2 σ1 ( MPa ) σ2 = 0
37.5

σ ' = 62.5
(b) τ max = r = 62.5 MPa
125 MPa

62. MPa x'

26.6 o
x

SOLUTION (1.7)
(a) τ ( MPa ) σ ' = 12 ( −150 + 80) = −35 MPa
θ p " = 12 tan −1 115
70
= 15.66 o
σ'
r = (1152 + 70 2 ) 2 = 134.6
1
115
σ
σ2 2θ p" C 0 σ1 ( MPa ) Thus,
(-150, -70) σ1 = −35 + 134.6 = 99.6 MPa
σ 2 = −169.6 MPa

99.6 MPa τ max = r = 134.6 MPa


(b)
x
o
15.66
x'
169.6 MPa

134.6 MPa 35 MPa

SOLUTION (1.8)
(a) y
sin 40 o
area=1
x' x'
σ x'
60
σx τ x' y' θ
x
θ 40 o
x
τ xy = 60 MPa o y' (CONT.)
cos 40

3
(1.8 CONT.)
We have σ y = 0 and τ xy = 60 MPa .
∑F x = 0: σ x sin 40 o = 60 cos 40 o + 60 cos 40 o
or σ x = 143 MPa ( comp. )

Apply Eqs. (1.11) with θ = 90 + 25 = 115o :


σ x = − 143
' 2 − 2 cos 230 + 60 sin 230 = −715
143 o o
. MPa
τ x ' y ' = 715
. sin 230 o + 60 cos 230 o = −9334
. MPa
σ1
(b) x
τ max = [( −715
. )2 + 60 2 ] = 9334
1
. MPa 2
20 o
σ ' = −715. MPa x'
It may be seen from a sketch of Mohr’s Circle that σ2
θ p " = tan1
2
−1 60
71.5 = 20 o 93.3 MPa
71.5 MPa

SOLUTION (1.9)

7.58 MPa
y' 105.6 MPa σ x = σ y = 80 cos 45o = 56.57 MPa
x' τ xy = 80 sin 45o = 56.57 MPa

θ = 30 o
x
28.29 MPa

Apply Eqs. (1.11):


σ x ' = 12 (56.57 + 56.57) + 0 + 56.57 sin 60 o = 105.6 MPa
σ y ' = 56.57 − 48.99 = 7.58 MPa
τ x ' y ' = −0 + 56.57 cos 60 o = 28.29 MPa

SOLUTION (1.10)

We have σ x = 0, σ y = −50 MPa , τ xy = 100 MPa , θ = 70 o


Apply Eqs. (1.11):
σ x ' = − 502 + 502 cos 140 o + 100 sin 140 o
x'
= 2013
. MPa 70.13
σ y ' = −25 + 1915 . − 64.28 = −7013 . MPa MPa
τ x ' y ' = −25 sin 140 o + 100 cos140 o y'
= −92.67 MPa 70 o
x
20.13 MPa 92.67 MPa

4
SOLUTION (1.11)
40
(a) 50 30 τ xy = −30 MPa , σ y = 60 MPa
y
1
σx ∑F x = 0: σ x ( 1
2
) = 30( 1
2
) + 40( 1
2
) − 50( 1
2
)
1
2 or σ x = 20 MPa ( comp.)
o
45
x
70 MPa
σy
( b ) It may be seen from a sketch of Mohr’s circle that 30 MPa
θ p " = tan 1
2
−1 3
4 = 18.43 o
x'
σ1, 2 = −20+ 60
± [( ) + ( −30) ]
−20− 60 2 1
2 2
2 2
18.43o
x
= 20 ± 50
or σ1 = 70 MPa , σ 2 = −30 MPa

SOLUTION (1.12)

We have
σ x'
x’
τ x' y'
θ = 25 + 90 = 115o θ 10 MPa
σ x = −10 MPa o
25
σ y = 30 MPa y’
x

τ xy = 0 30 MPa
(a) σ x ' = 12 (σ x + σ y ) + 12 (σ x − σ y ) cos 2θ
= 12 (−10 + 30) + 12 (−10 − 30) cos 230o = 22.86 MPa
Thus,
σ w = σ x ' = 22.86 MPa

(b) τ x ' y ' = − 12 (σ x − σ y ) sin 2θ


= − 12 (−10 − 30) sin 230o = −15.32 MPa
So
τ w = τ x ' y ' = −15.32 MPa τw

SOLUTION (1.13)

(a) εx = α ΔT = 117 . (10 −6 )(80 o ) = 936 μ


Thus δ = ε x L = 936(10 −6 )250 = 0.234 mm
(b) εx = α ΔT = 117 . (10 −6 )( 80Lx ) = 9.36(10 −4 ) Lx
∫ ε dx = 4.68(10
−4
Hence u = ) xL + c
2

x (a)
We have u=0 at x=0; c=0. Equation (a) for x=250 mm gives δ = 0117
. mm

5
SOLUTION (1.14)

From solution of Prob. 1.11:


σ x = −20 MPa , σ y = 60 MPa , τ xy = −30 MPa
Thus, εx = 1
70 (103 )
[−20 − 0.3( 60)] = −543 μ , ε y = 1
70 (103 )
[60 + 0.3( 20)] = 943 μ
γ xy = − 30
70 ( 103 )
= −1114 μ
2.6

( a ) Equations (1.25) with θ = 180 − 60 = 120 o :


ε' x ' = −5432+943 + −5432−943 cos 240 o − 1114 sin 240 o
= 200 + 3715 . + 964.7 = 1536 μ
γ max = 2[( −5432−943 )2 + ( − 1114
2 ) ] = 1857 μ
2 1
2
(b)

SOLUTION (1.15)

ε1, 2 = 50+2250 ± [( 50−2250 ) 2 + ( − 150


2 )] = 150 ± 125
1
2
(a)
or ε1 = 275 μ , ε2 = 25 μ

( b ) Apply Hooke’s Law with σ z = 0:


275(10 −6 ) = 210(10
1
)
(σ1 − 0.3σ 2 );
9 57.75 = σ1 − 0.3σ 2
25(10 −6 ) = 1
210 (109 )
(σ 2 − 0.3σ1 ); 5.25 = σ 2 − 0.3σ1
Solving, σ1 = 6519
. MPa , σ 2 = 24.81 MPa

SOLUTION (1.16)

We have
(a) εx = 0.50006 = 120 μ ε y = − 0.25004 = −160 μ
γ xy = −1000 − 50 = −1500 μ rad
γ max = 2μ[( 120+2160 )2 + ( − 1500
2 ) ] = 1526 μ
2 1
2
(b)

SOLUTION (1.17)

(a) y
B

40
θ α = tan −1 43 = 5313
. o,
α
x θ = 126.87 o
30 D
Apply Eq. (1.25);
εx ' = 400+2800 + 400−2800 cos 2(126.87 o ) + 200
2 sin 2(126.87 ) = 560 μ
o

ε1, 2 = 400+2800 ± [( 400−2800 )2 + ( 200


2 ) ] = 600 ± 224
2 1
2
(b)
or ε1 = 824 μ ε2 = 376 μ
It may be seen from a sketch of Mohr’s Circle that;
θ p " = 12 tan −1 ( 12 ) = 13.28 o

6
SOLUTION (1.18)

(a) εx = 100(10
1
)
(150 + 903 ) = 1800 μ ,
3 ε y = 100(10
1
) 3 3 ) = −1400 μ
( −90 − 150
ε z = − 100(10 ) (150 − 90 ) = −200 μ
1
3
3

Δa = 1800μ (100) = 180 μm, Δb = −1400μ (50) = −70 μm,


Δt = −200μ (10) = −2 μm
Thus,
a' = 100018
. mm, b' = 49.93 mm, t ' = 9.998 mm

A' C' = [100.8 2 + 49.932 ] 2 = 111933


1
(b) . mm

SOLUTION (1.19)

( a ) We have σx = σ y = σz = − p , AC =111.8034 mm.


Equation (1.34),
ε x = ε y = ε z = − EP (1 − 2ν ) = − 100120 ( 1 ) = −400 μ
(10 ) 3 3 (a)
and
Δa = −400μ (100) = −40 μm, Δb = −400μ (50) = −20 μm
Δt = −400μ (10) = −4 μm
Hence
a' = 99.96 mm, b' = 49.98 mm, t ' = 9.996 mm
( b ) A' C' = [99.96 + 49.98 ] = 1117587
2 2 1
2
. mm

SOLUTION (1.20)

∠DAC = θ = tan −1 34 = 36.9o


LAC = LBD = 902 + 1202 = 150 mm G= E
2(1+1 3) = 83 E
ε x = E1 (150 + 753 ) = 175(10
6
)
E

ε y = E1 (75 + 150
6

3 )=
125(10 )
E

γ xy = 150
G =
400(106 )
3E

We have
θ1 = 180 − 36.9 = 143.1o
Taking x’ along BD
ε x ' = (25 + 150 cos 286.2o + 66.67 sin 286.2o ) 10E
6

= (25 + 41.85 − 64) 10E = 2.85(106 ) E


6

= 1.35 ×10−3 mm
6
ΔLBD = 100 2.85(10 )
210(109 )

7
CHAPTER 2

SOLUTION (2.1)

Free Body: CD
∑M C = 0: FAB = 1.7 P
C
and
σ AB = FAB
AAB 1m
1.7 m
FAB B
P
D
Substitute the numerical values:
50(106 ) = 1.7 P
500(10−6 )
, P = 14.71 kN

SOLUTION (2.2)
From geometry d+L
2a = da ; d=L
2a x
= xx ; wx = aL x
a w
L a
and Ax = wx t = atL x P wx P 0
Thus,
δ =∫ ∫
2L 2L
Pdx
Ax E = PL
atE
dx
x = PL
atE ln x|2LL L d
L L

= PL
atE ln 2

SOLUTION (2.3)
J = π2 ( 754 − 654 ) = 21661
. × 10 −6 m4 weld x'
c=0.075 m θ = 40 o
τ xy = TrJ , σ x = σ y = 0 θ
o x
Using Eq. (1.11a); 50
σ x ' = 0 + 0 + τ xy sin 2θ
or
200(10 6 ) = T = 58.65 kN ⋅ m
T ( 0.075 )( 0.985 )
21.661(10 − 6 )
;

SOLUTION (2.4)
. (10 −8 ) m4
J b = π2 ( 0.034 − 0.02 4 ) = 102102
Statics: Ts + Tb = 1 kN ⋅ m (a)
Geometry:
φc = =
Tb b Ts b
π π
2 ( 0.03 − 0.02 )( 42 ×10 ) )( 80×109 )
4 4 9 4
2 ( 0.02

or
Tb = 21328
. Ts (b)
From Eqs (a) and (b):
(CONT.)

8
(2.4 CONT.)
Tb = 680.8 N ⋅ m Ts = 319.2 N ⋅ m
It is required that, φA = φC + φAC :
0.01 = 1
J b Eb [680.8b + 1000( 0.5 − b)] = 500−319.2 b
102.102 (10 − 8 )( 42×109 )
Solving, b=0.223 m = 223 mm

SOLUTION (2.5)
State of pure shear; σ1 = −σ 2 = τ
ε max = E1 (σ1 − νσ 2 ) = τE (1 + ν )
Thus,
Eε max
τ= 1+ν = 120 (103 )(1900)
1+ 0.33 = 1714
. MPa
We have
J= = = 13127
150( 0.015 )
Tr
τ 171.4(106 )
. (109 ) m4
Hence
J = 32π (30 4 − d 4 ) = 13172
. (103 ) mm4
Solving, d = 28.68 mm

SOLUTION (2.6)

(a) τ max = 3V
2A = 3 pL 2
2 bh = 3 pL
4 bh (1)

σ max =
2 2
Mc
I = ( pL 8)( h 2)
bh3 12
= 3 pL
4 bh 2 (2)
Thus,
τ max σ max = h L (3)
Equation (3):
σ
L = h τ allall = 0.15( 1.59 ) = 0.9 m

( b ) Equation (1):
pall = 4 bh
3 L τ all = 4 0.05×0.15
3 0.9 (1.5 × 106 ) = 16.67 kN m

SOLUTION (2.7)
EIw IV = p = ( L2 − x 2 ), EIw' ' ' = ( L2 x − ) + c1
po po x3
L2 L2 3
Boundary Condition (B.C.):
w'''( L ) = 0; c1 = − 23 po L
EIw" = x2
− 12 ) + c1 x + c2
po 4

L2
( L2 2
x

B.C.:
po L2
EIw"( L ) = 0; c2 = 4

EIw" = (3L4 − 8 L3 x + 6 L2 x 2 − x 4 )
po
12 L2

EIw' = (3L4 x − 8 L3 x2
+ 6 L2 x3
− x5
) + c3
po
12 L2 2 3 5
B.C.:
EIw'( 0) = 0; c3 = 0.
(CONT.)

9
(2.7 CONT.)
EIw = 12 L0 2 (3L4 x2 − 4 L3 + 2 L2 − ) + c4
2
p x3 x4 x6
3 4 30

B.C.: w( 0) = 0; c4 = 0
po x 2
Thus, w= 360 EIL2
( 45L4 − 40 L3 x + 15L2 x 2 − x 4 )
At x=L;
19 po L4 po L3
wB = 360 EI θB = wB ' = 15 EI

SOLUTION (2.8)
pa
p
Refer to Table B.5 ( Case 5 and 7 ):
− pL3 pL ( 4 a 2 + L2 )
MB = θB = + =
2 MBL
Pa
2 24 EI 3 EI 24 EI
L
B C Deflection w1 of A due to only θB :
w A θB
paL ( 4 a 2 − L2 )
w1 = θB a = 24 EI

Table B.5 ( Case 2 with b=0 ), cantilever:


pL3 pL4
w2 = 24 EI (4 L − L) = 8 EI
Total deflection
w A = w1 + w2 = (3L4 + 4a 2 L − L3 )
pa
24 EI

SOLUTION (2.9)
EIw IV = p0 sin πLx ; EIw' ' ' = − po ( πL ) cos+ c1
EIw" = − po ( πL ) 2 sin πLx + c1 x + c2
B.C.:
w"( 0) = 0, c2 = 0; w' ' ( L ) = 0, c1 = 0
EIw' = po ( πL )3 cos πLx + c3
EIw = po ( πL ) 4 sin πLx + c3 x + c4
B.C.:
w( 0) = 0, c4 = 0; w( L ) = 0, c3 = 0
Thus
po L4
w= π 4 EI
sin πLx
po L3
Slope at x=0: θ A = w'1 ( 0) = π 3 EI
= −θB

SOLUTION (2.10)

x P
Symmetry
MA MB M A = − MB
A L C L
B
2 2 RA = RB = P2
Segment AC
EIw IV = 0, EIw' ' ' = c1 , EIw' ' = c1 x + c2
(CONT.)

10
(2.10 CONT.)

EIw' = 12 c1 x 2 + c2 x + c3
EIw = 16 c1 x 3 + 12 c2 x 2 + c3 x + c4 (a)
We have
EIw' ' ' (0) = c1 = −V = − P2
EIw' ' ( 0) = c2 = −( − M A ): c2 = M A
w' ( 0) = 0: c3 = 0, w' ( L2 ) = 0: M A = − M B = PL
8
w( 0) = 0: c4 = 0

Equation (a) is thus


w= Px 2
48 EI ( 3L − 4 x )

SOLUTION (2.11)

We have 250 2 = 125 MPa . Equation (2.30b) gives the limiting value of
pressure for the tangential stress as

p = σ allr t = 125×100.2(0.005) = 3.125 MPa


6

Note that, the axial stress formula, Eq. (2.30a) requires


σ all t
p=2 r = 6.25 MPa

SOLUTION (2.12)
1.2 kN/m
V = 3 − 15 . ) = 12
. (12 . kN
M = 3(15. ) − 12 (12 . ) 2 = 315
. )(15 . kN ⋅ m
M A
x Point A
V B
σ = McI = 0, σθ = pr
t = 4 (104 )( 0.5 )
0.005 = 4 MPa
z 1.5 m R=3 kN σ x = σ a = 2 MPa

Table B-4;
12000 ( πrt )( 2 r π )
τ= VQ
Ib = πr 3t ( 2 t )
= 153 kPa
Thus
τ max = [( 2−2 4 ) 2 + ( 0153
. ) 2 ] = 1012
1
. MPa 2

θs = 12 tan −1 2( 20−.153
4
) = −40.65
o

Point B;
σ= = = 8021 τ= =0
Mc 3150 ( 0.5 ) VQ
I π ( 0.5 )3 ( 0.005 )
. kPa , Ib

σ1 = σθ = 4 MPa , σ 2 = 2 + 0.802 = 2.802 MPa


and
τ max = 12 ( 4 − 2.802) = 599 kPa , θs = 45o

11
SOLUTION (2.13)

(a) σθ = pr
t A = 2π rt
= 2π (250)(10) = 15, 708 mm 2
A σθ = 4(106 )(0.25)
= 100 MPa
σa = + P Pr 0.01

σ x = 15,708(10
A 2t 3
500(10 )
)
+ 50 = 81.8 MPa
−6

(b) τ max = 12 [100 − 81.8] = 9.1 MPa


σ ' = 90.9 MPa

9.1 MPa
θ = 45o σ ' = 12 (σ θ + σ x ) = 90.9 MPa

SOLUTION (2.14)
At a point on circumference, we have
σa = = = 25 MPa , σθ = 50 MPa
pr 4 (100)
2t 2( 8 )
50 (103 )
σ= P
2πrt = 2π (100)( 8 )10 − 6
= 9.947 MPa
−30 (103 )( 0.1)
τ= Tr
= 2π ( 0.1)3 ( 0.008 )
= −59.68 MPa
σa
J
σ
Thus
σ1, 2 = ± [( 34.952 −50 ) 2 + 59.68 2 ] 2
34.95+50 1
2

τ = 42.48 ± 6012
.
σθ or
σ1 = 102.6 MPa , σ 2 = −17.64 MPa
(a) σ1 ≤ σ u ; 102.6 < 240 ∴ no failure
σ1 σ2
(b) σu = σ uc = 1; 102.6
240 − −17.64
600 =1
or
0.428+0.029 < 1 ∴ no failure
SOLUTION (2.15)

σ1, 2 = −100−50
± [( −1002+50 ) 2 + 30 2 ] 2
1
2

or
σ 2 = −36 MPa , σ 3 = −114 MPa , σ1 = 60 MPa
σu
(a) n= σ1 = 150
60 = 2.5
or n= 150
114 = 132
.
(b) 60
150 − −114
600 = 1n ; 0.4 + 019
. = 1
n
Solving n=1.7

12
SOLUTION (2.16)
At a point on the surface of the shaft, we have

J = 32π ( 75) 4 = 3106


. (10 6 ) mm4
A = π4 ( 75) 2 = 4.418(103 ) mm2
σ 40 (103 )
σ= P
A = 4.418 (10 − 3 )
= 9.054 MPa
τ 6 (103 )( 0.0375 )
τ= Tr
J = 3.106 (10 − 6 )
= 72.44 MPa
Thus
σ1, 2 = 9.054
2 ± [( 2 ) + 72.44 ] = 4.527 ± 72.581
1
9.054 2 2 2

σ1 = 7711 . MPa , σ 2 = −68.05 MPa

= [( 7711
. ) 2 − ( 7711
. )( −68.05) + ( −68.05) 2 ] 2
1
250
(a) n
or n=1.99

σ yp
(b) σ1 − σ 2 = n ; . + 68.05 =
7711 250
n
or n=1.72

SOLUTION (2.17)
Q p
Using equations of statics;
RA = + RB = +
pL Qb pL Qa
A B 2 L , 2 L
x D x' Then
L px 2 px '2
M AD = RA x + 2 , M BD = RB x '+ 2 ;
∂M AD ∂M BD
∂Q = bx
L , ∂Q = ax '
L
Applying Eq. (2.49):
[∫ ( Lx − x 2 )( bx )dx + ∫ ( Lx '− x ' 2 )( ax ' )dx ' ]
a b
wD =
p
2 EIL
0 0
Integrating, we have
wD = [4 L( a 2 + b 2 ) − 3( a 3 + b3 )]
pab
24 EIL

SOLUTION (2.18)
Q p
Segment AC
A B ∂M1
M1 = ( + 2 )x, =
pL Q x
C 8 ∂Q 2
pL
+
Q x x' 3 pL
+
Q
8 2 8 2 Segment BC
L/2 L/2 px '2 ∂M 2
M2 = ( + 2 ) x '− =
3 pL Q x'
8 2 , ∂Q 2
Let Q=0, Thus, Eq. (2.49):
L L

EIwC = ∫ dx + ∫ ( 3 8pL x '− px '3


) x2' dx ' = 165 pL4
2 pLx x 2

8 2 2 48
0 0
4
or wC = 5 pL
768 EI

13
SOLUTION (2.19)
P
(a) a We have
Q M AB = Px M BC = Pa
x B A
∫ M dx
∂M i
L x C δv = 1
EI i ∂P

[∫ ( Px )( x )dx + ∫ ( Pa )( a )dx ]
a L
= 1
EI
0 0
C
= 1
EI ( a3
3 + a L) =
2 Pa 2
3 EI ( a + 3L )
( b ) Add Q at A. Hence,
[∫ M AB dx + ∫ M BC
a ∂M AB L ∂M BC
δH = 1
EI ∂Q ∂Q dx ,
0 0
where M AB = Px, M BC = Pa + Qx
Let Q=0, Then
δ H = 0 + ∫ ( Pa )( x )dx =
L
PaL2
2 EI
0
( c ) Add C at A. We have

∫ M ∂ dx where M = Px + C ,
∂M i
θ= 1
EI i C AB M BC = Pa + C

[∫ ( Px + C )dx + ∫ ( Pa + C )dx ]
a L
θA = 1
EI
0 0
For C=0:
θA = ( a2 + aL ) = (a + 2 L)
2
P Pa
EI 2 EI

SOLUTION (2.20)

(a)
M M AC M AC = Rx + M , M BC = Rx + M − P( x − L2 )

∫M ∫M
∂M i ∂M i
A θ= 1
EI i ∂M dx , w = 1
EI i ∂R dx
R x
We have
L

{∫ ( Rx + M )dx + ∫L [ Rx + M − P( x − L2 )]dx}
L
θA =
2
1
EI
0 2
L
(a)
{∫ ( Rx + M ) xdx + ∫L [ Rx + M − P( x − )]xdx}
L
wA =
2
1 L
EI 2
0 2

Boundary conditions are θ ( 0) = 0 and w( 0) = 0 . Thus, after integrating Eqs.(a):


θA = RL2
2 EI + ML
EI − PL2
8 EI =0
wA = + − =0
3 2
PL ML 5 PL3
3 EI 2 EI 48 EI
From which
RL2 + 2 ML = PL4
2

RL3 + 23 ML2 = 165 PL3


Solving
M = − PL8 = PL
8 R= P
2
L L

wC = [∫ ( Rx + M ) xdx ] = [∫ ( P2 x −
2 2
1 1 PL
(c) EI EI 8 ) xdx ]
0 0

= 192PLEI = wmax
3

14
SOLUTION (2.21)
M MB M AC = RA x , M BC = RA x − M

∫ M ∂ dx
∂M i
A C B wA = 1
EI i RA
x
RA RB
[∫ ( R x )( x )dx + ∫ ( R
a 2a
a a wA = 1
EI A A x − M )( x )dx ]
0 a
3
= − 23 Ma
EI = 0
2
8 RAa
3 EI

from which RA = 169 Ma


Statics:
∑ F = 0:
y RB = − RA
∑ M = 0: B M B − M + RA ( 2a ) = 0
or
MB = − M
8 = M
8

15
CHAPTER 4

SOLUTION (4.1)

Flexural rigidity of the plate is


D= Et 3
12(1−ν 2 )
= 200×109 t 3
12(1− 0.09 ) = 18.315 × 109 t 3
Maximum deflection occurs at center (r=0). Equation (4.19) is thus,
3×106 ( 0.03 )4
wmax = 0.001 = 64×18.315(109 ) t 3
from which
t = 0.001275 m = 128
. mm

SOLUTION (4.2)

We have
200 (109 ) t 3
D = 12(1Et−ν 2 ) = = 18.315(109 )t 3
3

12 (1− 0.09 )

p0a 4 5+ν 3(106 )( 0.03 )4


wmax = 64 D 1+ν ; 1 × 10 −3 = 5.3
64 (18.315×109 ⋅t 3 ) 1.3
−3
Solving, t = 2.037(10 ) m = 2.04 mm

SOLUTION (4.3)

Cylinder end can be approximated as a clamped edge plate subjected to uniform loading.

( a ) Equation (c) of Sec. 4.5,


90 (106 )× 4
σ r ,max = ( at )2 = = (10.954)2
3 p0
4 ( at ) 2 ; 3×1(106 )
or a/t=10.954. Hence,
t= 250
10.954 = 22.8 mm

( b ) Then, Eq.(b) of Sec. 4.5, gives


= 1×6410× 200
( 0.25 ) ×12 (1− 0.3 )
p0a 4 6 4 2
wmax = 64 D ×109 ( 0.0228 )3
= 0.281 mm

SOLUTION (4.4)

From Fig.4.4, we have


σ 1 = σ r ,max = 34 p0 ( ar )2 , σ 2 = 14 p0 ( at ) 2

Equation (2.34b) gives


σ yp σ yp
σ1 = n ; 3
4 p0 ( ar ) 2 = n

Substituting the given values with σ yp = 250 MPa (Table B.3):

(0.6 × 106 )( 0.02


3
4 ) =
0.25 2 250×106
n
Solving, n = 3.56

SOLUTION (4.5)

25
Equation (e) of Sec. 4.5,
3( 3+ν ) p0
σ max = 8 ( at ) 2
Upon substitution of the given numerical values becomes
241(106 ) 3( 3.3 )× 0.5(106 )
n = 8 (10 2 ) = 61875
. × 10 6
From the above,
n = 3895
. ≈ 39
.
SOLUTION (4.6)

( a ) From Figure 4.4:


σ max = 34 p0 ( at ) 2 = 34 p yp ( 150
12 ) = 117.19σ yp
2

Setting σ max = p yp
σ
p yp = 117.19
yp
= 117.19
345
= 2.944 MPa
200(109 )(0.012)3
We have D = Et 3
= = 31.648 kPa
12(1−ν 2 ) 12(1− 0.32 )
Equation (b) of section 4.5 is then
p yp a 4 2.944(106 )(0.15)4
wmax = 64 D = 64(31.648×103 )
= 0.736 mm

( b ) pallow = = = 1.963 MPa


p yp 2.944
n 1.5

SOLUTION (4.7)

Expression (4.10b) becomes,


)]} = ( a 20D )r 3 = p1r 3
d d p
dr {r dr [ 1r d
dr (r dw
dr

where p1 = 0 a 2 D .
p

Successive integrations lead to


p1r 4 p1r 4
1 d
r dr [ 1r d
dr (r dw
dr )] = 4 + C1 ; 1 d
r dr (r dw
dr )= 16 + C1 ln r + C2
6
= + 12 C1r 2 ln r − 14 C1r 2 + C2 r 2 + C3
dw p1r
r dr 96
or
p1r 5
= + C1 ' r ln r + C2 ' r +
dw C3 '
dr 96 r
Then, in terms of new constants, displacement is found to be:
p1r 6
w= + c1 ln r + c2 r 2 ln r + c3r 2 + c4
576
In order w be finite at r=0 : c1 = c2 = 0.
Boundary conditions ( w=0 and dw/dr=0 at r=a ) give
p1a 6 6 c1a 5
0= 576 + c3a 2 + c4 , 0= 576 + 2c3a
from which,
3 p1q 4 2 p1a 6
c3 = − 576 , c4 = 576
Deflection is thus,
p1a 6 p0a 4
w= ( ar 6 − 3 ar 2 + 2) = [( ar ) 6 − 3( ar ) 2 + 2]
6 2

576 576 D Q.E.D.

SOLUTION (4.8)

26
Apply Eq.(4.26a):
r = 16
. ( 0.4t ) 2 + t 2 − 0.6757t = 0.446t = 0.015a
E ( a 30 )3
We have: D = Et 3
12 (1−ν 2 )
= 12 (1− 0.09 ) = Ea 3
294 , 840

wmax = 16PaπD 3+ν


= 14,809 aEP
2
and 1+ν

Equation(4.31); z = t
2 , since σθ ,max > σ r ,max :
σ max = 3 P( a / 30 )
2π ( a 30 )3
[13 a
. ln 0.015 a + 1 − 0.3] = 2,647 a 2
P

SOLUTION (4.9)

By Eq.(4.26b), re = rc = 0.6(15) = 9 mm. Since


σθ ,max > σ r ,max , by Eq.(4.29) at z = t 2 :
σ max = 3P
2π ( 0.015 )2 9 − 0.3] = 7627.7 P
. ln 180
[13
Thus, 7627.7 P = 60 × 10
6
or P = 7.87 kN

SOLUTION (4.10)

p0 = γh = 9.81(103 )50 = 490.5 kPa


3( 3+ν )
( a ) We have σ max = 8 p0 ( at ) 2 = 12(10 6 )
or
3( 3+ν ) p0 3( 3.3 )( 490.5×103 )
t=a 8 (12×106 )
= 014
. 8 (12×106 )
= 0.031 m = 31 mm
100 (109 )( 0.031)3
(b) D= 12 (1− 0.22 )
= 258.6 kN ⋅ m
4
490.5(103 )( 0.14 )4 5.2
We have wmax = 5+ν
= = 49.6(10 −6 ) m = 0.05 mm
p0a
12 (1−ν 2 ) 1+ν 64 ( 258 , 600 ) 1.2

SOLUTION (4.11)

Refer to Solution of Prob. 4.10:


p0 = γ h = 490.5 MPa

( a ) We now have (Fig. 4.4):


σ max = 34 p0 ( at ) 2 = 12(106 )
From which
3(490.5×103 )
t=a 3 p0
48(106 )
= 0.14 48(106 )
= 0.025 m = 25 mm

100(109 )
(b) D= = 135.634 kPa
12(1− 0.22 )
Hence, Eq. (b) of Sec. 4.5:
p0 a 4
wmax = 64 D = 490.5(103 )(0.14) 4
64(135.634×103 )
= 2.17(10−5 ) = 0.022 mm

SOLUTION (4.12)

27
210 (109 ) t 3
D= Et 3
12 (1−ν 2 )
= 12 ( 0.91) = 19.23(109 )t 3
( a ) Table 4.2 ( Case 1):
M 0a 2
wmax = 6(1 − ν )
500 ( 0.3 )2
Et 3
. (10 −3 ) = 6( 0.7) 210(109 ) t 3
; 15
or t = 8.434(10 −3 ) m = 8.43 mm

( b ) Table 4.2 ( Case 1 ):


σ max = ; 40(10 6 ) =
6 M0 6 ( 500 )
t2 t2
or t=10.95 mm

SOLUTION (4.13)

( a ) Equation (4.33): M 2 = 0, M r = M1 and


Mθ = − M1 aa 2 +− bb2 = −15
. 00..442 +− 00..332 = −5.37 kN
2 2 2 2

σ r ,max = = , σθ ,max = = − 32t,142


6 Mr 9 , 000 6 Mθ
t2 t2 t2 2

and τ max = 12 (σ1 − σ 2 ) = 1


2t 2
(9,000 + 32,142 ) = 20 , 571
t2
Thus,
180(10 6 ) = 32 ,142
t2
, t=13.36 mm
100(10 6 ) = 20 ,571
t2
, t=14.34 mm

70(109 )(14.34×10−3 )3
(b) D= 12(1− 0.09 ) = 18.914 kN ⋅ m
Equation(4.33) at r=b
2
wmax = 1 b2 − a 2 M1b
+ a 2b2 M1
2 a 2 + b2 (1+ν ) D a 2 − b2 (1−ν ) D
ln ab
= − ( 0.00549) + ( 0.2057)( 01133
1
2 . )( −0.28768 )
= −9.45 mm

SOLUTION (4.14)

We have
w = c1 ln r + c2 r 2 ln r + c3r 2 + c4 (r ≥ c)
Since w is finite at the origin,
w = c5 r 2 + c 6 (r ≤ c)

Boundary conditions:
w = 0, M r = 0 (r=a); ( w ) r ≤c = ( w ) r ≥c (r=c)
dw
( )
dr r ≤ c =( ) dw
dr r ≥ c (r=c); ( d 2w
)
dr 2 r ≤ c
=( d 2w
)
dr 2 r ≥ c
(r=c)
and
Qr = − P1 (r=c)

Six constants c1 , c2 , c3 , c4 , c5 , c6 are obtained from the above described 6 equations.


Result for r=0 and r=a listed in case 2 of Table 4.2.

SOLUTION (4.15)

28
We have
Qr (b) = − D ( 4bc2 + )=0
p0b
2D

{
M r (b) = − D −(1 −ν ) bc12 + 2c3 (1 + ν )

+ c2 [3 +ν + 2(1 + ν ) ln b] +
(3+ v ) p0b 2
16 D }=0 (P4.15a)
Likewise
p0 a 2
w(a ) = c4 + c1 ln a + c3 a 2 + c2 ln a + 64 D =0
{
M r (a ) = − D −(1 −ν ) a2 + 2c3 (1 + ν )
c1

+ c2 [3 +ν + 2(1 + ν ) ln a ] + (3+16v )Dp0 a


2

}=0 (P4.15b)

From these equations, we obtain for c4 , c1 , c3 , and c2 as:

p0 a 4 ⎧ (1 + ν ) ln ba ln a (5 −ν ) ln a
c4 = − ⎨ −
⎩ (1 −ν )( b ) [( b ) − 1] 4(1 −ν )( b )
a 2 a 2 a 2
4D
( a ) 2 ln a − ln b (3 + ν )[( ba ) 2 − 1] 1 ⎫
+ ba 2 a 2 − + ⎬
2( b ) [( b ) − 1] 8(1 + ν )( ba ) 2 16 ⎭

p0 a 4 ⎧ (1 +ν ) ln ba (3 + ν ) ⎫
c1 = ⎨ − a 2⎬
(P4.15c)
⎩ (1 −ν )( b ) [( b ) − 1] 4(1 −ν )( b ) ⎭
a 2 a 2
4D

p0 a 2 ⎧ ( ba ) 2 ln a − ln b (3 + ν )[( ba ) 2 − 1] ⎫
c3 = ⎨ − ⎬
8 D ⎩ ( ba ) 2 [( ba ) 2 − 1] 4(1 + ν )( ba ) 2 ⎭

p0b 2
c2 = −
8D
Substitution of these constants into Eqs. (4.13) and Eqs. (4.9), readily yields the
displacement and stress resultants.

SOLUTION (4.16)

From Eq.(4.12),
w = ∫ 1r ∫ r ∫ 1r ∫ Dr ( p0 ar )drdrdrdr
r
p0 a
p0
= c1 ln r + c2 r ln r + c3r + c4
2 2
r
∫ ∫ r∫ ∫ r
z
+
p0 1 1 2
aD r r drdrdrdr a

(CONT.)
(4.16 CONT.)

29
In order w be finite at r = 0, c1 = c2 = 0. Thus,
w = c3 r 2 + c 4 + ∫ ∫ rdr ∫
p0 3
dr dr
aD r r ( r3 )

= c3r 2 + c4 + ∫ ∫ rdr( r3
) = c3r 2 + c4 + ∫
p0 p0 5
dr dr r
aD r 9 aD r ( 45 )
= c3r 2 + c4 +
p0 5
r
aD ( 225 )
Boundary conditions, w=0 and Mr = 0 at r=a lead to :
p0a 2 p0 a 4
c3 = − 90 D ( 14++νν ), c4 = 45 D [ 2(14++νν ) − 15 ] Q.E.D.

SOLUTION (4.17)

*
Introduce given f n and f n ' s into Eq.(P4.17) to obtain the characteristic (or auxiliary)
equation for each case :
λ ( λ − 1)( λ − 2)( λ − 3) + 2λ ( λ − 1)( λ − 2) − (1 + 2n 2 )λ ( λ − 1) + (1 + 2n 2 )λ
+ n 2 ( n 2 − 4) = 0
from which
λ1 = n, λ2 = − n, λ3 = n + 2, λ4 = − n + 2 (a)
For n = 0, λ1 = λ2 = 0, λ3 = λ4 = 2. We know that ( for an equidimensional equation )
λ λ
a double root yields to the terms r and (ln r )r . Thus,
f 0 = A0 r 0 + B0 r 2 + C0 r 0 ln r + D0 r 2 ln r = A0 + B0 r 2 + C0 ln r + D0 r 2 ln r
Q.E.D.
For n = 1, we have one set of double roots : λ1 = λ4 = 1, λ2 = −1, λ3 = 3.
It follows that
f1 = A1r + B1r 3 + C1r −1 ln r + D1r ln r , f1* = A1*r + B1*r 3 + C1*r −1 + D1*r ln r
Q.E.D.
For n > 1, we have distinct roots and solution of Eqs.(P4.13) is :
f n = An r n + Bn r − n + Cn r n+ 2 + Dn r − n+ 2
Q.E.D.
f n* = An*r n + Bn*r − n + Cn*r n + 2 + Dn*r − n + 2

SOLUTION (4.18)

( a ) Expression (4.12) becomes


w = ∫ 1r ∫ r ∫ 1r ∫
rp0
D drdrdrdr
Thus, after integrations,
w = ∫ 1r ∫ r ∫ 1r [( ) + C1 ]drdrdr = ∫ 1r ∫ r[(
r 2 p0 r 2 p0
2D 4D ) + C2 ]drdr + C1 ln r
= ∫ 1r [ 16 D0 + C3 ]dr + C1r ln r +
r4 p C2r 2 r 4 p0 C2r 2
= + C4 + C3 ln r + C1 ( r2 ln r − r4 ) +
2 2

2 64 D 2

or
p0r 4
w= 64 D + c1r 2 ln r + c2 r 2 + c3 ln r + c4 Q.E.D.

(CONT.)
(4.18 CONT.)

30
( b ) Equation (4.10a) may be written
d 4w
+ 2 d 3w
− 1 d 2w
+ =
1 dw p0
dr 4 r dr 3 r 2 dr 2 r 3 dr D (a)
To ascertain the homogeneous solution, let
−x
r = e x , dx
dr
= e x , dx
dr = e
Hence,
= = e− x ; d 2w
= dr ) dr = e
−x −x
) = ( ddxw2 − )e −2 x
2
d
dw
dr
dw dx
dx dr
dw
dx dr 2 dx ( dw dx d
dx ( dw
dx e
dw
dx
−x
d 3w
= dr = e [( ddxw2 − )e −2 x ] = ( ddxw3 − 3 ddxw2 + 2 dw −3 x
2 2 3 2
d
dr 3 dx ( ddrw2 ) dx d
dx
dw
dx dx )e

d 4w
=
3
d
dr 4 dx ( ddrw3 ) dx
dr

= e−x [( ddxw3 − 3 ddxw2 + 2 dw −3 x


] = ( ddxw4 − 6 ddxw3 + 11 ddxw2 − 6 dw −4 x
3 2 4 3 2
d
dx dx )e dx )e
Introduction of the above derivatives into Eq.(a) leads to
d 4w
− 4 ddxw3 + 4 ddxw2 = 0
3 2

dx 4
(b)
λx
Denoting w = e , Eq.(b) is
λ4 − 4λ3 + 4λ2 = 0 or λ2 ( λ − 2) 2 = 0
from which
λ1, 2 = 0, λ3, 4 = 2
Thus, w = a + bx + ce + xde 2x 2x

where a , b, c and d are constants.


In as much as r = e ( or ln r = x ), homogeneous solution may be written in the form
x

wh = c1 + c2 ln r + c3r 2 + c4 r 2 ln r Q.E.D.
The particular solution of Eq.(a) can be obtained by the method of undetermined coefficients, by
using the ordinary procedure, as follows
p0r 4
wp = 64 D Q.E.D.
The general solution is thus w = wh + w p .

SOLUTION (4.19)

Using the expressions for ∂ 2 w / ∂x 2 , ∂ 2 w / ∂ y 2 in terms of polar coordinates (Sec. 4.2), we


∂ 2w ∂ 2w ∂ 2w 1 ∂w 1 ∂ 2w
have ∂x 2
+ ∂y 2
=∇ w= 2
∂r 2
+ r ∂r + r 2 ∂θ 2
Similarly, after carrying out some algebraic work:
∂ 2w ∂ 2w
− ( ∂∂x∂wy ) 2 = 1 ∂ 2 w ∂w
+ 1 ∂ 2w ∂ 2w
− ∂ w 2
r) − ( ∂∂θw ) 2 + 2 ∂ 2 w ∂w
2 2
1 1
∂x 2 ∂y 2 r ∂r 2 ∂r r 2 ∂r 2 ∂θ 2 r2
( ∂θ∂ r4 r 3 ∂θ∂r ∂θ
Equation (3.44) is therefore,
U= ∫∫ {( ∂∂ + 1r ∂w
+ 1 ∂ 2w 2
) − 2(1 − ν )[ ∂∂rw2 ( 1r ∂w
+ 1 ∂ 2w
2 2
D w
2 r2 ∂r r 2 ∂θ 2 ∂r r 2 ∂θ 2
)
A

− ( ∂∂r∂θw ) 2 − ( ∂∂θw ) 2 + 2 ∂ 2 w ∂w
]}rdrdθ
2
1 1
r2 r4 r 3 ∂r∂θ ∂θ
or
U= ∫∫ {( ∂∂ + 1r ∂∂wr + 1 ∂ 2w 2
) − 2(1 − ν ) ∂∂rw2 ( 1r ∂∂wr + 1 ∂ 2w
2 2
D w
2 r2 r 2 ∂θ 2 r 2 ∂θ 2
)
A

+ 2(1 − ν )( 1r ∂ 2w
∂r∂θ − 1 ∂w 2
r 2 ∂θ
) }rdrdθ Q.E.D.

SOLUTION (4.20)

31
Flexural rigidity is D= Et 3
12(1−ν 2 )
= Et 3
10.92

From expression (4.33):


M b2 − M a 2 a 2b2 M1 − M 2
w = − 2(1+ν1 ) D ( a 22 − b2 ) ( a 2 − r 2 ) + a 2 − b2 (1−ν ) D
ln ar (1)
or, simply :
w = − k1 ( a 2 − r 2 ) + k2 ln ar ( 1' )
For maximum deflection, dw / dr = 0, 2 k1r + k 2 / r = 0. Hence,
− k2
r 2 = − 2 k21 or r =
k
2 k1

Upon substitution of given data : r = 0.93a = 0.96a. This is the radius at which
2

maximum deflection occurs. Equation (1) is thus, for r = 0.96a:


0.0003329 M1a 2 M1a 2
wmax = D = 0.0036 Et 3

SOLUTION (4.21)

Deflection of the plate is given by Eq.(4.35). Maximum deflection (approximately) occurs near or
at r=b. Thus, substituting r=b, a=2b, and ν = 0.3 into Eq.(4.35):
3
wmax = {(1 − 14 )[ 23..36 − ln 12 ] + 14 ln 12 + . ) ln 12 ⋅ ln 12 }
Pa 1 2
1
8D 4 (1− 1 4 ) 4 (1− 1 4 ) (186
or
3
wmax = = 21113
1 ( 0.19334 )
Pa a 3 P
8D . 1
E (t)

SOLUTION (4.22)

The problem is solved, by applying the method of superposition, replacing original plate as shown
in Figs. 4.10b and 4.10c.
2πbP1 = πp0 b 2 P1 =
p0b
We have or 2

Edge moment, is M1 = 160 (3 + ν )( a − b ) = 0193


p 2 2 2
. p0 a .
Deflection for plate of Fig. 4.10b shown, from Eq.(4.22), is
p0a 4
wu = ( ar 4 − 5.077 ar 2 + 4.077)
4 2

64 D

Owing to the shear force P1 , deflection of plate shown in Fig. 4.10c, from Eq.(4.35) :
p a4
ws = − 1280 D (136
. − 136
. ar 2 + ar 2 ln ar − 0.343 ln ar )
2 2

Owing to the edge moment M1 , deflection of the plate in Fig. 4.10c is using Eq.(4.33) :
( r 2 − a 2 ) M1 a2 M
wm = − − 10.5 D1 ln ar = −[ r39−Da + 10a.5 D ln ar ]( 0193
2 2 2

39 D . p0 a 2 )

The deflection of original plate, by superposition:


w = wu + ws + wm
Maximum deflection occurs at r=b and is obtained as
p0a 4 p0a 4
wmax = 0.076 D = 0.83 Et 3
which is about the same as the result given in, Case 5, Table 4.3.

SOLUTION (4.23)

32
The original plate is replaced as shown in Figs. 4.10b, and 4.10c.
Deflection for plate of Fig. 4.10b, from Eq.(4.22), is
[ ar 4 − 2 ((13++νν )) ( ar 2 ) + ( 15++νν )] ,
p0a 4
wu = ν = 0.3,
4 2

64 D r=b=0.5a
p0 a 4 2.8701923 p0a 4 p0a 4
= 64 D [0.0625 − 12692308
. + 4.0769231] = 64 D = 0.0448468 D

=
p0b
Due to shear force P1 2 , deflection of plate shown in Fig. 4.10c, from Eq.(4.35):
Pr a 2b
ws = {(1 − )[ 2(13++νν ) − ln ab ] + [ ar 2 ln ar ] + [( a22 −bb2 )( 11+−νν ) ln ab ln ar ]}
2
r b2 2 2

4D a2 a 2 − b2
−0.03125 p0 a 4
= D {( 0.75)[12692308
. + 0.2310491] + ( −01732868
. )
+ [( 3 )(18571429
2
. )( −0.6931472)( −0.6931472)]}
−0.03125 p0 a 4
= D {11252099
. − 01732868
. + 0.5948466}
−0.03125 p0 a 4
−0.0483366 p0a 4
= D {15467697
. }= D
Due to edge moment M1 , deflection of plate in Fig. 4.10c, using Eq. (4.33):
M1b2 − M 2 a 2 M −M
wm = 12 [ ar 2 −−ab2 ][ ] + [ aa2 −bb2 ][ (1−1 ν ) D2 ]ln ar
2 2 2 2

(1+ν ) D

M 2 = 0,
M1 = − 160 (3 + ν )( a 2 − b 2 ) = − 160 (33
. )( a 2 )(1 − 0.25) = −01546875
p p
. p0 a 2
p0a 4 p0 a 2
wm = 12 [−1][−0.0297476 D ] + [ 13 a 2 ][−0.2209822 D ][−0.6931472]
4 4 4
= 0.0.48738 0D + 0.0510577 = 0.0659315
pa p0a p0a
D D
Superposition, w = wu + ws + wm :
p0a 4 p0a 4
w = [0.0448468 − 0.0483366 + 0.0659315] D = 0.0614417 D

D= Et 3
12 (1−ν 2 )
= Et 2
10.92

p0a 4 p0a 4
w = 0.0614417(10.92) Et 3 = 0.670 Et 3
4
wmax = k1 ( = 2 k1 = 0.664
p0 a a
From Case 5, Table 4.3: Et 3
) b
4
wmax = 0.664
p0a
Et 3

Comparison: − 0.664
{ 0.6700.664 } × 100 % = 0.9 % Difference between derived and tabulated values.

SOLUTION (4.24)
r = eα , α = ln r
dθ dθ dt 1 dθ d 2θ
dr = dt dr = r dt , = ( ddtθ2 − dθ
2
1
dr 2 dr 2 dt )
Substituting into Eq.(j) of Example 4.4, we have
pb3e −2 t
d 2θ
dt 2
+ 3 ddtθ + (3ν − 1)θ = − 2πD1
The auxiliary equation of this is
m2 + 3m + (3ν − 1) = 0, where m1, 2 = −15
. ± 3.25 − 3ν
and θh = c1r m + c2 r m 1 2

(CONT.)
(4.24 CONT.)

33
We have
θ p = Ae −2 t , dθ
dt = −2 Ae −2 t , d 2θ
dt 2
= 4 Ae −2 t
pb3
4 A − 6 A + 3νA = − 2πD1 , A = − 6πD (ν −1)
pb3
Hence,

θ p = − 6πDpb(ν −1)
3
and 1

Thus, θ = θh + θ p Q.E.D.

SOLUTION (4.25)

Variation of flexural rigidity of the plate can be written as


Et02
D= Et 3
12 (1−ν 2 )
= 12 (1−ν 2 )
(1 − dr )3 = D0 (1 − dr ) 2 (a)
Using Eq.(a) with ν= 1
3 ,λ = r
d , and Qr = 0, Eq.(4.18) becomes
λ2 (1 − λ )3 d θ
+ λ (1 − 4λ )(1 − λ ) 2 dθ
− (1 − λ )2 θ = 0
2

dλ2 dλ
The solution of this is
θh = c1 ( 1+λ2 λ ) + c2 [ 3(1λ−−λ2)λ ]
2
2

that may be rewritten as Eq.(P4.25), by inserting λ= r


d .

SOLUTION (4.26)
Strain energy expression (P4.19) becomes, by taking w = c0 ( a 2 − r 2 ):
U = πD ∫ [( −2c0 − 2c0 )2 − 2(1 − ν )( −2c0 )( −2c0 )]rdr
a

= πD ∫ [16c02 − 5.6c02 ]rdr = 5.2πDa 2 c02


a
(a)
0
The work done by the load is given by

∫ ∫ p wrdrdθ = 2πcc ∫
a a πc0a 4
W= 0 0 0 p0 ( a 2 − r 2 )rdr = 2 (b)
0 0
Here Π = U − W. The Ritz method yields
∂Π πp0a 4
∂ c0 = 10.4πDa 2 c0 − 2 =0
2
c0 = 200.8 D
pa
from which
Hence, w = c0 ( a − r ) gives deflection at any point. Maximum deflection, occurring
2 2

p0a 4
at r=0, is wmax = 20.8 D

SOLUTION (4.27)

Strain energy is already obtained in solution of Prob. 4.26.


The work done by load is W = Pwr =0 = Pc0 a 2 . Hence,
Π = U − W = 5.2πDa 2 c02 − Pc0 a 2
dc0 = 10.4πDa c0 − Pa = 0; c0 = 10.4PπD
dΠ 2 2
Then,
The deflection is given by w = c0 ( a 2 − r 2 ). The maximum displacement is thus,
wmax = Pa 2
10.4πD
SOLUTION (4.28)

34
P
Let w = A + Br 2 + Cr 4 (a) a t
r
We have,
W = P⋅ A (b) z

∫ ∫
a (1−ν ) dw d 2 w
U1 = + ) −2 ]rdrdθ
2
D d w 1 dw 2
2 [( dr 2 r dr r dr dr 2 (c)
0 0
π
=∫ ∫
2 a
U2 1
2 kw 2 ⋅ rdrdθ (d)
0 0
Substitution of Eq.(a) into Eqs.(c) and (d), after integration, yields
U1 = πD[4 B 2 a 2 (1 + ν ) + 16 BCa 4 (1 + ν ) + 16 C3 a (5 + 3ν )]
2 6

U 2 = πk[ A 2a + + + + +
2 2
B2a 6 C 2 a10 ABa 4 BCa 8 ACa 6
6 10 2 4 3 ]
The potential energy is
Π = U1 + U 2 − W
Application of the Ritz method
∂Π ∂Π ∂Π
∂A = 0, ∂B = 0, ∂C =0
results in three equations. From these equations, we obtain:
A= P
πka 2
− Ba 2
2 − Ca 4
3 (1)

B[8 D(1 + ν ) + ka 4
12 ] + C[16 Da 2 (1 + ν ) + ka 6
12 ] = − 2Pπ (2)
B[16 D(1 + ν ) + ka 4
12 ] + C[ Da (5 + 3ν ) +
32
3
2 4 ka 6
45 ]= − P
3π (3)
Substituting the given data into Eqs.(1) to (3) and solving :
A = 9.072 × 10 −9 P, B = 2.676 × 10 −8 P, C = 3.068 × 10 −8 P
The maximum deflection occurring at center (r=0) is :
wmax = 9.072 × 10 −9 P.

SOLUTION (4.29)
D = 12(1Et−ν 2 ) = 146.52 N ⋅ m , m = ( 7.86 × 10 6 )( 2 × 10 −3 ) = 15.72 kg / m2
3

β = 0.6689( 7.186 )( 2×010.1 ) = 4.26 −3

fn = 1.625
( 0.1)2 1+ 4.262
146.52
15.72 = 216.4 Hz
Thus, e = 1−( f1 2 = 1−(181 2 = 1.007
fn ) 216.4 )

p d = m g in ⋅ e = (15.72)(3 × 9.81)(1.007) = 465.9Pa


pd a 4 465.9 ( 0.1)4
wmax = 64 D = 64 (146.52 ) = 4.9(10 −3 ) mm
and σ r ,max = 43 pd ( at ) = 43 ( 465.9 )(50) 2 = 873 kPa

SOLUTION ( 4.30)

, and ∂ w ∂r 2 to be finite at r=0, we must have in Eqs.(4.8):


∂w 2
In as much as w, ∂r

Bn = Dn = 0 ( n = 1,2,⋅ ⋅ ⋅ ⋅ ⋅). Thus,



w= ∑(A r
n = 0 ,1,⋅⋅⋅⋅
n
n
+ Cn r n+ 2 ) cos nθ (a)

(CONT.)
(4.30 CONT.)

35
Boundary conditions, at r=a, are :
w = 0, − D[ 1r ∂w
∂r + 1 ∂ 2w
r 2 ∂θ 2
+ν ∂ 2w
∂r 2
] = Cn cos nθ (b)
From the first of Eqs.(b),
An = − Cn a 2
The second of Eqs.(b) yield then the Cn . Upon introducing the Cn , obtained this way,
and An into Eq.(a) we obtain the result given by Eq.(P4.30).

SOLUTION (4.31)

Substituting, (n=0, 1), Eqs.(3.10), (P4.26), and (4.2) into Eq.(4.3):


σ r = − 1−Ezν [ ∂∂rw + ν ( 1r ∂∂wr + 1 ∂ 2w
2
2 2
r 2 ∂θ 2
]
= 6z
t3
{1+1ν ( 2 M 0 + 2νM 0 ) + 1
a ( 3+ν ) [6 M1r cos θ + νr (3 M1r 2 − a 2 ) cos θ
− ν
r2
( M1r 3 − a 2 r ) sin θ ]}
or
σr = 6z
t3
[2 M 0 + 3 M1 r 2+ν
a 3+ν cos θ − νra cos θ −sin θ
3+ν − M1 3+νν ar sin θ ]

SOLUTION (4.32)

Introduction of Eqs.(a) and (P4.32) into Eq.(4.5) lead to



+ ∑ [ dr 4n +
d 4 F0 3 2 3
d 2 Fn
+ − + − 1+r22n
2
2 d F0 1 d F0 1 dF0 dF 2 d Fn
dr 4 r dr 2 r 2 dr 2 r 3 dr r dr 3 dr 2
1

n2 ( n2 − 4 )
Fn ]cos nθ + ∑ [
d 4 Gn 3
d 2 Gn
+ 1+r23n + + − 1+r22n
2 dFn 2 d Gn 2

dr r4 dr 4 r dr 3 dr 2
1
n 2 ( n 2 − 4 ) Gn
+ 1+ 2 n 2 dGn
r3 dr + r4
]sin nθ
∞ ∞
=
P0 ( r )
D + 1
D ∑ Pn cos nθ +
1
1
D ∑R
1
n sin nθ
In order the above be valid for all values of r and θ it has to be written in the form given
by Equation (b) of Problem 4.32.

SOLUTION (4.33)
p0
Assume that maximum
deflection occurs at the center
of the plate. Using the Reciprocity r
theorem : P1 w21 = P2 w12 .
z
Here
P2 = 1, P1 = p0 + p1 ar cos θ , w12 = wc , and w21 is given by Eq.(4.30).
Thus,

∫ ∫
a
wc = [2r 2 ln ar + 13++νν ( a 2 − r 2 )]rdrdθ + 0
p0
16πD
0 0

∫ [2r
a p0 a 2
= ln ar + 13++νν r ( a 2 − r 2 )]dr = [2 13++νν − 1]
p0 3
8D 64 D
0
For ν = 0.3, the above reduces to the result given by Eq.(d) of Sec.4.5.

SOLUTION (4.34)

36
Apply the Reciprocity
theorem Pw1 21 = P2 w12 ,
where, P P1 = 1
P1 = P, P2 = 1, w21 = w A , w12 = wc r
A b a
Thus, using Eq.(4.28) :
wc = P[ 161πD ( 2r 2 ln ar + a 2 − r 2 )]r =b z
= 16PπD ( 2b 2 ln ab + a 2 − b 2 ) = 16PbπD [2 ln ab + ( ab2 − 1)]
2 2

Note that if b=0, the above reduces to the result (c) of Sec.4.8.

SOLUTION (4.35)

Following the procedure of the P P2 = 1


solution of Prob.4.34,
and using Eq.(4.30) : r
b a
z
wc = P
16πD [2r ln +
2 r
a
3+ν
1+ν ( a − r )]r =b =
2 2 Pb2
16πD [2 ln +b
a
3+ν
1+ν ( a2
b2
− 1)]
Clearly, when b=0, the above reduces to the result (e) of Sec.4.8.

37
CHAPTER 3

SOLUTION (3.1)

Let r = ry , then rx = rxy = ∞ , M = My, M x = M xy = 0


Hence, Eq.(3.7), for z= − t
z :
70 (109 )( 0.005 )
σ max = Et
2 (1−ν 2 ) r
; 96(10 6 ) = 2 ( 0.91) r , or r=2 m, D= 4 m
Equation (3.11),
96(10 6 ) = = M max = 400 N
6 M max 6 M max
t2 ( 0.005 )2
;

SOLUTION (3.2)

Using Eq.(3.3b)
εmax = rz = t
2r = 0.5
2 ( 250 ) = 1000(10 −6 ) = 1000 μ
From Eq.(3.7), with z= − t
2 ;
210 (109 )( 0.5×10 −3 )
σ max = Et
2 (1−ν 2 ) r
= 2 ( 0.91)( 0.25 ) = 230.8 MPa

SOLUTION (3.3)

Referring to Mohr’s circle, we write


κ x = 12 (κ1 + κ 2 ) + κ12 cos θ1 x
y'
κ y = 12 (κ1 + κ 2 ) − κ12 cos θ1 κ2 θ1 κ1
κn
and θ2
κ x' = 12 (κ1 + κ 2 ) + κ12 cos θ2 x'
κ y' = 12 (κ1 + κ 2 ) − κ12 cos θ2 y
κ nt κ12
Adding,
κ x + κ y = κ x ' + κ y ' = κ1 + κ 2 = ∇ 2 w Q.E.D.
where,
κ x = ∂ 2 w / ∂x 2 and κ y = ∂ 2 w / ∂y 2 .

SOLUTION (3.4)

From Fig. 3.3,


κ x −κ y 2
κ1, 2 = 12 (κ x + κ y ) ± ( 2 ) + κ xy
Hence,
(κ1 + κ 2 ) 2 = κ12 + 2κ1κ 2 + κ 22 = κ x2 + 2κ xκ y + κ y2
(κ1 − κ 2 ) 2 = κ12 − 2κ1κ 2 + κ 22 = (κ x2 − 2κ xκ y + κ y2 ) + 4κ xy2
It follows that,
(κ1 + κ 2 ) 2 − (κ1 − κ 2 ) 2 = 4κ1κ 2 = κ x2 + 2κ xκ y + κ y2 − κ x2 + 2κ xκ y − κ y2 − 4κ xy2
or
κ xκ y − κ xy2 = κ1κ 2 Q.E.D.

16
SOLUTION (3.5)

At θ = 0, we have
Mx = M y = 0 M xy My = 0
For any angle (θ = α ), Eqs.(3.12)
x
become α
M x ' = 0 = 0 + 0 + M xy sin 2α (1) θ
M x ' y ' = 0 = 0 + M xy cos 2α (2) x'
y
Thus, Eq.(1) result in
for θ ≠ 90 o , sin 2α ≠ 0 ∴ M xy = 0
at θ = 90 o , sin 2α = 0 ∴ M xy = const.
Equation (2) implies also that
at θ = 90 o , cos 2α = −1 ∴ M x ' y ' = − M xy as required (see Fig. 3.6).

SOLUTION (3.6)

200 (109 )(12×10 −3 )3


D= Et 3
12 (1−ν 2 )
= 12 (1− 0.32 )
= 3165
. × 103 N ⋅ m
( a ) Equation(3.37), for x = y = 0;
p0a 4 5(106 )( 0.14 )4
wmax = 64 D = 64 ( 31.65×103 )
= 0.948 mm

( b ) Let W = a − 3x − y . Then
2 2 2

∂w ∂W
w =W2 = 2 640D W = − 160D ( xa 2 − x 2 − xy 2 )
p0 p p
64 D , ∂x ∂x
∂ 2w ∂ 2w ∂ 2w
= − 160D ( a 2 − 3x 2 − y 2 ), = − 160D ( a 2 − x 2 − 3 y 2 ) , =
p p p0 xy
∂x 2 ∂y 2 ∂x∂y 8D

Thus, at x = y = 0 and z= t
2 ;
2
εx = ε y = ( 2t ) = 5(106 )( 0.14 )2 ( 0.006 )
= 1161 μ
p0a
16 D 16 ( 31.65×103 )

γ xy = 0, ∂ w
=0
2
since ∂x∂y

( c ) Equation (3.7) for z = t


2;

σx = [a 2 (1 + ν ) − x 2 (3 + ν ) − y 2 (1 + 3ν )]
Ezp0
(1−ν 2 )16 D

= [a 2 (1 + ν ) − x 2 (3 + ν ) − y 2 (1 + 3ν )]
3 p0
8t 2
Substituting the data given
σ x = 1302
. (1010 )[13 . a 2 − 33
. x 2 − 19
. y2]
For x = a = 0.14 m and y = 0: σ x ,max = −510.4 MPa

SOLUTION (3.7)
D= Et 3
12 (1− 19 ) = 3
32 ( 200 × 109 )(10 × 10 −3 )3 = 18,750 N ⋅ m
5 p0b4 5(10×103 )( 0.4 )4
wmax = 384 D = 384 (18, 750 ) = 018
. mm
(CONT.)

17
(3.7 CONT.)
σ y ,max = 43 p0 ( bt ) 2 = 43 (10 × 10 3 )( 40) 2 = 12 MPa , σ x ,max = 4 MPa
ε y ,max = E1 (σ y ,max − νσ x ,max ) = 106
200 (109 )
. μ
(12 − 43 ) = 533
t
Equation (3.3b), for z= 2 ;
ry = t
2 ε y ,max = 0.01
2 ( 53.3×10 − 6 )
= 938
. m
We thus have wmax / t = 0.018 and b = 234.5. These results show that the
ry

deflection curve extremely flat, as usually is the case for small deflections.

SOLUTION (3.8)
πy πy
(a) Dw IV = p0 sin b Dw'' = −( πb ) 2 p0 sin b + c1 y + c2
πy πy
Dw''' = −( πb ) p0 cos b + c1 Dw' = ( πb )3 p0 cos b + 12 c1 y 2 + c2 y + c3
Thus
πy
Dw = ( πb ) 4 sin b + 16 c1 y 3 + 12 c2 y 2 + c3 y + c4 (a)
Boundary conditions:
w' ( 0) = 0: c3 = −( πb )3 p0 ; w( 0) = 0; c4 = 0
w( b) = 0: c2 = − 13 c1b + π23 p0 b 2
w' ( b) = 0: c1 = 0 ∴ c2 = 2
π3
p0 b 2
Equation (a) gives then
p0 b4 πy π π
w= Dπ 2
[sin b + b2
y2 − b y] (b)

p0b4
( b ) For y= − b
2 : wmax = π 4D
(1 − π4 )
2 p0 b2 p0 tb2
εmax = z ddyw = 2t [0 + 0 + ]=
2
At y= 0: 2
π D
3
π 3D

= d 2w
= [−( πb ) 2 sin π3 + 2 b2
1 p0
( c ) At y= b 3 : ry dy 2 D π3
]
We have
200 (109 )(15×10−3 )3
D = 12(1Et−ν 2 ) = = 63,28125
. N ⋅m
3

12 (1− 19 )

Thus
50 (103 )( 0.6 )2
1
ry = 63, 281.25 [− π12 sin 60 o + π23 ]
or ry = −1512
. m
The minus sign means that ry is measured in negative z direction, as intuitively expected.

SOLUTION (3.9)

(a) Dw IV = p0, Dw' ' ' = p0 y + c1 , Dw' ' = 1


2 p0 y 2 + c1 y + c2
Dw' = 1
6 p0 y 3 + 12 c1 y 2 + c2 y + c3
and

(CONT.)

18
(3.9 CONT.)
Dw = 1
24 p0 y 4 + 16 c1 y 3 + 12 c2 y 2 + c3 y + c4 (a)
Boundary conditions:
w( 0) = 0: c4 = 0, w' ' ( 0) = 0: c2 = 0
p0b4
w( b) = 0: + c1 + c3b = 0
3
b
24 6
p0b3
w' ( b) = 0: + c1 + c3 = 0
2
b
6 2

Solving, c1 = − 83 b c3 =
3
b
48
Equation (a) is thus
p0b4
w= [( b ) − 3( b )3 + 2( b ) 4 ]
y y y
48 D (b)
( b ) We have
p0 b4 4 y3 9 y2
= [ b4 − + d 2w
= [y2 −
dw 1 p0 3 yb
dy 24 D 2 b3 2b ], dy 2 2D 4 ]
At y = b:
M max = − D ddyw2 = − p0 σ y ,max = = −0.75 p0 ( bt )2
2
b2 6 M max
8 , t2

( c ) For ν= 1
4 , D= Et 3
12 (1− 116 ) = 4 Et 3
45

At y= b 2 ,
p0 b2 p0b2
ε y ,max = z ddyw = 2t ( 16 D ) = 128
2
45
2
Et 2

SOLUTION (3.10)

(a) Dw IV = p0, Dw' ' ' = p0 y + c1 , Dw' ' = 1


2 p0 y 2 + c1 y + c2
Dw' = 1
6 p0 y 3 + 12 c1 y 2 + c2 y + c3
and
Dw = 1
24 p0 y 4 + 16 c1 y 3 + 12 c2 y 2 + c3 y + c4 (a)
Boundary conditions:
w( 0) = 0: c4 = 0, w'( 0) = 0: c3 = 0
3
w' ( b) = 0: + c1 + c2 b = 0
p0b b2
6 2
p0b4
w( b) = 0: + b6 c1 + 22 c2 b 2 = 0
3 c
24

Solving, c1 = − 12 p0 b c2 = 121 p0 b
2

Equation (a) is thus


p0b4
w= [( b ) 4 − 2( b )3 + ( b )2 ]
y y y
24 D (b)
p0b4
( b ) For y= b
2 : wmax = 384 D
p0b2 p0 b2t
y = 0: ε max = z ddyw = 2t ( )=
2
At 2 12 24 D

( c ) At y = b
3 :
p0 b2 p0 b2
1
ry = d 2w
dy 2
= 24 D [ 129 − 4 + 2] = − 361 D

Substituting the given numerical values,


( 50×103 )( 0.6 )2
1
ry = − 361 63, 281.25 ; ry = −126.6 m.

19
SOLUTION (3.11)

Dw IV = y , Dw' ' ' = y 2 + c1 , Dw' ' = y 3 + c1 y + c2


p0 p0 p0
(a) b 2b 6b

Dw' = y 4 + 12 c1 y 2 + c2 y + c3
p0
24 b

Dw = y 5 + 16 c1 y 3 + 12 c2 y 2 + c3 y + c4
p0
120 b (a)

Boundary conditions:
w( 0) = 0: c4 = 0, w' ' ( 0) = 0: c2 = 0
3
p0b4
w' ( b) = 0: 240 + b2 c1 + c3 = 0 , w( b) = 0: + c1 + c3b = 0
2
pb b2
120 6

Solving, c1 = − 101 p0 b c3 = 120


1
p0 b 3
Thus
w = 1200Db [ y 5 − 2b 2 y 3 + b 4 y ]
p
(b)

p0b3
( b ) At y = 0: θ = w'( 0) = 120 D (c)

( c ) At y = b:
M max = D ddyw2 = b 2 − 101 p0 b 2 =
2 p0
6
1
15 p0 b 2
and
σ y ,max = 6 M max
t2
= 2
5 p0 ( bt ) 2 = 52 (100 × 103 )(50)2 = 100 MPa
Check:
p0b2
d
dy ( w' ' ) = 0: 60 y12 = 12b 2 or y1 = b
5
, and M y1 = 15 5
< M ( b).

SOLUTION (3.12)

Boundary conditions are


w = 0, ∂∂wx = 0 (x=0, x=a) (1)
w = 0, ∂∂wy = 0 (y=0, y=b) (2)
( a ) We have
y4 2 y3 y2
w = c0 ( ax 4 − + )( b4 − +
4
2 x3 x2
a3 a2 b3 b2
) (a)
4 3 2
∂w
= c0 ( 4ax4 − 6ax3 + )( b4 − +
3 2 y 2y y
2x
∂x a2 b3 b2
) (b)
3 2
∂w
= c0 ( ax 4 − 2ax3 + − +
4 3 2 4y 6y 2y
x
∂y a2
)( b4 b3 b2
) (c)
It is seen that conditions(1) and (2) are satisfied by Eqs.(a) to (c).

( b ) The second order derivatives are :


∂ 2w y4 2 y3 y2
= c0 ( 12ax4 − 12a 3x + )( b4 − +
2
2
∂x 2 a2 b3 b2
) (d)
2
∂ w
= c0 ( ax 4 − + − +
2 4 3 2 12 y 12 y
2x x 2
∂y 2 a3 a2
)( b4 b3 b2
) (e)
∂ 2w 4 y3 6 y2
= c0 ( 4ax4 − + − +
3
6 x2 2x 2y
∂x∂y a3 a2
)( b4 b3 b2
) (f)

Upon substitution of Eqs.(d) to (f) into (3.7), we obtain


(CONT.)

20
(3.12 CONT.)
σ x = − 1Ezc ) + ν ( ax 4 −
4
2 y3 y2 12 y 2
[( 12ax − 12a x + a2 )( b − + + − + b22 )]
2 4
y 2 x3 x2 12 y
−ν
0
2 4 3 2 4
b3 b2 a3 a2
)( b4 b3
y4 2 y3 y2
σ y = − 1Ezc [( ax − 2ax + ax )( b − b + b2 ) + ν ( 12ax − 12a x + a2 )( b −
2
+
12 y 4
12 y 3 2 2

−ν
0
2 4 3 2 4 3 2 4 3 2 4
b3 b2
)]
τ xy = − 1Ezc
3 2
[( 4ax − 6ax + 2a x )( b − b + b )]
4y 6y3 2y 2
0
−ν 2 4 3 2 4 3 2

At center of plate surface (x=y=a/2, z=t/2):


σx = σ y = τ xy = 0.
Etc0
32 a 2 (1−ν )
,

SOLUTION (3.13)

We have
πy ∂w πy
w = c( ax ) 2 sin b , ∂x = 2c
a2
x sin b
∂w cπ πy ∂ w πy
= =
2

∂y a 2b
x 2 cos b , ∂x 2
2c
a2
sin b
∂ w πy πy
= − ac2πb2 x 2 sin ∂ w
= 2 cπ
2 2 2

∂y 2 b , ∂x∂y a 2b
x cos b

∂ w πy
∂ w
= 0, = − 2ac2πb2 x sin
3 3 2

∂x 3 ∂x∂y 2 b
∂w
( a ) At x=0: w = 0, ∂x =0 satisfied
At x=a: ∂ 2w
∂x 2
+ν ∂ 2w
∂y 2
≠0 not satisfied
∂ 3w
+ ( 2 − ν ) ∂∂x∂wy 2 ≠ 0
3

∂x 3
not satisfied
∂ w ∂ w
= 0. =0
2 2
At y=0: w=0, ∂y 2
At y=b: w=0, ∂y 2
satisfied

( b ) For z = t
2 , a = b, ν = 0.3, x = y = a 2 :
ε x = − z ∂∂xw = − act ε y = − ∂∂yw = π 2ct
γ y = −2 z ∂∂x∂wy = 0
2 2 2
2 2 , 2
8a2
,

SOLUTION (3.14)

w = c( ax ) 2 sin πay ∂w
∂x = 2c
a2
x sin πay
∂w
∂y = cπ
a3
x 2 cos πay ∂2w
∂x 2
= 2c
a2
sin πay
∂2w
= − caπ4 x 2 sin πay ∂2w
= 2 cπ
x sin πay
2

∂y 2 ∂x∂y a3
∂3w
∂x3
=0 ∂2w
∂x∂y 2
= 2 cπ 2
a4
x sin πay

( a ) At x=0: w = 0, ∂w
∂x =0 satisfied
∂ w
+ν ∂ w
≠0
2 2
At x=a: not satisfied
∂x 2 ∂y 2
∂3w
+ (2 −ν ) ∂∂x∂wy 2 ≠ 0
3
not satisfied
∂x3

At y=0: w = 0, ∂2w
=0
∂y 2
both satisfied
At y=b: w = 0, ∂ w
=0
2

∂y 2

21
SOLUTION (3.15)

C W = 1 − ax − a .
y
B Let
x
a Then w = cx y W .
2 2

a ∂w
y=a−x ∂x = 2cxy W + 2cx y W ( − a )
2 2 2 2 1

A
y
∂ 2w
∂x∂y = 4cxyW + 4cxy 2W ( − a1 ) + 4cx 2 y ( − a1 ) + 2cx 2 y 2W −1 ( − a1 )
∂w
∂y = 2cx 2 yW + 2cx 2 y 2W ( − a1 )
∂ 2w
∂x 2
= 2cy 2W 2 + 4cxy 2W ( − a1 ) + 4cxy 2 ( − a1 ) + 2cx 2 y 2W −1 ( − a1 ) 2
∂ 2w
∂y 2
= 2cx 2W 2 + 4cx 2 yW ( − a1 ) + 4cx 2 y ( − a1 ) + 4cx 2 y 2W −1 ( − a1 ) 2

∂w
( a ) At x=0: w=0, ∂x =0
∂w
At y=0: w=0, ∂y =0
∂w ∂w
At y=a-x: w=0, ∂x = 0, ∂y =0
( b ) At x=0, y=a:
σ y = − 2(1Et−ν ) [ ∂∂yw + ν ∂∂xw ] = ⋅ ⋅ ⋅ ⋅ ⋅ ⋅ ⋅ = 0 , τ xy = ∂ 2w
= ⋅⋅⋅⋅⋅⋅⋅ = 0
2 2
E
2 2 2 2 (1+ν ) ∂x∂y

∂ w ∂ w ∂ w
x = a2: = = 0, =0
2 2 2 2
ca
At y=0, ∂y 2 8 , ∂x 2 ∂x∂y

and
σ y ,max = − 2(1Et−ν ) [ ca8 ] = − 16Ecta τ xy = 0
2 2
2
(1−ν 2
)
,

SOLUTION (3.16)

We have
M xy = − D(1 − ν ) ∂∂x∂wy = M 0
2

∂ 2w − M0
Let, ∂x∂y = D (1−ν ) =k
Integrating with respect to x :
∂w
∂y = kx + f ( y ) + c1
Then, integrating the above with respect to y gives
w = kxy + ∫ f1 ( y )dy + c2 where f1 ( y ) = f ( y ) + c1
Due to the symmetry in deflection : ∫ f ( y )dy = 0.
1

Also, owing to the symmetry, center (a/2, a/2) should be free of displacement,
w = 0 = 14 ka 2 + c2 ∴ c2 = − 14 ka 2
It follows that
w = − D (1−0ν ) ( xy − a2
M
4 )
We observe that this solution satisfies boundary conditions, M x = 0 and M y = 0
at plate edges.

22
SOLUTION (3.17)

Let
A= Pb2
2π 3 D ∑ 1
n3
sin n2π
Thus, w = A sin αy(1 + αx )e −αx
and
∂w
∂x = −αA sin αy(1 + αx )e −αx + αA sin αye −αx = − xα 2 A sin αye −αx
∂ 2w
∂x 2
= −α 2 A sin αy (1 − αx )e −αx , ∂ 2w
∂y 2
= −α 2 A sin αy (1 + αx )e −αx
We observe that, boundary conditions
∂ 2w
w=0 ( x → ∞) , w= ∂y 2
=0 ( y = 0, y = b)
are satisfied.

M = − D( ∂∂xw2 + ν ∂ 2w
)= ∑α sin n2π sin αye −αx
2 2
Pb2
(a) ∂y 2 2π 3 n3

= P
2π ∑( 1
n sin n2π sin αy )e −αx
Also,
M x = − D( ∂∂xw2 + ν ∂∂yw2 ) = ∑ sin n2π sin αy[1 + ν + (1 − ν )αx ]e −αx
2 2
P 1
2π n

It follows that
M x = (1 + ν ) M − (1 − ν ) x ∂∂Mx

( b ) For x = 0, ν = 1 3 , n = 1, and y = b
2 :
M x ,max = 2Pπ (1 + ν ) = 23πP
Hence,
σ x ,max =
6 M x ,max
t2
= 4P
πt 2

SOLUTION (3.18)

( a ) From Eqs.(3.12) with M xy = 0 and M x = M y = M 0 , for any corner angle θ of the plate:
M x' = M 0 , M x' y' = 0 , M y' = M 0
Here θ is a corner angle of the plate.

( b ) From Eq.(3.40) with M x = M y = M 0 :


M0 ( x 2 + y 2 )
w=− 2 D (1+ν ) (a)
This represents a paraboloid of revolution. On the other hand, since M x = M y , from Eqs.(3.9)
we conclude that
κ x = κ y = − D (1M+ν ) 0
(b)

Expression (b) implies that the surface is a sphere. The results are inconsistent because
approximate expressions
∂ 2w ∂ 2w
∂x 2
and ∂y 2

are used for κx and κy, respectively, in deriving Eq.(a).

23
SOLUTION (3.19)

Strain energy is expressed by (Sec.3.12) :

∫∫
b a
U= [( ∂∂xw2 ) 2 + ( ∂∂yw2 ) 2 + 2ν ∂ 2w ∂ 2w
+ 2(1 − ν )( ∂∂x∂wy ) 2 ]dxdy
2 2 2
D
2 ∂x 2 ∂y 2
(a)
0 0
Referring to solution of Prob.3.12, it can be shown that,

∫ ∫ ( ∂∂
b a
∫∫
b a
) dxdy = 0.325c0 ( ∂∂yw2 ) 2 dxdy = 0.325c0
2 2
w 2 b a
x2 a3
, b3
0 0 0 0

2∫ ∫ ∂∂ dxdy = 2∫ ∫
b a b a c2
w ∂ 2w
( ∂∂x∂wy ) 2 dxdy = 0.0929 ab0
2 2

0 0 x 2 ∂y 2 0 0
Formula (a) is thus,

∫∫
b a
U= [( ∂∂xw2 )2 + ( ∂∂yw2 ) 2 + 2 ∂∂xw2 ∂ 2w
2 2 2
D
2 ∂y 2
]dxdy (b)
0 0
Upon substituting the values obtained into Eq.(b), we have
Dc02
U= a +b
+ 0.0929).
4 4

ab ( 01625
. a 2 b2

SOLUTION (3.20)

Refer to Solution of Prob. 3.19. We now have


a a a a
∫∫ ( ∂∂xw2 ) 2 dxdy = 0.325 ac02 , ∫∫ ( ∂∂yw2 ) 2 dxdy = 0.325 ac02
2 2

0 0 0 0
a a a a
2∫ ∫ dxdy = 2∫ ∫
c2
∂2w ∂2w
( ∂∂x∂wy ) 2 dxdy =0.0929 a02
2

0 ∂x ∂y
2 2
0 0 0

Formula (a) is therefore,


a a
U= ∫∫ [( ∂∂xw2 ) 2 + ( ∂∂yw2 ) 2 + 2 ∂∂xw2 ∂2w
2 2 2
D
2 ∂y 2
]dxdy (b)
0 0
Introducing the values obtained into Eq. (b), we obtain
Dc02
U= (0.325 + 0.0929) = 0.4179 D( a0 ) 2
c
a2

24
CHAPTER 5

SOLUTION (5.1)

We take a=1.5 m and b=0.9 m (Fig.5.1).


( a ) Equations (3.11) and (5.7) yield,
∞ ∞
σx = 96 p0
π 4t 2 ∑∑m n
( m a )2 +ν ( n b )2
mn [( m a )2 + ( n b )2 ]2
sin maπx sin
nπy
b

Maximum stress occurs at the center ( a 2 , b 2 ):


∞ ∞

∑ ∑ ( −1)
m+ n
−1 ( m a )2 +ν ( n b )2
σ x ,max = 96 p0
π 4t 2
2
mn [( m a )2 +( n b )2 ]2
m n
Retaining first four terms (m=1, n=1,3; m=3, n=1,3):
σ x ,max = [0.2890 − 0.0094 − 0.0532 + 0.0036] = 1007.435 p0
96 p0
π 4 ( 0.015 )2
Similarly,
∞ ∞

∑∑
m+ n
ν ( m a )2 + ( n b )2 −1
σ y ,max = 96 p0
π 4t 2 mn [( m a)
2
+( n b)
2 2
]
( −1) 2

m n

= [0.4852 − 0.0281 − 0.0296 + 0.0060]


96 p0
π 4 ( 0.015 )2

= 1886.97 p0 ≤ σ yp
Hence,
1886.97 p0 = 240 × 10 6 or p0 = 127.2 kPa .
For all values of p0 greater than the above, yielding occurs.

( b ) Employing Eq.(b) of Sec.5.3 and D = 200 × 10 ( 0.015) / 10.92 = 61813


9 3
. , we obtain
∞ ∞ m + n −1

wmax =
16 p0
π 6D ∑∑
m n
( −1 ) 2
mn [( m a )2 +( n b )2 ]2

= [0.3547 − 0.0025 − 0.0122 + 0.0005] = 0.01166 m.


16×127.2
961.403× 61.813
= 1166
. mm.

SOLUTION (5.2)

200×109 ( 2×10 −3 )3
D= Et 3
12 (1−ν 2 )
= 12 (1− 0.09 ) = 146.5 kN ⋅ m
wmax = = 30×10
= 8.4 mm
p0 3
(a) π 4D( 2
)2 π 4 (146,500 )( 14 )
a2
(1+ν ) p0 (1+ν )
M x ,max = M y ,max = =
p0
(b) π2( 2 )2 a2 π2
a2

6 p0 (1+ν ) 6 ( 30×10 )1.3


σ max =
3
= = = 59.3 MPa
6 M max
t2 π 2t 2 π 2 ( 0.02 )2

( c ) At x = y = a
2 :
p0 ( 3−ν )
= − 30×104π( 2.7 )2 = −12.9 kN / m
3
Rx ,max = R y ,max = − 4π
2 p0 (1−ν ) 2 ( 30×103 )( 0.7 )
(d) Rc = π 2a 2 ( 2 )2
2
= π2
= 4.26 kN
a

(CONT.)

38
(5.2 CONT.)
Total distributed reaction:
4 p0a 2 8 p (1−ν )
Rd = − π2
− π 2a 20 ( 2 )2
= −65.6 kN
a2

Alternately;
Rd = 4∫ − 12.9(10 3 ) sin( 2 )dy = −65.6 kN
2 πy
0
Weight (Load):
4 p0 a 2
P= π2
= 48.6 kN

SOLUTION (5.3)

From Example 5.1, for m=n=1:

12 (1− 0.09 )
(a) wmax = 0.00416 p0 a4
D = 0.00416(30 × 103 )( 2) 4 200 (109 )( 0.02 )3

= 13.63 mm

( b ) From Example 5.1, for m=n=1: M max = 0.0534 p0 a 2


σ max = 6 M max
t2
= 6( 0.0534)(30 × 103 )( 0.202 ) 2 = 9612
. MPa

4 p a2 πy
Rc = 2 D(1 − ν ) ∂∂x∂wy = 2 D(1 − ν )[ π 40D cos cos πax ]
2
(c) a ( x=a, y=a )
= 6899 N

SOLUTION (5.4)

( a ) Equation (5.3): pmn = p0 . Equation(5.5) for a=b:


∞ ∞
w=
p0a 4
π 4D ∑∑
m n
sin( mπx a )sin( nπy b )
( m2 + n 2 )2
(a)

At x=y= b 2 :
∞ ∞ m + n −1

wmax =
p0a 4
π 4D ∑∑
m n
( −1 ) 2
( m2 + n 2 )2

We have
∞ ∞
M xy ,max = −
(1−ν ) p0 a 2
π 2 ∑∑ m n
mn
( m2 + n 2 )2

Hence
∞ ∞
Rc = −2 M xy ,max =
2 (1−ν ) p0 a 2
π2 ∑∑m n
mn
( m2 + n 2 )2

−3 3
)( 25×10 9
D = 12(1Et−ν 2 ) = 165(10 = 229167 kN ⋅ m
3 )
(b) 12 (1− 0.252 )
.
Equation (a) is thus
p0 ( 4 )4
10 × 10 −3 = π 4 ( 229,167 ) 4
[ 1 − 100
1
] or p0 = 3.633 kPa

39
SOLUTION (5.5)

Equation (5.12), taking m=n=1, becomes


πy
w= 4P 1
π 4 Dab [( 1 a )2 +( 1b )2 ]2
sin πax sin b

Derivatives of w are
∂w π πy πy
∂x = 4P 1
π 4 Dab [( 1 a )2 +( 1 b )2 ]2 a
cos πax sin b ≡ A πa cos πax sin b

∂ 2w π π πx πy ∂ 3w π2 π πx πy
∂x∂y = A ⋅ a b cos a cos b , ∂x 2∂y
= − A ⋅ a2 b sin a cos b

∂ w πy πy
= − A ⋅ πa π
cos πax sin ∂ w
= − A ⋅ πa 2 sin πax sin
3 2 2 2

∂x∂y 2 b2 b , ∂x 2 b

∂ 3w π3 πx πy ∂ w πy
= − A ⋅ a3 cos = − A ⋅ πa3 sin πax cos
3 3

∂x 3 a cos b , ∂y 3 b

Equations (3.23) lead to then,


πy πy
Rx = D[− A( πa )3 cos πax sin b − ( 2 − ν ) A( πa )( πb ) 2 cos πax sin b ]
πy πy
R y = D[− A( πb )3 sin πax cos b − ( 2 − ν ) A( πa )2 ( πb )sin πax cos b ]

From the above expressions, we obtain:


πy
Rx = A ⋅ D πa sin b [( πa ) 2 + ( 2 − ν )( πb ) 2 ] ( at x=0 or x=a )
πy
R y = A ⋅ D b sin π
a [( b ) + ( 2 − ν )( a ) ]
π 2 π 2
( at y=0 of y=b )

SOLUTION (5.6)

∫ ∫ x sin π sin dxdy


b a nπy
pmn =
4 P0 m x
a 2b 0 0
a b

∫ [ ( π ) sin π x − π x cos
b nπy
= mπ
4 p0 a 2 x a a
a 2b m a m a x 0 sin b ]dy
0

π cos mπ ∫ sin
π b
= − bm 0 dy = π cos mπ (cos nπ − 1)
4p n y 4 p0
0
b mn 2

Letting m=n=1, Eq.(5.5):


p0 4 p0 cos π (cos π −1) πy
w= π 4 D π 2 [( 1 a )2 +( 1 b )2 ]2
sin πax sin b
πy
( −1)( −1−1) πy sin πax sin b
= sin πax sin =
4 p0 8 p0
π 6 D [( 1 a )2 +( 1b )2 ]2 b π D [( a )2 +( 1 b )2 ]2
6 1 Q.E.D.

SOLUTION (5.7)

We have
πy πy
∂ 2w sin πax sin ∂ 2w cos πax cos b
= − π 4a 20D =
8p b 8 p0
∂x 2 [( 1 2
+( 1 2 2 , ∂x∂y π abD [( 1 a )2 +( 1b )2 ]2
4
a) b) ]
πy
∂ w sin πax sin
= − π 4b20D
2 8p b
∂y 2 [( 1 a )2 +( 1 b )2 ]2

( a ) For a=b, ν = 0.3:


2.6 p0a 2 πy
M x = M y = − D( ∂∂xw2 + ν ∂ 2w
)= sin πax sin
2

∂y 2 π4 a
2
At x = , y = a2 , Mx = M y =
a 2.6 p0 a
2 π4
(CONT.)

40
(5.7 CONT.)
Then, at z = t
2 :
σ x ,max = . p0 ( at ) 2 = σ y ,max
= 016
6 Mx
t2

1.4 p0a 2 πy
M xy = − D(1 − ν ) ∂∂x∂wy = − cos πax cos
2
(b) π4 b
2
At x=0, y=0: M xy = −
1.4 p0a
π4

and at z = t
2 , τ xy ,max = −0.086 p0 ( at ) 2
πy
Also Qx =
4 p0 a
π3
cos πax sin b , τ xy = 3 Qx
2 t [1 − ( 2tz ) 2 ]
Thus,
τ xz ,max = 01935
. p0 ( at ) 2 ( x = 0, z = 0, y = a2 )
Similarly
τ yz ,max = 01935
. p0 ( at ) 2 ( y = 0, z = 0, x = a2 )
Equation (3.30), for p = ( x t ) p0 :
σ z = − 43 x[ 23 − + 13 ( 2tz )3 ]
p0 2z
a t

At z = − t 2 and x=a:
σ z ,max = − p0

SOLUTION (5.8)

The pmn , given by Eq.(i) of Example 5.3, becomes


m+ n
2 −1
pmn = ( −1) sin m4π sin n4π ( m, n = 1,3,⋅ ⋅ ⋅)
16 p0
π 2 mn
Deflection, Eq.(5.5), appears then
∞ ∞ m + n −1

∑∑ sin( mπ
( −1 ) nπ nπy
w= sin maπx sin
16 p0 2
4 )sin( 4 )
π D 6
mn [( m a )2 +( n b )2 ]2 b
m n
The maximum displacement occurs at center ( a/2, b/2 ) and is:
∞ ∞
wmax =
16 p0
π 6D ∑∑ m n
sin( mπ 4 )sin( nπ 4 )
mn [( m a )2 +( n b )2 ]2
( −1) m+ n− 2
Retaining the first two terms,
0.304 p0a 4
wmax = [ 2( 1 21+ 1 2 ) − ]=
16 p0 1
π 6D 6( 1 + 9 ) π 6D
a b a2 b2

Bending moment is,


∞ ∞ m + n −1

∑∑ sin( mπ
( − 1) nπ nπy
Mx = [( ma ) 2 + ν ( nb ) 2 ]sin maπx sin
16 p0 2
4 )sin( 4 )
π 4D mn [( m a )2 +( n b )2 ]2 b
m n
∞ ∞
and M x ,max =
16 p0
π 4D ∑∑
m n
sin( mπ 4 )sin( nπ 4 )
mn [( m a )2 +( n b )2 ]2
[( ma )2 + ν ( nb )]( −1) m+ n−2

σ x ,max = ν = 0.3 :
6 M x ,max
Then, t2
, taking the first two terms and
1 +ν 1 +9ν
σ x ,max = 48 p0
[ ( 1a 2 + 1b2 ) − 3( 1a 2 + 9b 2 ) ]
2 2 2 2

π 4t 2 a b a b

= [0.8a 2 − 0.01a 2 ] = 0.389 p0 ( at ) 2


48 p0
π 4t 2

41
SOLUTION (5.9)

Apply Eq.(5.12):
∞ ∞
∂w
∂x = 4 Pa 2
π 4D ∑∑
m n
sin( mπx1 a )sin( nπy1 a )
( m2 + n 2 )2
( maπ ) cos( maπx ) sin(
nπy
a )
∞ ∞
∂ 2w
∂x∂y = 4 Pa 2
π 4D ∑∑ m n
sin( mπx1 a )sin( nπy1 a )
( m2 + n 2 )2
( maπ )( naπ ) cos( maπx ) cos(
nπy
a )
Equation (3.24): for x=a, y=a:
Rc = −2 D(1 − ν ) ∂∂x∂wy
2

∞ ∞
Rc =
8 P (1−ν )
π 2 ∑∑ m n
mn
( m2 + n 2 )2
sin(
mπx1
a ) sin(
nπy1
a ) cos( maπx ) cos(
nπy
a ) ( m, n = 1,3,⋅ ⋅ ⋅)

Here x1 = y1 = a / 2 . Taking first two terms (m, n=1,3):


8 P (1−ν )
Rc = π2
[ 14 (1)(1) + 100
3
( −1)] = 0119
. P

SOLUTION (5.10)

Maximum deflection using Eq.(5.12) with m=n=1:


sin( πx a )sin( πy b )
wmax = 4P
π 4 Dab [( 1 a )2 + ( 1 b )2 ]2
= 4P 1
π 4 Dab [( 1 a )2 + ( 1 b )2 ]2
For a=2b, the above becomes
wmax = = 0.00328 PaD
2
8P 1
π 4 Da 2 [( 1 a )2 +( 2 a )2 ]2

We have
∞ ∞ ∞ ∞
M x ,max = 4P
π 2ab ∑∑ m n
( m 2 b )2 +ν ( n b )2
[( m 2 b )2 +( n b )2 ]2
= 8P
π2 ∑∑ m n
m2 + 4νn 2
( m2 + 4 n 2 )2

∞ ∞

∑∑ ν m +4n2 2
M y ,max = 8P
π2 ( m2 + 4 n 2 )2
m n
Stresses, setting m=n=1 and ν = 0.3, are thus
σ x ,max = = = 0.428 tP2
6 M x ,max 48 P 2.2
t2 π 2t 2 25

σ y ,max =
6 M y ,max
t2
= 48 P 4.3
π 2t 2 25
= 0.836 tP2

SOLUTION (5.11)

It is observed that
sin( mπc / a ) sin( nπd / b )
lim c = 00 , lim d = 0
0
c→0 d →0
Since the above expressions are undefined, we apply
L’Hospital’s rule:
sin( mπc / a )
lim c = lim maπ cos maπc = mπ
a (a)
c→0 c→ 0
and
sin( nπd / b )
lim d = lim nbπ cos nπbd = nπ
b (b)
d →0 d →0
Employing Eqs.(a) and (b) obtained above, Eq.(i) of Sec.5.3 appears in the form given by
Eq.(5.10).

42
SOLUTION (5.12)

Equation(5.12), for a = b, x1 = a
2 , y1 = a 4 , x 2 = a 2 , and y 2 = 3a 4 :
∞ ∞
w1 =
4 P1
π Da 2
4 ∑∑ m n
sin( mπ / 2 )sin( nπ / 4 )
( m2 + n 2 )2 / a 4
sin m2π sin n2π
∞ ∞
w2 =
4 P2
π 4 Da 2 ∑∑ m n
sin( mπ / 2 )sin( 3 nπ / 4 )
( m2 + n 2 )2 / a 4
sin m2π sin n2π
Retaining the first two terms (m=1; n=1,3) of the above,
2 2
w1 = + (1)(1/ (1)( −1)] =
4 Pa Pa
1
π 4D
[ (1)(1/4 2)
100
2) 1
πD
0.96
2
4 P2a 2 P2a 2 0.96
w2 = π 4D
[ (1)(1/4 2)
+ (1)(1/
100
2)
(1)( −1)] = πD 2
It follows that
0.96 ( P1 + P2 ) a ( P1 + P2 ) a 2
2
w = w1 + w2 = π 2
4 D = 0.00697 D

SOLUTION (5.13)

We have b=2a, Fig.5.1. Hence α m = mπb / 2a = mπ . Maximum deflection is expressed by


Eq.(5.25):
∞ m −1

wmax =
5 p0a 4
384 D −
4 p0a 4
π 5D ∑ ( −1 ) 2

m=1, 3,⋅⋅⋅
m5
mπ tanh mπ + 2
2 cosh mπ

5 p0a 4 4 p0a 4
= 384 D − π 5D
[ π 2tanh π +2
cosh π −
1 3π tanh 3π + 2
35 2 cosh 3π
+ ⋅ ⋅ ⋅]
4 4
= − ( 0.221268 − 0.000003794 + ⋅⋅⋅)
5 p0a 4 p0a
384 D π 5D
4
p0 a 4
= − 4× 0.22127
) = 0.0101286
p0 a 5
D ( 384 π5 D

p a4
or wmax = 0.01013 0D
This solution agrees with result given in Table 5.1.

SOLUTION (5.14)

We let b=2a and α = mπb / 2a = mπ in Eq.(5.24).The derivatives of are, from Eq.(5.24),


∑ mπy mπy mπy


4
∂ 2w mπ tanh mπ + 2
= [1 − +
4 p0a 1 1
then : ∂x 2 π 5D m5 2 cosh mπ cosh a 2 cosh mπ a sinh a ]
m

× ( maπ ) 2 ( −1) sin maπx



∂ 2w
∂y 2
=
4 p0a 4
π 5D ∑ m
1
m5
{ mπ2 tanh mπ + 2
cosh mπ cosh
mπy
a ( maπ )2
mπy mπy mπy mπy
+ 1
2 cosh mπ [( maπ ) 2 cosh a + ( maπ ) 2 cosh a + a sinh a ( maπ ) 2 ]}
× sin maπx
It follows that

∇ w=
2 4 p0 a 2
π 3D ∑ m
1
m3
{−1 + 1
cosh mπ cosh
mπy
a [1 + mπ tanh mπ + 2]}sin maπx

(CONT.)

43
(5.14 CONT.)
Equation (3.16) is thus
Qx = − D ∂∂x ( ∇ 2 w )

=
4 p0a
π 3 ∑m
1
m2
{1 − 1
cosh mπ cosh
mπy
a (3 + mπ tanh mπ )]cosh maπx

SOLUTION (5.15)

We substitute f m = K m e λm y into
d 4 fm d 2 fm
dy 4
− 2( maπ ) 2 dy 2
+ ( maπ ) 4 f m = 0 to obtain ( maπ ) 4 − 2( maπ ) 2 λm2 + λm4 = 0
The latter equation may be rewritten
[λm2 − ( maπ ) 2 ][λm2 − ( maπ ) 2 ] = 0 having roots : λm = mπ
a , mπ
a ,− mπ
a ,− mπ
a .
The general solution for f m ( y ) is thus given by Eq.(5.16).
Employing the identities
mπy mπy mπy mπy mπ y
mπy
sinh a = 12 ( e a
− e− a
), cosh a = 12 ( e a
+ e− a
)
We can readily represent Eq.(5.16) in the form given by Eq.(5.17).

SOLUTION (5.16)

For a=b, x=y=a/2, and m=1, Eqs.(h) of Example 5.5 give :


p0a 4 p0 a 3
A1 = D ( 0.0104624), D1 = − D ( 0.0328686)
3
p0 a 4
C1 = B1 = −
p0a
D (9.02502) 267.44−1 6.2832 , D ( 0.0130711)
Expression (5.27) is thus,
p0a 4
w= D ( 0.0028323)
For a=b, x=y=a/2, and m=3, in a like manner :
p0a 4 p0a 3
A3 = D (53740421
. × 10 −5 ), D3 = − D (5.0649 × 10 −4 )
p0a 4 p0a 3
B3 = − D (5.37903 × 10 −5 ), C3 = D (3889005
. × 10 4 ) 767764671.7− 6.2832
Expression (5.27) is thus,
p0 a 4
w=− D (397734
. × 10 −5 )
The center deflection (x=y=a/2), for a=b and m=1, 3, is therefore :
p0a 4 p0 a 4
w= D ( 0.0028323 − 0.0000398) = 0.0027925 D

SOLUTION (5.17)

Refer to Example 5.6. It is now necessary to replace Eq. (k) by


w=0 ∂2w
∂y 2
=0 ( y = 0) (k’)
Equations (j) and (l) remain the same.
The particular and homogeneous solutions are given by Eqs. (5.22) and (5.18),
respectively, both of which satisfy the conditions (j). Applying Eqs. (k’) and (l) to
wh + wp gives definite values of the constants Am , Bm , Cm , Dm . The deflection
is then found by adding Eqs. (5.18) and (5.22).

44
SOLUTION (5.18)

Homogeneous and particular solutions are given by Eqs.(5.28) and (5.22), respectively.
Boundary conditions w = 0 and ∂w / ∂y = 0 at y=0, lead to

w = ∑ [( Bm' + Dm' y )e − mπ / a +
4 p0a 4 1
π 5 D m5
]sin maπx = 0
m

∂w
∂y = ∑ [− maπ Bm' + Dm' − mπ
a Dm' y ]e − mπ / a sin maπx = 0
m
Solving,
4 p0a 4 1 mπ
Bm' = − π 5 D m5
, Dm' = a Bm'
The deflection is thus,

[ ax 4 − 2 ax 2 + ax − π965 ∑ m15 (1 +
mπy
)e − mπy / a sin maπx ]
p0a 4
w=
4 2

24 D a
m

SOLUTION (5.19)

Case B. Apply Eq.(5.21), setting p( x , y ) = p0 :


x1 + e x1 + e
pm = ∫ p0 sin maπx dx = − cos maπx
2 2 p0
a mπ x1 − e
x1 − e
mπx1
=− [cos maπ ( x1 + e ) − cos maπ ( x1 − e )] = − ( −2 sin sin maπe )
2 p0 2 p0
mπ mπ a
or
mπx1
pm = sin maπe
4 p0
mπ sin a Q.E.D.
Case C. Using Eq.(5.21) with line load at x = x1
x1 + 0

mπx1
pm = p0 sin maπx dx =
2 2 p0
a a sin a Q.E.D.
x1 − 0

SOLUTION (5.20)

We have, wh and w p are given by Eqs.(5.18) and (5.22). Deflection is


w = wh + w p (a)
Expression (a) satisfies Eq.(j) of Example 5.6. Substitution of w into Eqs.(k) of Example 5.6
leads to
4 p a4
Bm = − π 5 Dm
0
5 , Am = − maπ Dm

Introducing w into Eqs.(l) of Example 5.6 :


4 p0 a 4 ( 3+ν )(1−ν )cosh 2 βm + 2ν cosh β −ν (1−ν ) βm sinh β m − (1−ν 2 )
Cm = π 5 Dm5 mπ
a
( 3+ν )(1−ν )cosh 2 βm +(1−ν )2 βm2 +(1+ν )2
and
4 p0a 4 ( 3+ν )(1−ν )sinh βm cosh βm +ν (1+ν )sinh βm −ν (1−ν ) β m cosh βm −(1−ν )2 βm
Am = π 5 Dm5 ( 3+ν )(1−ν )cosh 2 βm +(1−ν )2 βm2 +(1+ν )2

Here βm = mπb / a. Substitution of these constants into Eq.(a) yields and expression for
deflection.

45
SOLUTION (5.21)
When a=b/3 and m=1, we have α m = 3mπb / 2b = 3π / 2. Deflection (5.35) at center of plate
(x=a/2, y=0) is then :


m −1
2 p0a 4 ( −1)m +1
w= (1 − 0.06029 )( −1) 2 , pm = ( −1) m+1
2 p0
π 5D m5 mπ
m
or
p0a 4
w = 0.00614 D
Bending moment about x axis is using Eq.(5.34) :
M x = − D( ∂∂xw2 + ν ∂ 2w
2

∂y 2
)

= a2
π 2 ∑ m
pm
m2
[1 − Bm cosh
mπy
a (1 − ν ) + Cm sinh
mπy
a (1 − ν )]sin maπx
At x=a/2 and y=0,

Mx =
2 p0 a 2
π3 ∑m
1
m3
[1 − Bm (1 − ν )]sin m2π
For m=1,
2 p0 a 2
Mx = π3
(1 − 0.06029 × 0.7 ) = 0.06178 p0 a 2

SOLUTION (5.22)
We have a = b, α = mπb / 2a = 0.5π , and pm = ( 2 p0 / a ) sin(π / 2) =
2 p0
a
On applying Eq.(5.35), then we obtain :
1
w= a4
π 4D ∑ m
2 p0
a [1 − 2+ 0.5π ( 0.9172 )
2 ( 2.509 ) cosh
πy
a +
( πy / a )sinh( πy / a )
2 ( 2.509 ) ]sin πax
or
1
w=
2 p0a 3
π 4D ∑ (1 − 0.287 × 1) = 0146
m
.
p0a 3
D

SOLUTION (5.23)

Load function, from Table 5.2, is



pm =
4 p0
mπ sin 2 2
Expression (5.35) becomes then

w= a4
mπD ∑ m
4 p0
mπ sin 2 mπ
2 (1 −
2+α m tanh α m
2 cosh α m cosh
mπy
a +
( mπy / a )sinh( mπy / a )
2 cosh α m )sin m2π
For m=1, x=a/2, and y=0:
p0a 4 p0a 4
w= ( 0.01307)(1)(1 − 22.50918
D × 2 × 1 + 2.50918× 2 )(1) = 0.00308
+1.8343 0
D

“Exact” solution is given by w = 0.00406 p0 a / D.


4

SOLUTION (5.24)

Expression (d) of Example 5.11 gives


σ max t 2
6 = 0.0513 p0 a 2
(CONT.)

46
(5.24 CONT.)
from which
(100×106 )( 0.01)2
a2 = 6 ( 0.0531)( 7×10054 ) = 0.46163; a = 0.679 m = 679 mm
We have
p0a 4 200(109 )( 0.01)3
wmax = 0.00126 D , where D = 12( 0.91) = 18,315 N ⋅ m
Thus,
0.00126 ( 7×10054 )( 0.679 )4
wmax = 18315 = 0.001029 m = 103
. mm.

SOLUTION (5.25)

Equation (5.31): p( x ) = ∑p
m=1
m
mπx
a

We have
pm = ∫ p( x ) sin mπx
dx = (1 − cos m2π )
2 2 p0
a a mπ

Thus
pm = mπ0 for m = 1,3,⋅⋅⋅, pm = for m = 2,6,⋅⋅⋅,
2p 4 p0

pm = 0 for m = 4,8,⋅⋅⋅,
It follows that
∞ ∞
p( x ) =
2 p0
π ∑
m=1,3,⋅⋅⋅
1
m sin mπx
a +
4 p0
π ∑ m
1

m= 2 , 6 ,⋅⋅⋅⋅
sin maπx (a)

Proceeding as in Sec. 5.4, we obtain


∞ ∞ ∞
w p = ∑ k m sin maπx = ∑ ∑
2 p0a 4 4 p0a 4
π 5D
1
m5
sin maπx + π 5D m5
1
sin maπx (b)
m=1 m=1, 3,⋅⋅⋅ m= 2 , 6 ,⋅⋅⋅⋅
Boundary conditions:
w = 0, ∂ 2w
∂y 2
+ν ∂ 2w
∂x 2
= ∂ 2w
∂y 2
=0 ( y = ± b2 ) (c)
wherein w = wh + w p . Now, Eq.(5.23) becomes

w = ∑ ( Bm cosh
mπy mπy
a +Cm y sinh a + k m )sin maπx (d)
m
For a specific value of m, introducing Eq.(d) into (c), we obtain two expressions from
which
Bm and Cm are found as:
mπb
Bm = − 2 cosh(m mπb ) [2 + tanh maπb ]
k
2a
2a

Cm = a km
mπ 2 cosh( mπb )
2a

The homogeneous solution becomes



wh = − π04 D
p a4

m
dm
m5
cosh λ
[ 2 cosh αmm ( 2 + α m tan α m ) − 2 2 coshm α m sinh λm ]sin maπx
λ

mπy
Here λm = a αm = mπx
2a and
d m = 2 for m = 1,3,⋅ ⋅ ⋅, d m = 4 for m = 2,6,⋅⋅⋅, d m = 0 for m = 4,8,⋅ ⋅ ⋅,
For a=b, the center deflection is
p0a 4
w = 0.00203 D ( x = a2 , y = 0 )

47
SOLUTION (5.26)
Deflection, substituting Eq. (a) of Sec.5.5 into Eq. (5.35), is
∞ mπy mπy

wh =
p0 a 4
∑ ( −1)m +1
(2 −
2+α m tanh α m mπy
+ ) sin maπx
( )sinh
π D
5
m5 cosh α m cosh a
a
cosh α m
a

m
The slope, along the side at y=b/2, is thus

∂wh
∂y =
p0a 3
π 4D ∑m
( −1)m+1
m4
[α m − tanh α m (1 + α m tanh α m )]sin maπx

This slope is prevented by moments, f ( x ) = M y at y = ± b / 2, given by Eq.(a) of Sec.5.6.


To ascertain the values of M m in the series f(x) we set the derivative of w (given by Eq.5.40),
∂w / ∂y , equal to − ∂wh / ∂y at y=b/2. In so doing, we have
2 p0a 2 ( −1)m+1 α m − tanh α m (1+α m tanh α m )
Mm = π 3m3 α m − tanh α m ( α m tanh α m −1)

Introducing the above in Eq.(a) of Sec.5.6, we thus have f ( x ) = ( M y ) y =± b/ 2 .

SOLUTION (5.27)
We have, center deflections, for strips a
a and b : a
5 p a4 p b4 2
wa = 384a EI , wb = 384b EI
Using wa = wb and pa = pb ,
b
p0 b4 5 p0a 4 2
pa = 5 a 4 + b4
, pb = 5 a 4 + b4 x b
Then,
pa a 4 p0a 4b4
wmax = k1 D = 5
384 ( 5 a 4 + b4 ) D

p a4
When a=b : wmax = 0.00217 0D y
4
This result compared to 0.00192 p0 a / D, Example 5.11, differs
( 0.00217 − 0.00192 ) / 0.00217 = 13 %
Next, setting a=b :
4
M x = k 2 pa a 2 = 1 p0b
8 5 a 4 + b4 a 2 = 0.0208 p0 a 2 .
4
M y = k 3 pb b 2 = 1 5 p0a
12 5 a 4 + b4 b 2 = 0.0694 p0 b 2 .
Compared to value M y = 0.0697 p0 a , above differs only 0.3%.
2

SOLUTION (5.28)
p0
We assume that : strip a is
under hydrostatic loading with
maximum intensity pa ; strip b b
2
is under uniform loading pb . b x
Then, at the center [Ref. 5.9]:
pa a 4 4
wa = wb or 0.00651 EI = 5 pbb
384 EI
a
from which, 2
y
pa = 2 p
4
b
b a4 (1)
a

48
(CONT.)
(5.28 CONT.)

Load at center under assumed loading must add up


to effective load at center:
1
3 pa + pb = 13 p0 (2)
Thus, from Eqs.(1) and (2), taking a=b :
p0 a 4
pb = =
p0
2 b 4 + 3a 4 5

Hence, center deflection of plate is given by


pbb4 p0 a 4 b4
wc = k1 D = 5
384 ( 2 b4 +3a 4 ) D

For a=b, the above becomes


4
p0a 4
wc = 5 p0a
384 5 D = 0.002604 D

This result, compared to 0.00203 p0 a / D, differs ≈ 28 %.


4

Similarly,
2
p0 b2
My = 1 p0 b
8 5 = 40 = 0.025 p0 b 2
“Exact” solution is M y = 0.0239 p0 b . Error is thus, 4.6 %.
2

SOLUTION (5.29)

Table 5.3 k1 = 5
384 , k2 = 1
8 , k3 = 0
p0a 4 p0a 4
b
a = 1: wmax = k1 2D = 0.00651 D
p0a 4 ( 3a )4 p0 a 4
b
a = 3: wmax = k1 D [ a + ( 3a ) ]
4 4 = 0.0129 D

p0a 4 ( 5 a )4 p0a 4
b
a = 5: wmax = k1 D [ a +( 5a ) ]
4 4 = 0.0130 D

p0a 6
b
a = 1: M x = k2 2a4
= 0.0625 p0 a 2 = M y
p0 a 2 ( 3a )4
b
a = 3: M x = k2 a 4 + ( 3a ) 4
= 01235
. p0 a 2
p0a 4 ( 3a )2
M y = k2 a 4 + ( 3a ) 4
= 0.0137 p0 a 2
p0a 2 ( 5 a )4
b
a = 5: M x = k2 a 4 + ( 5 a )4
= 01248
. p0 a 2
p0a 4 ( 5 a )2
M y = k2 a 4 + ( 5 a )4
= 0.00499 p0 a 2

Compared with Table 5.1:


b
a 1 3 5
Δδ1 +60.1% +5.48% +0.23%
Δδ2 +30.5% +3.87% +0.16%
Δδ3 +30.5% -66.3% -86.7%

Note: + larger values, - smaller values.

49
SOLUTION (5.30)

At center wa = wb p 0 = p a + pb
4 4
wa = 384 wb =
17 a p a 1 pbb
EI , 384 EI

Thus 17 pa a = pb b
4 4

and b
pa = p b4
0 17 a 4 + b4 , pb = p 17 a 4
0 17 a 4 + b4

At y = ± b
2 with k3 = 1
12 :
4 2 4 2 a x
M max = k3 =
p0 (17 a b ) 1 p0 (17 a b )
17 a + b4 4 12 17 a 4 + b4
We have
4 2
σ max = =
6 M max 1 17 a b p0
t2 2 (17 a 4 + b4 ) t 2

SOLUTION (5.31)

Refer to Solution of Prob. 5.30 with a = b.


17 Pa = pb
and
pa = p0 18aa4 = 181 p0 pb = 17
4

18 p0

At y = ± a 2 with k3 = 1 12 :
2 2
M max = k3 1718pa = 17216
p0 a

Thus
σ max = 6 Mt max
2 = 17
36 p0
a2
t2

SOLUTION (5.32)

Upon substituting the given numerical values into Eq.(g) in Sec.5.9, we obtain
A1 = D1 {a × 0.0869135[0.159155 M1a + 0.020532 p0 a 3 (10.591953)]
+ 0.0130711 p0 a 4 (1003742
. − 1570796
. − 0.0865895)}
or
p0a 4 M1a 2
A1 = 0.010358 D + 0.013833 D
Similarly,
p0a 4 p0a 3
B1 = −0.013071 D , C1 = 0.020532 D
p0a 3 M1a 2
D1 = −0.01378 − 0.018831 E1 = −0.319503
M1a
D D , D

F1 = 0, G1 = 0159155 H1 = −0159750
M1a M1a
. D , . D

Now from Eq.(f) in Sec.5.9, substituting the above obtained value of constants, we have
M1 = 0.01359 p0 a 2

50
(CONT.)
(5.32 CONT.)
The constants are then rewritten as follows :
A1 = 0.010546 p0 a 4 / D D1 = −0.019018 p0 a 3 / D
B1 = −0.013071 p0 a 4 / D E1 = 0.004342 p0 a 4 / D
C1 = 0.020532 p0 a 3 / D G1 = 0.002163 p0 a 3 / D
H1 = −0.00271 p0 a 3 / D
Expression (5.52a), at x1 = a / 2 and y = a / 2, is thus
w1,max = 0.005495 p0 a 4 / D
Similarly, from Eq.(5.52b) at x 2 = a / 2 and y = a / 2:
w2,max = −0.010227 p0 a 4 / D

SOLUTION (5.33)

For a=b, Table 5.4 gives


δ1 = 0.00581, δ2 = 0.0331, δ3 = 0.0331
Taking m=2 and a=b, Eq.(5.54) results in at x=y=0:
p0 a 4 p0 a 4
.9963 ) + 8 (π − ( 0.9963 )2 )]
[− 18 11.π549+ 0(.09963 π + 0.9963
wmax = 0.0026 D − 2π 3 D
1

or
p0a 4
wmax = 0.0054 D
General deflection is defined by Eq.(5.53):

) + B0 + ∑ ( Bm cosh
p0 b4 4 y2 2 mπy mπy mπy
w= 384 D (1 − b2 a + Em a sinh a ) cos maπx
m
Here, from Eq.(g) of Sec.5.10, setting a=b and m=2:
p0a 4
B2 = − ( −1)( 2aπ )3 11.π549+ 0(.09963
.9963 ) = 0.000725
p0a
2D D
p0a 4
E2 = ( −1)( 2aπ )3 11.1549 = −0.000028
p0a
2D D
Derivatives of w given above are then, at x=y=0;
∂ 2w p0a 2 ∂ 2w p0a 2
∂x 2
= −0.0286 D , ∂y 2
= 0.0126 D

The bending moment M x ,


M x = − D( ∂∂xw2 + ν ∂ 2w
2

∂y 2
)
at the center (x=y=0) for ν = 0.3 is thus,
p a2
M x = D( 0.0286 + 0.3 × 0.0216) 0D = 0.0324 p0 a 2
Based upon symmetry in bending M x = M y = 0.0324 p0 a .
2

SOLUTION (5.34)

Table 5.4: δ2 = 0,0092 , δ3 = 0.0411 . Then


6 ( 0.0411) p0 a 2
σ1 = = =
0.494(106 )
6 M max
t2 t2 t2
6( 0.0092 ) p0a 2 0.11(106 )
σ2 = = =
6 M min
2 2
t t t2

51
(CONT.)
(5.34 CONT.)
σu
(a) σ1 = n ; 0.494×106
t2
= 200 (106 )
2 ; t = 70 mm
σ1 σ
(b) σ2 − σ uc2 = 1n ; 0.494
100 − 0.11
300 = t 2; t = 67.6 mm

SOLUTION (5.35)

From Example 5.3, pmn = 4 P / ab. Then Eqs.(5.58) leads to


∞ ∞
w = ∑ ∑ ab4πP4 D
nπy
1
[( m/ a )2 + ( n / b )2 ]2 + k
sin maπx sin b
m n

SOLUTION (5.36)

Referring to Eq.(5.11) we write


∞ ∞
w= 4P
π 4 Dab ∑∑
m n
1
[( m/ a )2 +( n / b )2 ]2 + k
sin maπx sin
nπy
b

Substitution of w into expression for moment leads to


M x = − D( ∂∂xw2 + ν ∂ 2w
2

∂y 2
)
∞ ∞
= 4P
π 4 Dab ∑∑m n
m2π 2
[( m/ a )2 +( n / b )2 ]2 + k
( a12 + ν
b2
) sin maπx sin
nπy
b

Maximum moment occurs at center (x=a/2 and y=2a). Thus, for m=n=1, b=4a, and
ν = 0.3:
M x ,max = P 1
π 2a 2 [(1/ a )2 +(1/ 4 a )2 ]2 + k
( a12 + 160.a32 )
It follows that
σ x ,max = = =
6 M x ,max 6P 1 16.3 P
t2 π 2a 2t 2 (17/16 a 2 )+ k 16 a 2 1.61(1.13+ ka 4 ) t 2

SOLUTION (5.37)

Expression (b) of Sec.5.3 becomes


∞ ∞
w=
16 p0
π 6D ∑∑
m n
sin( mπx / a )sin( nπy / b )
mn [( m/ a )2 +( n / b )2 ]2 + k

Maximum deflection occurs at the center (x=a/2 and y=b/2) and is


∞ ∞ m + n −1

wmax =
16 p0
π 6D ∑∑ m n
( −1 ) 2
mn [( m/ a )2 +( n / b )2 ]2 + k

Taking m=n=1 :
wmax =
16 p0 1
π 6 D [(1/ a )2 +(1/ b )2 ]2 + k
200(109 )( 0.003)3
Here D= 12( 0.91) = 494.5 N ⋅ m
Upon substitution of given numerical values
32.423(106 )
wmax = 0.006 = 494.5( π 4 )( 2.0727 )+ k
from which
k = 5403.6 MPa / m

52
SOLUTION (5.38)

f m = Em e λm y . Substitute this into Eq.(f) of Sec. 5.11 to obtain


λm4 − 2 maπ λm2 + ( maπ + Dk ) = 0
2 2 4 4
2 4

The roots of this characteristic equation are


λm1 = βm + iγ m , λm2 = βm − iγ m , λm3 = − βm + iγ m , λm4 = − βm − iγ m
where
βm2 = 12 ( maπ + m4π 4
+ Dk ), γ m2 = 12 ( maπ − m4π 4
+ Dk )
2 2 2 2
2
a4 2
a4
Thus,
f m = E1m e( βm + iγ m ) y + E2 m e ( βm − iγ m ) y + E3m e ( − βm + iγ m ) y + E4 m e ( − βm − iγ m ) y
Let E1m = Am , E2 m = Bm , E3m = Cm , and E4 m = Dm .

Then, using trigonometric identities, the above equation is written in the form given by Eq.(g) of
Sec. 5.11.

SOLUTION (5.39)

Strain energy U is given by Eq.(5.69). We have


mπx1 nπy1
W = Pw| x = x1 = Pa m sin a sin b
y = y1

The condition ∂ (U − W ) / ∂a m = 0 gives


4 P sin( mπx1 / a )sin( nπy1 / b )
a mn = π 4abD [( m/ a )2 +( n / b )2 ]
Introducing this into Eq.(5.1b), we have once more the result (5.11).

SOLUTION (5.40)

∞ ∞
w = ∑ ∑ a mn sin maπx sin
nπy
b ( m, n = 1,3,⋅ ⋅ ⋅⋅)
m n
Loading:
p= ( 0 < x < a2 ), p = 2 p0 − ( a2 < x < a )
2 p0 x 2 p0 x
a a
Thus
∞ ∞
W = ∫∫ wpdxdy = 2∑ ∑ ∫
a


a 2p x nπy
a mn sin maπx sin
2
0
a b dxdy
0 0
m n
∞ ∞
= ∑ ∑ m2 n0π 3 a mn sin m2π
8p a

m n

We have Π = U − W. Hence ∂Π / ∂a mn = 0 gives


32 p0a 4 sin( mπ / 2 )
a mn = m2 nπ 7 D ( m2 + n 2 )2

and w is given by Eq.(P5.40).

53
SOLUTION (5.41)

Boundary conditions are


∂ 2w
∂x 2
= 0, w=0 (x=0, x=a)
(a)
∂ w
= 0, w=0
2

∂y 2
(y=0, y=b)
Integrating the last term in strain energy expression given by Eq.(5.64) by part and substituting
the conditions (a), we obtain
U1 = ∫∫ ( ∂∂ + ∂ 2w 2
2
D w
2 x2 ∂y 2
) dxdy
Deflection is given by
∞ ∞
w = ∑ ∑ a mn sin maπx sin
nπy
b (b)
m n
Introducing Eq.(b) into the strain energy expression
∞ ∞

∫ ∫ ∑ ∑ [a
b a nπy 2
U1 = ( maπ2 + n 2π 2
) sin maπx sin
2 2
D
2 mn b2 b ] dxdy
a 0
m n
∞ ∞
U1 = π 4 Dab
∑∑a ( ma2 +
2
2 n2 2
or 8 mn b2
)
m n
The work done is given by
W = P ⋅ a mn
From Sec.4.9, strain energy due to deformation of elastic foundation is
U2 = ∫ ∫ ∫∫
b a b a nπy
mπx
1
2 kw 2 dxdy = k
2
2
a mn sin 2 a sin 2 b dxdy = abk
8
2
a mn
0 0 0 0
Potential energy is thus,
∞ ∞
Π = U1 + U 2 − W = π 4 Dab
8 ∑∑(
m n
m2
a2
+ b2
) +
n2 2 abk
8
2
a mn − P ⋅ a mn
Applying Ritz method :
∂Π
= P− π ( ma2 + ) + a mn = 0
4 2
Dab n2 2 abk
∂a mn 4 b2 4

or
a mn = 4P 1
ab ( π 4 D + k ) [( m/ a )2 +( n / b )2 ]2
(c)
Deflection is obtained substituting Eq.(c) into (b).

SOLUTION (5.42)

Boundary conditions are,


∂ 2w
w = 0, ∂x 2
=0 (x=0, x=a ) a
∂w
w = 0, ∂y =0 (y=0, y=b ) x
We choose
∞ ∞
w = ∑ ∑ a mn (1 − cos 2 maπx ) sin
nπy b
b (a)
m n
Clearly, boundary conditions are satisfied by Eq.(a).
Work done by uniform load p0 is
y

54
∞ ∞
W = p0 ∫ ∫ ∑ ∑ [a
b a nπy
mn (1 − cos 2 maπx )sin b ]dxdy
0 0
m n
(CONT.)
(5.42 CONT.)

After integration, taking m=n=1, we obtain


W=
2 p0ab
π a11
Strain Energy reduces to Eq.(5.65) :
U= ∫∫ ( ∂∂ + ∂ 2w 2
2
D w
2 x2 ∂y 2
) dxdy
Upon introducing Eq.(a) into the above one has,
∞ ∞

∫ ∫ ∑∑a
b a nπy
U= D
2
2
mn {[( 2 maπ )2 cos 2 maπx sin b ] + [−( nbπ ) 2 (1 − cos 2 maπx ) ⋅
a 0
m n
nπy
sin b ]}2 dxdy
Squaring the terms in parenthesis and integrating, this equation reduces to
U= Db
4
2
a mn [ a2 ( 2 maπ ) 4 + a( 2 maπ ) 2 ( nbπ ) 2 + a( nbπ ) 4 + a2 ( nbπ ) 4 ] ≡ Db
4
2
a mn A0
The Ritz method yields then, setting Π = U − W :
∂Π
∂a mn = 2 A0 a mn − π a mn = Dπ0A0
Db 2 p0ab 4p a
or
For example, letting a=b and m=n=1:
8π 4 4π 4
A0 = + + πa3 + π4
= 13.5π 4
4

a3 a3 2 a3 a3

Then, a11 = 0.000968 p0 a 4 / D .

SOLUTION (5.43)

Potential energy Π, Eq.(5.64), with x = aα and y = bλ:


Π = ab∫ p0 wdαdλ − ∫ [( ∂∂α ) + ( ab ) 2 + ( ∂∂λw2 ) 2 + 2(1 − ν )( ab ) 2 ( ∂α∂λ
∂ w 2
2 2 2
Db w 2
2a3 2 )
A A

+ 2ν ( ab ) 2 ∂ 2w ∂ 2w
∂α 2 ∂λ2
]dαdλ
Introducing w = Cα sin πλ into the above, we obtain :
2

Π = p0 ab∫ ∫ ∫ ∫ [( 2C sin πλ )
1 1 1 1
Cα 2 sin παdαdλ − Db
2a3
2
0 0 0 0

+ ( ab ) 4 ( −π 2 Cα 2 sin π ) 2 + 2(1 − ν )( ab ) 2 ( 2πCα cos πλ ) 2


+ 2ν ( ab ) 2 ( 2C sin πλ )( −π 2 Cα 2 sin πλ )]dαdλ
∂Π
Hence, ∂C = 0 leads to the expression:
p0 ab∫ ∫ ∫ ∫ [8C sin
1 1 1 1
α 2 sin πλdαdλ − 2Dba 3
2
πλ
0 0 0 0

+ 2( ab ) 4 π 4 Cα 4 sin 2 πλ + 16(1 − ν )( ab ) 2 π 2 Cα 2 cos 2 πλ


− 8ν ( ab ) 2 Cπ 2α 2 sin 2 πλ ]dαdλ = 0
After integration, we obtain
a 4 p0
C= D 4
2
3π [ 2+ π10 ( ab )4 + 43 π 2 (1−ν )( ab )2 − 23 π 2ν ( ab )2 ]

w = C( ax ) 2 sin π b
y
Then,
For a / b = 1, ν = 0.3, x = a and y = b / 2 :

55
p0a 4
w = 0.01118 D
This result differs from “exact” solution about 1 percent.

SOLUTION (5.44)

We have
72 (109 )(1×10 −3 )3
D= Et 3
12 (1−ν 2 )
= 12 ( 0.91) = 6.593 N ⋅ m
m = ρt = 2.8( 0.001) = 2.8 kg / m2
Apply Eq.(5.73)
ω n = 4[2( a1 + + ) mD ] 2 = = 431634
1
0.145 0.0625 6.239 D
4
a4 a4 a2 m .
ω
f n = 2πn = 4312.π634 = 68.7 Hz
e = 1−( f1 ) 2 = 1−( 621 ) 2 = 5.39
fn 68.7

and
p d = m g in ⋅ e = 2.8(10 × 9.81)(5.39) = 1.481kPa
From Sec. 5.7, Eq. (e):
pd a 4 4
wmax = 0.00254 D = 0.00254 1, 4816.(593
0.15 )
= 0.289 mm
6 ( 0.0829 pd a 2 )
σ max = 6 M max
t2
= t2
= 16.58 MPa

SOLUTION (5.45)

D = 6.593 N ⋅ m, m = 2.8 kg / m3 , a=b=0.2m, f=50 Hz


We have
fn = 1.446
a2
D
m = 55.5 Hz , e = 1−( f1 2 = 4.03
fn )

p d = m g in ⋅ e = 2.8(10 × 9.81)(4.03) = 1.107kPa


Refer to Eq. (d) of Sec. 5.7:
pd a 4 4
wmax = 0.00126 D = 0.00126 (1,1076.593
)( 0.2 )
= 0.375 mm
and
M max = 0.0513 pd a 2 = 0.0513(1107
, )( 0.2) 2 = 2.272 N
σ max = 6 M max
t2
= 6( 2.272 )
( 0.001)2
= 1363
. MPa

56
CHAPTER 6

SOLUTION (6.1)

( a ) Deflection, taking the first term, from Eq.(6.4) is


16 p0 a 4 2 sin( πx / a )sin( 2πy / a ) 2 sin( πy / a )sin( 2πx / a )
w= π 6D
( (1)( 4−1)(1+ 4 )2
+ (1)( 4−1)(1+ 4 )2
)
16 p0a 4 2πy πy
= 75π 6 D
( 2 sin πax sin a + 2 sin a sin 2aπx ) (a)
At x=a/4 and y=a/4, the above gives
16 p0a 4
w= 75π 6 D
[( 2 )( 2 / 2 )(1) + ( 2)( 2 / 2 )(1)]
or
4
p0a 4
w= 32 2 P0 a
75π 6 D
= 6.28(10 −4 ) D
( b ) Derivatives of Eq.(a) are :
∂ 2w 16 p0 a 4 2πy πy
= ( − 2aπ2 sin πax sin − 8π 2
sin 2aπx )
2

∂x 2 75π 6 D a a2
sin a

∂ 2w 16 p0a 4 2πy πy
= ( − 8aπ2 sin πax sin − 2π 2
sin 2aπx )
2

∂y 2 75π 6 D a a2
sin a

These derivatives, for x=a/4 and y=a/4, yield


p0a 2 p0a 2
∂ 2w
∂x 2
= − 1675×5π 4 2 D , ∂ 2w
∂y 2
= − 1675×5π 4 2 D

Moment, using ν = 0.3, is thus


M x = − D( ∂∂xw2 + ν ∂ 2w
2

∂y 2
)
p0a 2
= − D( − Dπ 4
)( 8075 2 + ν 80 2
75 ) = 0.02 p0 a 2

SOLUTION (6.2)
sin( π 4 )sin( π 4 ) πy sin( π 4 )sin( π 2 ) 2πy
(a) w1 = 4 Pa 2
π 4D
{ 4 sin πax sin a + 25 sin πax sin a }
sin( π 4 )sin( π 4 ) πy sin( π 4 )sin( π 2 ) 2πy
w2 = − 4πPa4 D { sin πax sin − sin πax sin
2

4 a 25 a }
Thus
π π 2πy
w = w1 + w2 = 4 Pa 2 2 sin( 4 )sin( 2 )
π 4D 25 sin πax sin a
2πy
w = 2.322(10 −3 ) PaD sin πax sin
2

At x = a4 = y and ν = 0.3
. (10 −2 ) Pa
w = 1794
2

Et 3
( b ) Using Eq.(3.9),
2πy 2πy
M x = 2.322(10 −3 ) Pa 2 {( πa ) 2 sin πax sin a + ν ( 2aπ ) 2 sin πax sin a }
= 0.0356 P
SOLUTION (6.3)
( a ) Equation (6.4), with m=1, n=2 and m=2, n=1, at x = a
3 , y = a2 :
16 p0 a 4 2 sin( π3 )sin( π ) 2 sin( 23π )sin( π2 ) p a4
w= π D
6 [ 3( 25 ) + 3( 25 ) ] = 0.3695 π06 D
p a4
= 39413
. 0
π 6 Et 3
(CONT.)

57
(6.3 CONT.)
∂ 2w −32 p0 a 4 2πy πy
(b) ∂x 2
= 75π 6 D
( πa ) 2 [sin πax sin a + 4 sin 2aπx sin a ]
∂ 2w −32 p0 a 4 2πy πy
∂y 2
= 75π 6
( πa ) 2 [4 sin πax sin a + sin 2aπx sin a ]
∂ 2w 64 p0a 4 2πy πy
∂x∂y = 75π 6 D
( πa )2 [cos πax cos a + cos 2aπx cos a ]

Moments at x = a
3 and y = a 2 :
32 p0 a 2
M x = − D( ∂∂xw2 + ν ∂ 2w
)= ( 23 )( 4 + ν )
2

∂y 2 75π 4
32 p0a 2
My = 75π 4
( 23 )(1 + 4ν )
32 p0a 2
M xy = 75π 4
(1 − ν )

Maximum stresses are thus


σx = = 9.607 π 04 ( at ) 2 σ y = 5173 τ xy = 1707
6 Mx p p p0
t2
. π
( at ) 2
0
4 . π
( at ) 2
4

2
σ1, 2 = p0a
π 4t 2
[ 9.6072+5.173 ± ( 9.6072−5.173 ) 2 + 1707
. 2]
σ1 = 1019 σ 2 = 4.59 π ( at ) 2 τ max = 2.80 πp ( at )2
p0 p0
. π ( at ) 2 4 4
0
4

SOLUTION (6.4)

Substitution of Eq.(6.7) into Eqs.(3.9) leads to


Mx = [1 + ν − (1 − ν )]
M0 3x
2 a

My = [1 + ν + 3ax (1 − ν )] , M xy = (1 − ν ) y
M0 3 M0
2 2a

To determine twisting moment along side AC, we use Eq.(3.12) together θ = 60 o (Fig.3.5b) and
ν = 13 then
M x ' y ' = − 12 ( M x − M y ) sin 2θ + M xy cos 2θ )
3(1−ν ) M 0
= ( 3x − y ) = ( 3x − y )
M0
4a 2a
Concentrated reactions at the corners are thus,
Rc = −2 M x ' y ' = − = −1155 (x = , y = 0)
2 M0 2a
3
. M0 3

SOLUTION (6.5)

( a ) From Sec.6.3, σ max = 0155


. p0 ( at ) 2 . Thus
100(10 6 ) = 0155
. (5 × 10 6 )( 0.24) 2 / t 2
or t=21.1 mm

200 (109 )( 211. ×10 −3 )3


(b) D= Et 3
12 (1−ν 2 )
= 12 ( 0.91) = 172.05 kN ⋅ m
Equation (6.9) at x=y=0:
p0a 4 5(106 )( 0.24 )4
w0 = 972 D == 0.099 mm
972 (172, 050 )

Note: It can be verified that wmax = w0

58
SOLUTION (6.6)

Equations (6.11) ae = 0.35a = 84 mm.


( a ) From Sec. 4.5, σ max = 34 p0 ( at ) 2 . e
Hence
100(106 ) = 34 (5 ×106 ) (0.084) t 2
from which t = 16.3 mm

200(109 )(16.3×10−3 )
(b) D= Et 3
= = 79.318 kN ⋅ m
12(1−ν 2 ) 12(0.91)

From Sec. 4.5 at x=y=0:


p0 ae4 5(106 )(0.084)4
wmax = 64 D = 64(79,318) = 0.049 mm

SOLUTION (6.7)
Section 6.3, σ y ,max = 0155
. p0 ( a t ) 2 . Hence

= 0155 a = 272.6 mm
2
240 a
1.5 . ( 2) ( 0.012 )2
:
and LAB = 2a
3
= 314.8 mm
200 (109 )(12×10 −3 )3
We have D = = = 316484 kN ⋅ m
3
Et
12 (1−ν 2 ) 12 (1− 0.09 ) .
Equation (6.9) for x=y=0:
p0a 4 2(106 )( 0.2726 )4
wmax = 972 D = 972( 31, 648.4 ) = 0.36 mm

SOLUTION (6.8)

Equations (6.11): ae = 0.35a.


σ max
Section 4.5: n = 34 p0 ( ate ) 2 . Thus
2
240
1.5 = 34 (2) (0.012)
ae
2 , ae = 123.9 mm
So a = 123.9 0.35 = 354 mm
and
LAB = 2a
r3 = 408.8 mm
We have
200(109 )(12×10−3 )3
D = 12(1Et−ν 2 ) = = 31.6484 kN ⋅ m
3

12(0.91)

p0 ae4 2(106 )(0.1239)4


Section 4.5: wmax = 64 D = 64(31,648.4) = 0.233 mm

SOLUTION (6.9)
200 (109 ) t 3
D= Et 3
12 (1−ν 2 )
= 12 (1− 0.09 ) = 18.315(109 )t 3
Equation (c) of Sec. 6.4:
k= a 4 b4
p0
8 D 3a 4 + 2 a 2 b 2 + 3 b 4
5(106 ) ( 0.3 )4 ( 0.2 )4
= 8 D 3( 0.3 )4 + 2 ( 0.3 )2 ( 0.2 )2 +3( 0.2 )4 = 223.14
D

(CONT.)

59
(6.9 CONT.)

( a ) Equation (6.15):
M y ,max = M max = =− = 44.628 kN ⋅ m
8 Dk 8 ( 223.14 )
b2 ( 0. 2 ) 2
and
σ max = ; 120 × 10 6 =
6 M max 6 ( 44 , 628 )
t2 t2
or t=47.2 mm

( b ) Equation (6.13):
wmax =
p0 1
8D ( 3 )+ ( 2 )+( 3/ a 4 )
b4 a 2b2

5(106 )
= 8 (18.315 )(109 )( 47.23 ×10 − 9 ) 3 + 2
1
+ 3
( 0 . 2 )4 ( 0 .3 ) 2 ( 0 . 2 )2 ( 0 .3 )4

= 0116
. mm

SOLUTION (6.10)

We have b=2c, a=c and b=2a.

( a ) Rectangle ( Table 5.1 ):


p0a 4 p0c 4
( wmax ) r = 0.01013 D = 011062
. Et 3
, ( M max ) r = 01017
. p0 c 2

Ellipse ( Table 6.1 ):


p0 ( c 2 )4 p0 c 4
( wmax ) e = 158
. Et 3
= 0.09875 Et 3
, ( M max ) e = 0.09475 p0 c 2
Thus,
( wmax )r
( wmax )e = 112
. ; ( M max )r
( M max )e = 107
.

6 ( 0.1017 )( p0 )r ( 0.22 )2
(σ max ) r = ; 270(10 6 ) =
6 ( M max )r
(b) t2 ( 0.01)2
or
( p0 ) r = 914.2 kPa
And
6 ( 0.09475 )( p0 )r ( 0.22 )2
270(10 6 ) = ( 0.01)2
, ( p0 ) r = 9813
. kPa

SOLUTION (6.11)

Equation (6.18), for n=1 at r = 3a


4 and θ = π 2 :
p0a 4 4 ( 3 a 4 )4 ( 3 a 4 )3
w= + −
3a
1 4 1+5+ 0.3 1+3+ 0.3
D [ a4 π (16 −1)( 4−1) a π (16−1)( 2+1)[1+ (1.3 2 )] a 3 π ( 4 +1)( 4 −1)[1+(1.3 2 )]
]
p0a 4
= 0.00588 D

Using Eq. (6.19) and Table 6.2 :


p0 a 4 p0a 4
w = δ1 D = 0.0560 D ( ar = 43 , α = π )

SOLUTION (6.12)

60
( a ) Referring to Table 6.1, for a/b=2 :
σ max = = = 20 × 10 6
6 M max 6 ( 0.0237 p0 )
t2 ( 0.02 )2
or
p0 = 5.625 × 10 4 Pa = 56.26 kPa
Center deflection is then
p0b4 56260 ( 0.25 )4
wmax = 158
. Et 3
= 158
. E ( 0.02 )3
= 4.3404×107
E

( b ) Expression (c) of Sec.6.4 yields


k= = 0.0625× 0.003906
p0 a 4 b4 p0
8 D 3 a 4 + 2 a 2 b 2 + 3b 4 D 8 ( 0.1875+ 0.03125+ 0.011718 )
−4 p0
= 1324
. × 10 D
Center moments, using Eqs.(d) of Sec.6.4, are
M x = 4 p0 (1324
. × 10 −4 )( 4 + 4.8) = 4.66 × 10 −3 p0 , M y = 9109
. × 10 −3 P0

End moments of minor and major axes, using Eqs.(6.15):


M x = −4.237 × 10 −3 p0 , . × 10 −3 p0
M y = 0.3 M x = −127 ( x = ±a, y = 0)
M y = −16.947 × 10 −3 p0 , M x = 0.3 M y = −5.084 × 10 −3 p0 ( x = 0, y = ± b)

Hence,
6 (16.947×10 −3 )
20 × 10 6 = ( 0.02 )2
p0 , or p0 = 78.677 kPa
−4
Maximum deflection is then : wmax = 1324
. × 10 p0 / D = 14219
. × 10 7 / E.

SOLUTION (6.13)

Maximum deflection
takes place at center of
plates, We have
p0 c 4 c 2c
( wmax ) c = 64 D x 0 x
0
( wmax ) e =
p0 1
8 D [( 3/ c 4 )+( 2 / 4 c 4 )+( 3/16 c 4 )]
c
Hence,
y y
= 21695
( wmax )e
( wmax )c .
Similarly,
p0 a 4 b4
( M y ,max ) e = − 8bDk2 , k= 8 D ( 3 a 4 + 2 a 2 b 2 + 3b 4 )
p0 c 2
( M max ) c = − 8

Substituting a=2c and b=c :


( M y ,max )e
( M max )c = 21695
.

SOLUTION (6.14)

61
C=
p0
Let 24 D [( 5/ a 4 )+(1/ b4 )+( 2 / a 2 b2 )]
Deflection is then written in the form,
w = Cx[1 − ( ax ) 2 − ( b ) 2 ]
y
(a)

Boundary conditions are


w = 0, ∂w / ∂x = 0 , ∂w / ∂y = 0 ( x = ± a, y = 0; x = 0, y = ± b)
The governing equation is
∇ 4 w = p0 / D
Derivatives of w are obtained as follows
∂w ∂w
= C[1 − ( ax ) 2 − ( b ) 2 ]2 − Cx 2 [1 − ( ax ) 2 − ( b ) 2 ] , = −4C b2 [1 − ( ax ) 2 − ( b ) 2 ]
y 4 y xy y
∂x a2 ∂y

Similarly,
∂ 2w
= −12C ax2 [1 − ( ax )2 − ( b ) 2 ] + 8C ax4
y 3

∂x 2
∂ 2w xy 2
= −4C bx2 [1 − ( ax ) 2 − ( b ) 2 ] + 8C
y
∂y 2 b4

∂ 2w xy 2
= −4C b2 [1 − ( ax ) 2 − ( b ) 2 ] + 8C a 2b2
y y
∂x∂y

It is readily verified that w and its derivatives satisfy the boundary conditions and the governing
equation. Moments are found as
ν νy 2
M x = 4 DCx[1 − ( ax ) 2 − ( b ) 2 ]( a32 + ) − 8 DCx ( ax 2 +
y 2

b2 b4
)
2

M y = 4 DCx[1 − ( ax ) 2 − ( b ) 2 ]( b12 + ) − 8 DCx ( b4 + νax4 )
y y 2

a2
y2 x2 y
M xy = D(1 − ν ){4C [1 − ( ax ) 2 − ( b ) 2 ] − 8C a 2b2 }
y
b2

For a=2b and x=y=0, the above reduce to M x = M y = M xy = 0 .


p0νa 5
For a=2b, y=0, and x = ± a , we have M x = ± 87 . Stress is thus,
6νp0a 5
σx = = ( x = ±a, y = 0)
6 Mx
t2 87 t 2

SOLUTION (6.15)

Maximum deflection occurs at center of both plates. Thus using Eq. (4.22) at r=0:
p0c 4 p0c 4 12 (1−ν 2 )( 5+ν )
( wmax ) c = 64 D ( 15++νν ) = 64 Et 3 1+ν
Maximum moment employing Eq.(4.23) and setting r=0,
p0 ( 3+ν ) c 2
( M max ) c = ( M r ,max ) c = ( Mθ ,max ) c = 16
We have, from Table 6.1 for b=c and a=2c :
( wmax ) e = 158
. p0 c 4 / Et 3 , ( M max ) e = 0.379 p0 c 2
It follows that
( wmax ) e / ( wmax ) c = 2.2713, ( M max ) e / ( M max ) c = 18376
.

62
CHAPTER 7

SOLUTION (7.1)

Equation (a) of Sec.7.2:


δ 3 y n = 12 ( y n+ 2 − 2 y n+1 + 2 y n−1 − y n−2 ) ≈ h 3 ( ddx y ) n
3

Referring to Fig. 7.1b, Sec. 7.2, and the foregoing, we have:


∂ 3w ∂
= ( ∂∂xw2 ) = ( w5 − 2w2 + w6 − w8 + 2w4 − w7 )
2
1
∂x 2∂y ∂y 2 h3
∂ 3w ∂
= ( ∂∂yw2 ) = ( w5 − 2w1 + w8 − w6 + 2w3 − w7 )
2
1
∂x∂y 2 ∂x 2 h3
and
Vx = − D[ ∂∂xw3 + ( 2 − ν ) ∂∂x∂wy 2 ]
3 3

= − 2Dh3 {w9 − 2(3 − ν )w1 + 2(3 − ν )w3 + w11 + ( 2 − ν )[w5 + w8 − w6 − w7 ]}


Q.E.D.
We obtain the same result using Table 7.1. Similarly, using Eq.(3.23b):
V y = − 2Dh3 {w10 − 2(3 − ν )w2 + 2(3 − ν )w4 + w12 + ( 2 − ν )[w5 + w6 − w7 − w8 ]}
Q.E.D.

SOLUTION (7.2)

Referring to Table 7.1, we write


wn +1 − 2wn + wn −1 = h 2 ( EI
M
)n (P7.2)

Applying Eq. (P7.2) at points 1 through 5 gives, respectively,

w2 − 2w1 + w0 = PL
9 EI h2
w3 − 2 w2 + w1 = 2 PL
9 EI h2
w4 − 2w3 + w2 = PL
6 EI h2 (a)
w5 − 2 w4 + w3 = PL
9 EI h 2

w6 − 2 w5 + w4 = 18PLEI h 2

For this case, h = L/6 and the boundary conditions are w0 = w6 = 0 . Then
Eqs. (a) may be represented in matrix form:

⎡ −2 1 0 0 0 ⎤ ⎧ w1 ⎫ ⎧2 ⎫
⎢ 1 −2 1 0 0 ⎥ ⎪ w ⎪ ⎪4 ⎪
⎢ ⎥ ⎪⎪ 2 ⎪⎪ ⎪⎪ ⎪⎪
⎢ 0 1 −2 1 0 ⎥ ⎨ w3 ⎬ = ⎨3 ⎬ C
⎢ ⎥⎪ ⎪ ⎪ ⎪
⎢ 0 0 1 −2 1 ⎥ ⎪ w4 ⎪ ⎪2 ⎪
⎢⎣ 0 0 0 1 −2 ⎥⎦ ⎩⎪ w5 ⎭⎪ ⎩⎪1 ⎭⎪

(CONT.)

63
(7.2 CONT.)
Here C = PL 648 EI . Solving the preceding, we obtain
3

w1 = −6.67C , w2 = −11.33C , w4 = −9.67C , w5 = −5.33C , w3 = −12C


Thus,
w3 = −0.01852 PL
EI = 0.01852 EI ↓
3 3
PL

Interestingly, the exact value of the deflection at the center (see Table B.5) is
0.01775 PL3 EI .

SOLUTION (7.3)

The boundary conditions w(0) = 0 and w '(0) = 0 , referring to Table 7.1, yield

w0 = 0 w1 = w−1 (a)

We have M 0 = −3Pa, M 1 = −2 Pa, M 2 = Pa, and M 3 = − Pa 2. When Eq. (P7.2)


is used at points 0, 1, 2, and 3, the following expressions are obtained:

w1 − 2w0 + w−1 = − 32Pa


EI h
2

w2 − 2 w1 + w0 = − Pa
EI h
2

w3 − 2 w2 + w1 = − 1.5PaEI h 2 (b)
w4 − 2w3 + w2 = − 2PaEI h 2

Note that at node 2, the average flexural rigidity is used. From Eqs. (a) and (b),

w1 = − 3C 4, w2 = − 5C 2, w3 = − 59C 12, w4 = − 47C 6

where C = ( Pa EI ) h . Therefore, after setting h = a 2 , we obtain


2

wB = w4 = − 4724PaEI = ↓
3
47 Pa3
24 EI

We note that, this deflection is approximately 2.2 percent larger than the exact
value.

SOLUTION (7.4)

In labeling nodal points, it is important too take into account any conditions of symmetry which
may exist. This has been done in Fig. P7.4b. We observe that only half of the beam span need be
considered. From the boundary conditions

w(0) = w "(0) = 0 (Table 7.1) and symmetry


we obtain:

(CONT.)

64
(7.4 CONT.)
w0 = w4 = 0 w1 = − w−1 w1 = w3 (a)

Referring to Table 7.1 at points 1 and 2:

w3 − 4w2 + 6w1 − 4w0 + w−1 = 0


(b)
w4 − 4w3 + 6w2 − 4 w1 + w0 = − Ph 4
EI

Introducing Eqs. (a) and (b), we find

6w1 + 4w2 = 0
−4 w1 + 3w2 = − 2PhEI
4

Solving the foregoing and setting h = 4 L gives w1 = − 0.0039 PL EI and


4

w2 = wmax = −0.0059 PL
EI = 0.0059 EI ↓
4 4
PL

From Table 7.1, we then obtain


θA = ( w1 − w−1 ) = −0.0156 PL
EI = 0.0156 EI
3 3
1 PL
2h

SOLUTION (7.5)

3 p0
p0 4
w0 = 0, w1 = − w−1 p0 2 p0
(a) 4 h= L
4
w4 = 0, w3 = − w5
x
-1 0 1 2 3 4 5
L

Refer to Table 7.1


wn+ 2 − 4wn +1 + 6wn − 4wn −1 + wn− 2 = h 4 ( EI ) n
p
(b)

Apply Eq.(b) at points 1, 2, and 3:


w3 − 4w2 + 6w1 − 4w0 + w−1 = L4 p0
256 4 EI

w4 − 4w3 + 6w2 − 4w1 + w0 = L4 p0


256 2 EI (c)
w5 − 4w4 + 6w3 − 4w2 + w1 = L4 p0
256 4 EI

Simultaneous solution of Eqs.(a) and (c) gives


p0 L4 p0 L4 p0 L4
w1 = 0.00464 EI w2 = 0.00684 EI w3 = 0.00513 EI

0.00684−( 5 768 )
Error: 5
768
× 100 = 51
. %

65
SOLUTION (7.6)
Due to symmetry, only ¼ of the surface need be considered.
Let N = − p0 h = − p0 a2
. The ∇ of Table 7.1 applied to ∇ M = − p at the nodes 1 to 6:
2 2 2
16

⎡− 4 1 0 1 0 0 ⎤ ⎧ M1 ⎫ ⎧1⎫
⎢ 1 −4 1 0 1 ⎥
0 M2 ⎪ ⎪ ⎪1⎪
⎢ ⎥⎪ ⎪ ⎪⎪
⎢ 0 2 −4 0 0 1 ⎥⎪ M 3 ⎪ ⎪1⎪
⎢ ⎥⎨ ⎬ = N ⎨ ⎬
⎢ 2 0 0 −4 1 0 ⎥⎪ M 4 ⎪ ⎪1⎪
⎢ 0 2 0 1 − 4 1 ⎥⎪ M 5 ⎪ ⎪1⎪
⎢ ⎥⎪ ⎪ ⎪⎪
⎣ 0 0 2 0 2 − 4⎦ ⎩ M 6 ⎭ ⎩1⎭
Solving M1 = −0.7754 N M 2 = −11031
. N M 3 = −11937
. N
M 4 = −0.9985N M 5 = −14433
. N M 6 = −15685
. N
p0 h 4 p0 a 4
Similarly with R = D = 256 D :
⎡− 4 1 0 1 0 0 ⎤ ⎧ w1 ⎫ ⎧− 0.7754⎫
⎢ 1 −4 1 0 1 0 ⎥ ⎪w2 ⎪ ⎪ − 11031
. ⎪
⎢ ⎥⎪ ⎪ ⎪ ⎪
⎢ 0 2 −4 0 0 1 ⎥ ⎪ w3 ⎪ ⎪ − 11937
. ⎪
⎢ ⎥⎨ ⎬ = R⎨ ⎬
⎢ 2 0 0 − 4 1 0 ⎥ ⎪w4 ⎪ ⎪ − 0.9985⎪
⎢ 0 2 0 1 − 4 1 ⎥ ⎪ w5 ⎪ ⎪ − 14433
. ⎪
⎢ ⎥⎪ ⎪ ⎪ ⎪
⎣ 0 0 2 0 2 − 4⎦ ⎩w6 ⎭ ⎩ − 15685
. ⎭
Solving
w1 = 0.7815 R w2 = 12695
. R w3 = 14304
. R
w4 = 10812
. R w5 = 17631
. R w6 = 19889
. R
It is seen that the largest moment and deflection occur at point 6. Thus
p0a 2
M max = M y , 6 = − 8 [( w3 − w6 ) + ν ( w5 − w6 )] = 0.79 p0 a 2
σ max = 6 M max
t2
= 6 ( 0.079 )
t2
a 2 p0 = 0.474 p0 ( at ) 2
Error: 0.4872− 0.474
0.4872 × 100 = 2.7 %
p0a 4
And wmax = w6 = 1256 D p0 a = 0.00777
.9889 4
D

Error: 0.00772 − 0.00777


0.0077 × 100 = −0.6 %

SOLUTION (7.7)
11 1 2 3
Due to symmetry, only ¼ of the 7 8 9 10
surface need be considered. x
a
1 11 1 2 3 2 1 4

4 12 4 5
At x=0: w7 = w8 = w9 = w10 = 0 6 5 4 a
at y=0: w11 = w12 = 0 1 2 3 2 1

a
y 4
15
. a (CONT.)
(7.7 CONT.)

66
The ∇ of Table 7.1 is applied to ∇ w = p0 / D at nodes 1 to 6:
4 4

3w1 + w3 + 2( w5 ) − 8( w4 + w2 ) + 20w1 =
p0
D h4
3w2 + 2( w4 + w6 ) − 8( w1 + w3 + w5 ) + 20w2 =
p0
D h4
2w1 + 2w3 + 2( 2w5 ) − 8( w6 + 2w2 ) + 20w3 =
p0
D h4
w4 + w6 + 2( w2 ) − 8( 2w1 + w5 ) + 20w4 =
p0
D h4
w5 + 2( 2w1 + 2w3 ) − 8( 2w2 + w4 + w6 ) + 20w5 =
p0
D h4
2w4 + 2( 4w2 ) − 8( 2w3 + 2w5 ) + 20w6 =
p0
D h4
or
23w1 − 8w2 + w3 − 8w4 + 2w5 + 0w6 =
p0
D h4
− 8w1 + 23w2 − 8w3 + 2w4 − 8w5 + 2w6 =
p0 4
D h
2w1 − 16w2 + 22w3 + 0w4 + 4w5 − 8w6 =
p0 4
D h
− 16w1 + 4w2 + 0w3 + 21w4 − 8w5 + w6 =
p0 4
D h
4w1 − 16w2 + 4w3 − 8w4 + 21w5 − 8w6 =
p0 4
D h
0w1 + 8w2 − 16w3 + 2w4 − 16w5 + 20w6 =
p0 4
D h

In matrix form, we have


⎡ 23 − 8 1 −8 2 0 ⎤ ⎧ w1 ⎫ ⎧1⎫
⎢ − 8 23 − 8 2 −8 2 ⎥ ⎪w2 ⎪ ⎪1⎪
⎢ ⎥⎪ ⎪ ⎪⎪
⎢ 2 − 16 22 0 4 − 8 ⎥ ⎪ w3 ⎪ 4 ⎪1⎪

⎥⎨ ⎬ =
p0 h
⎢ D ⎨ ⎬
⎢− 16 4 0 21 −8 1 ⎥ ⎪w4 ⎪ ⎪1⎪
⎢ 4 − 16 4 −8 21 − 8 ⎥ ⎪ w5 ⎪ ⎪1⎪
⎢ ⎥⎪ ⎪ ⎪⎪
⎣ 0 8 − 16 2 − 16 20 ⎦ ⎩w6 ⎭ ⎩1⎭
Solving,
p0 h 4 p0a 4 p0 h 4 p0a 4
w6 = wmax = 0.788 D = 0.003078 D and w3 = 0.506 D = 0.00198 D

Error in maximum deflection:


× 100 = 40 %
0.003078− 0.0022
0.0022

( M ) = [2w10 − 2w3 + ν ( 2w10 − w9 − w9 )]


y 10
D
h2
p0 h 4
= D
h2
( −2w3 ) = D
h2
( −2 × 0.506 D ) = −1012
. p0 h 2 = −0.06325 p0 a 2

(σ y ) max =
6 ( M y )10
t2
= 6
t2
( −0.06325 p0 a 2 ) = −0.3795 p0 ( at )2

Error
0.3795− 0.4542
0.4542 × 100 = 16 %

SOLUTION (7.8)

67
We have

1 2 p1 = p4 = [16( 20 ) + 4( 20 ) + p0 + 4 p0 + 4( 20 )]
1 p p p
36
3 4
=
p0
a 2
h = a4 p2 = p3 = 1
[16 p0 + 4 p0 +
p0
+4
p0
+
p0
+
36 2 2 2

4 p0 + +4 +
p0 p0 p0
2 2 2 ]
p0 = 5
6 p0

a
2
a
Then
⎡− 4 1 0 1 ⎤ ⎧ M1 ⎫ ⎧ 12⎫
⎢ 2 −4 1 0 ⎥ ⎪⎪ M 2 ⎪⎪ ⎪5 ⎪
⎢ ⎥ ⎨ ⎬ = − ⎪⎨ 6 ⎪⎬ p h 2
⎢ 0 2 − 4 2 ⎥⎪ M 3 ⎪ 5
⎪ 6⎪
0

⎢ ⎥
⎣ 2 0 1 − 4⎦ ⎪⎩ M 4 ⎪⎭ ⎪⎩ 1 2 ⎪⎭
Solving
M1 = 0.40625 p0 h 2 M 2 = 0.60417 p0 h 2
M 3 = 0.77083 p0 h 2 M 4 = 0.52083 p0 h 2

Hence, ∇ w = −
2 M
D :
⎡− 4 1 0 1 ⎤ ⎧ w1 ⎫ ⎧0.40625⎫
⎢ 2 −4 1 ⎥ ⎪
0 ⎪w2 ⎪ ⎪ ⎪0.60417⎪
⎢ ⎥ ⎨ ⎬ = − ⎪⎨ ⎪ p0h4
⎬ D
⎢0 2 − 4 2 ⎥ ⎪ w3 ⎪ ⎪0.77083⎪
⎢ ⎥
⎣2 0 1 − 4⎦ ⎪⎩w4 ⎪⎭ ⎪⎩0.52082⎪⎭
or
p0 h 4 p0a 4
w1 = 0.341145 D = 0.001326 D
p0 h 4 p0a 4
w2 = 0.48958 D = 0.001912 D
p0 h 4 p0a 4
w3 = 0.67187 D = 0.002624 D = wmax
4 4
w4 = 0.46875 = 0.001831
p0 h p0a
D D

SOLUTION (7.9)

Refer to the solution of Example 7.5. We have h=a/4 and ν = 0.3.


Thus, Mx = D
h2
[( 2w22 − w21 − w23 ) + ν ( 2w22 − w14 − w29 )]
4
= [( 2 × 0.46598 − 0.30383 − 0.54558)
D p0 h
h2 D
+ 0.3( 2 × 0.46598 − 0.38943 − 0)]
= 0.01533 p0 a 2
(CONT.)
(7.9 CONT.)

68
My = D
h2
[( 2 w22 − w14 − w29 ) + ν ( 2w22 − w21 − w23 )]
+ 0.3( 2 × 0.46598 − 0.30383 − 0.54559)]
= 0.03545 p0 a 2

D (1−ν ) p0 h 4
M xy = 4 h2
⋅ D ( 0.45037 − 0.25819 − 0 + 0 ) = 0.0021 p0 a 2
Then,
Mx + M y Mx − M y 2
M max = + [( ) + M xy2 ] 2 = 0.035666 p0 a 2
1
2 2

Hence, σ max = 6 M max / t 2 = 0.214 p0 a 2 / t 2 .

SOLUTION (7.10)

Refer to the solution of Example 7.5. We have h = a / 4 = 1 / 2 m .


Maximum principal strains at node 24; z = t
2:

15 16 17
3a
4 22 23 24 25 26
30 31

a
2
ε x = − z ∂∂xw = − 6 Etp h (1 − ν 2 )( w23 − 2w24 + w25 )
2
0
2 2

6 p0 h 2
=− Et 2
(1 − ν 2 )[0.54558 − 2( 0.63989) + 0.96226]
p0 h 2
= −13684
. Et 2
(1 − ν 2 )
2
ε y = − z ∂∂yw = − 6 Etp h (1 − ν 2 )( w16 − 2w24 )
2
0
2 2

6 p0 h 2 p0 h 2
=− Et 2
(1 − ν 2 )[0.51951 − 2( 0.63989)] = 4.5616 Et 2
(1 − ν 2 )
2
γ xy = −2 z ∂∂x∂wy = − 12Etp h (1 − ν 2 )( w17 − w15 )
2
0
2

12 p0 h 2 p0 h 2
=− Et 2
(1 − ν 2 )( 0.70839 − 0.45037) = −3.0962 Et 2
(1 − ν 2 )
Thus
εx +ε y εx −ε y 2 γ xy 2
ε1, 2 = 2 ± ( 2 ) +( 2 )
2
= (1 − ν 2 ){ −1.36842+4.5616 ± [( −1.36842− 4.5616 ) 2 + ( −3.0962
p0 h 1

Et 2 2 )2 ] 2 }
or
p0 h 2
ε1 = 4.9415 Et 2
(1 − ν 2 )

Substituting the given data:


4.9415(15×103 )( 1 2 )2
ε1 = ( 200×109 )( 0.02 )2
( 0.91) = 211μ

SOLUTION (7.11)

69
Due to symmetry, only ½ of the surface need be considered. Uniformly load : p0 , h = a3 ,
Finite difference expressions are, using ∇ w = p0 / D :
4

1 2 3

a
1 2 3 2 1 3
-1
4 5 6 5 4 a
-2

-4 -5 -7
a
3
2a
w3 + w1 − w1 + 2( w5 ) − 8( w2 + w4 ) + 20w1 =
p0
D h4
2w2 + 2( w4 + w6 ) − 8( w1 + w3 + w5 ) + 20w2 =
p0
D h4
2w1 + w3 + 2( 2w5 ) − 8( w6 + 2w2 ) + 20w3 =
p0
D h4
− 2w4 + w6 + 2( w2 ) − 8( 2w1 + w5 ) + 20w4 =
p0
D h4
− w5 + w5 + 2( w1 + w3 ) − 8( w2 + w4 + w6 ) + 20w5 =
p0
D h4
− w6 + 2w4 + 2( 2w2 ) − 8( 2w5 + 2w3 ) + 20w6 =
p0
D h4
or
20w1 − 8w2 + w3 − 8w4 + 2w5 + 0w6 =
p0
D h4
− 8w1 + 22w2 − 8w3 + 2w4 − 8w5 + 2w6 =
p0
D h4
2w1 − 16w2 + 21w3 + 0w4 + 4w5 − 8w6 =
p0 4
D h
− 8w1 + 2w2 + 0w3 + 18w4 − 8w5 + w6 =
p0 4
D h
2w1 − 8w2 + 2w3 − 8w4 + 20w5 − 8w6 =
p0 4
D h
0w1 + 4w2 − 8w3 + 2w4 − 16w5 + 19w6 =
p0 4
D h
In matrix form, we write
⎡ 20 −8 1 −8 2 0 ⎤ ⎧ w1 ⎫ ⎧1⎫
⎢−8 22 −8 2 −8 2 ⎥ ⎪w2 ⎪ ⎪1⎪
⎢ ⎥⎪ ⎪ ⎪⎪
⎢ 2 − 16 21 0 4 − 8 ⎥ ⎪ w3 ⎪ p0 h 4 ⎪1⎪
⎢ ⎥⎨ ⎬ = ⎨⎬
⎢−8 −8 D
2 0 18 1 ⎥ ⎪w4 ⎪ ⎪1⎪
⎢ 2 −8 2 −8 20 − 8 ⎥ ⎪ w5 ⎪ ⎪1⎪
⎢ ⎥⎪ ⎪ ⎪⎪
⎣ 0 4 −8 2 − 16 19 ⎦ ⎩w6 ⎭ ⎩1⎭

Solving,
p0 h 4 p0a 4 p0 h 4 p0 a 4
w1 = 0.259 D = 0.0032 D , w2 = 0.387 D = 0.0048 D
p0 h 4 p0 a 4 p0 h 4 p0 a 4
w3 = 0.422 D = 0.0052 D , w4 = 0.304 D = 0.0038 D
p0 h 4 p0a 4 p0 h 4 p0a 4
w5 = 0.460 D = 0.0057 D , w6 = 0.504 D = 0.0062 D = wmax

SOLUTION (7.12)

70
Note that w=0 and M=0 at the boundary. The vertical distance from point 2 to the boundary is
denoted by h1 . At node 2:
x 2 + y 2 = a 2 is ( a3 ) 2 + y 2 = a 2
h1
or y= 8
9 a.
2
Hence, h1 = ( − 23 )a = 0.276a.
8 a
3
9

Because of symmetry, only 1 8 of the circle need to be considered. For nodes 1, 3, 4, 5


standard finite difference operator applies, Fig.P7.12. For node 2 the irregular star of Fig.7.9b
applicable. We thus have
2 M 2 + M 3 − 4 M1 = − p0 a 2 / 9
M1 + 2 M 4 + M 5 − 4 M 3 = − p0 a 2 / 9
2 M 2 + 2 M 3 − 4 M 4 = − p0 a 2 / 9
4 M 3 − 4 M 5 = − p0 a 2 / 9
2
1+ 0 . 276
M 4 + M1 − [ 0.2276 + 2] M 2 = − p0 a 2 / 9
0 . 333 0 . 333

Solving,
M1 = 01316
. p0 a 2 M 2 = 01009
. p0 a 2 M 3 = 0.2135 p0 a 2
M 4 = 01850
. p0 a 2 M 5 = 0.2413 p0 a 2
Now apply ∇ 2 w = − M / D to obtain, similarly,
2w2 + w3 − 4w1 = −01316
. p0 a 4 / 9 D
w1 + 2w4 + w5 − 4w3 = −0.2135 p0 a 4 / 9 D
2w2 + 2w3 − 4w4 = −01850
. p0 a 4 / 9 D
4w3 − 4w5 = −0.2413 p0 a 4 / 9 D
. w4 + w1 − 4.413w2 = −01009
1094 . p0 a 4 / 9 D
Solution is
w1 = 0.02084 p0 a 4 / D w2 = 0.01518 p0 a 4 / D w3 = 0.03836 p0 a 4 / D
w4 = 0.03191 p0 a 4 / D w5 = 0.04506 p0 a 4 / D = wmax

SOLUTION (7.13)

Finite difference expressions are, using ∇ M = − p0 :


2

M 2 + M 5 − 4 M 1 = − p0 h 2 M 2 + M 7 + M 4 − 4 M 3 = − p0 h 2
2 M 1 + M 6 − 4 M 5 = − p0 h 2 2 M 3 + M 6 + M 8 − 4 M 7 = − p0 h 2
M 1 + M 3 + M 6 − 4 M 2 = − p0 h 2 . M 3 + M 8 − 5 M 4 = − p0 h 2
12
2 M 2 + M 5 + M 7 − 4 M 6 = − p0 h 2 2 M 4 + 1111
. M 7 − 4.5 M 8 = − p0 h 2
Solving this set of equations, we get
M1 = 0.735 p0 h 2 M 5 = 0.942 p0 h 2
M 2 = 0.998 p0 h 2 M 6 = 1298
. p0 h 2

(CONT.)
(7.13 CONT.)

71
M 3 = 0.960 p0 h 2 M 7 = 1293. p0 h 2
M 4 = 0.589 p0 h 2 M 8 = 0.793 p0 h 2
Then ∇ w = M / D leads to similar set of equations, from which we obtain :
4

w1 = 0.658 p0 h 4 / D w5 = 0.908 p0 h 4 / D
w2 = 0.991 p0 h 4 / D w6 = 1372
. p0 h 4 / D = 0.00536 p0 a 4 / D = wmax
w3 = 0.934 p0 h 4 / D w7 = 1302
. p0 h 4 / D
w4 = 0.589 p0 h 4 / D w8 = 0.713 p0 h 4 / D
where h = a / 4.

SOLUTION (7.14)

With reference to Figs.7.11 and P7.14:


− 2[1 + ( rλ0 ) 2 ] M 2 + [1 + h
2 r0 ] M1 + ( rλ0 ) 2 M 2 = − p0 h 2
− 2[1 + ( rλ0 ) 2 ] M1 + [1 + h
2 r0 ] M 2 + ( rλ0 ) 2 M1 = − p0 h 2
where,
λ = φh = =
a /3
π /6
2a
π , ( r0 ) 2 = 43a , ( r0 )1 = 53a , ( r0 )3 = a
λ
( r0 )1 = 0.382 , ( rλ0 ) 2 = 0.477 , ( 2λr0 )1 = 01
. , ( 2λr0 ) 2 = 0125
.
Thus, − 2[1 + 0.227] M 2 + [1 + 01
. ] M1 + 0.227 M 2 = − p0 h 2
− 2[1 + 0146
. ] M1 + [1 − 0125
. ] M 2 + 0146
. M 1 = − p0 h 2
from which
M1 = 0.8127 p0 h 2 M 2 = 0.8508 p0 h 2
Similarly,
− 2[1227
. ]w2 + [11 . ]w1 + 0.227w2 = −0.8505 p0 h 4 / D
− 2[1146
. ]w1 + [0.875]w2 + 0146. w1 = −0.8127 p0 h 4 / D
Solving, w1 = 0.6693 p0 a / D , w2 = 0.7125 p0 a / D
4 4

SOLUTION (7.15)

Referring to Figs.7.13 and P7.15 :


− 6 M1 + M 2 + M 4 = − 23 h 2 p0
− 6 M 2 + M1 + M 3 + M 4 + M 5 = − 23 h 2 p0
− 6 M 3 + M 2 + M 5 + M 6 = − 23 h 2 p0
− 6 M 4 + M1 + M 2 + M 5 + M 3 = − 23 h 2 p0
− 6 M 5 + M 2 + M 3 + M 6 + M 4 + M 3 + M 2 = − 23 h 2 p0
− 6 M 6 + M 3 + M 5 + M 2 + M1 = − 23 h 2 p0
Solving,
M1 = 0.48861 p0 h 2 M 2 = 0.715477 p0 h 2 M 3 = 0.64568 p0 h 2
M 4 = 0.715477 p0 h 2 M 5 = 0.94232 p0 h 2 M 6 = 0.71548 p0 h 2

(CONT.)
(7.15 CONT.)

72
In a like manner, applying ∇ w = − 23 h
2 2 M
D , we obtain five equations. Solution of these
expressions gives :
w1 = 0.29159 p0 h 4 / D , w2 = 0.50827 p0 h 4 / D , w3 = 0.50187 p0 h 4 / D
w6 = w4 = 0.50830 p0 h 4 / D , w5 = 0.72500 p0 h 4 / D = 0.00285 p0 a 4 / D

SOLUTION (7.16)

With reference to Figs.7.13 and P7.16 :


− 6 M1 + 2 M 2 = − 23 h 2 p0
− 5 M 2 + M1 + M 3 + M 4 = − 23 h 2 p0
− 6 M 3 + 2 M 2 + 2 M 4 + 2 M 5 = − 23 h 2 p0
− 6 M 4 + M 2 + M 3 + M 5 + M 6 = − 23 h 2 p0
− 5 M 5 + M 3 + M 4 + M 6 = − 23 h 2 p0
− 6 M 6 + M 4 + M 5 = − 23 h 2 p0

Solution is given by
M1 = 0.49995 p0 h 2 M 2 = 0.75015 p0 h 2 M 3 = 1000125 . p0 h 2
M 4 = 0.75015 p0 h 2 M 5 = 0.75015 p0 h 2 M 6 = 0.49995 p0 h 2
Using ∇ w = − M / D , we obtain equations of the similar type as given above. From these we
2

determine,
w1 = w6 = 0.48847 p0 h 4 / D w2 = 0.67972 p0 h 4 / D
w4 = w5 = 0.75525 p0 h 4 / D
w3 = 0.86723 p0 h 4 / D = 0.003388 p0 a 4 / D = wmax

SOLUTION (7.17)

From Figs.7.13 and P7.17 :


− 5 M1 + M 2 + M 3 = − 23 h 2 p0
− 6 M 2 + 2 M 3 + 4 M1 = − 23 h 2 p0
− 6 M 3 + M 2 + 2 M1 = − 23 h 2 p0
With the solution,
M1 = 01458
. p0 a 2 M 2 = 0.2033 p0 a 2 M 3 = 01458
. p0 a 2
Applying ∇ w = − M / D we get similar expressions,
2

− 5w1 + w2 + w3 = − 23 hD ( 01458
2
. ) p0 a 2
− 6w2 + 2w3 + 4w1 = − 23 h2
D ( 0.2023) p0 a 2
− 6w3 + w2 + 2w1 = − 23 h2
D ( 01458
. ) p0 a 2
Solution of the above set is
w1 = w3 = 0.02244 p0 a 4 / D w2 = 0.03335 p0 a 4 / D = wmax

SOLUTION (7.18)

73
We write, Fig.7.12 :
x = q1 + q2 cos α + q3 cos β y = q2 sin α + q3 sin β
from which
∂x
∂q1 =1 ∂x
∂q2 = cos α ∂x
∂q3 = cos β
∂y ∂y ∂y
∂q1 =0 ∂q2 = sin α ∂q3 = sin β
Now
∂w ∂w ∂q1 ∂w ∂y ∂w
∂x = ∂q1 ∂x + ∂y ∂q1 = ∂q1
∂w
∂q2 = ∂w
∂x cos α + ∂w
∂y sin α , ∂w
∂q3 = ∂w
∂x cos β + ∂∂wy sin β
Thus,
∂ 2w ∂ 2w
∂q13
= ∂x 2
∂ 2w
= ∂ 2w
cos 2 α + 2 ∂∂x∂wy sin α cos α + ∂ 2w
sin 2 α
2

∂q23 ∂x 2 ∂y 2
and
∂ 2w
= ∂ 2w
cos 2 β + 2 ∂∂x∂wy sin β cos β + ∂ 2w
sin 2 β
2

∂q33 ∂x 2 ∂y 2
These expressions, when written in matrix form, give Eqs.(c) of Sec.7.6.

SOLUTION (7.19)

We have

λ = φh = a /3
π /6 = 2a
π a/3 Fig. (a)
( r0 )1 = , ( r0 ) 2 = , ( r0 )3 = 0
5a
3
4a
3
1
( rλ0 )1 = 0.382 , ( rλ0 ) 2 = 0.477 , ( rλ0 )3 = 0.637 a/3 2
( h
) = 01
2 r0 1 . ,( h
) = 0125
2 r0 2 . ,( h
2 r0 3) = 0167
. 3
1
Referring to Figs.7.11b and Fig. (a), for nodes 1 and 2, 2
we write : 3

a a

− 2[1 + ( rλ0 ) 2 ] M1 + ( rλ0 ) 2 M1 + [1 − h


2 r0 ] M 2 = − p0 h 2
− 2[1 + ( rλ0 ) 2 ] M 2 + ( rλ0 ) 2 M 2 + [1 + h
2 r0 ] M1 + [1 − h
2 r0 ] M 3 = − p0 h 2
or
− 2146
. M1 + 0.875 M 2 = − p0 a 2 / 9 (1)
. M1 − 2.228 M 2 + 0.833 M 3 = − p0 a 2 / 9
11 (2)

Similarly, for deflections, one obtains the expressions


− 2146
. w1 + 0.875w2 = − M 2 a 2 / 9 D (3)
. w1 − 2.228w2 = − M 2 a 2 / 9 D
11 (4)
2w2 = − M 3a 2 / pD (5)

(CONT.)
(7.19 CONT.)

74
Solving,
M1 = 0.0718 p0 a 2 M 2 = 0.0492 p0 a 2 M 3 = −0.0966 p0 a 2
w2 = 0.0054 p0 a 4 / D w1 = 0.006 p0 a 4 / D w3 = 0

SOLUTION (7.20)
( a ) Element stiffness matrices:
w1 θ1 w2 θ2
⎡ 12 360 − 12 360 ⎤ w1
⎢ 360 14,400 − 360 7200 ⎥ θ1
EI ⎢ ⎥
[k ]1 =
216,000 ⎢− 12 − 360 12 − 360 ⎥ w2
⎢ ⎥
⎣ 36 7200 − 360 14,400⎦ θ2

w2 θ2 w3 θ3
⎡ 12 360 − 12 360 ⎤ w2
⎢ ⎥ θ2
EI ⎢ 360 14,400 − 360 7200 ⎥
[k ]2 =
216,000 ⎢− 12 − 360 12 − 360 ⎥ w3
⎢ ⎥
⎣ 360 7200 − 360 14,400⎦ θ3

( b ) System stiffness matrix:


w1 θ1 w2 θ2 w3 θ3
⎡ 12 360 − 12 360 0 0 ⎤ w1
⎢ 360 144000 − 360 ⎥
⎥ θ1
7200 0 0

EI ⎢ − 12 − 360 24 0 − 12 360 ⎥ w2
[K ] = ⎢ ⎥
216,000 ⎢ 360 7200 0 288000 − 360 7200 ⎥ θ2
⎢ 0 0 − 12 − 360 12 − 360 ⎥ w3
⎢ ⎥
⎣ 0 0 360 7200 − 360 − 14000 ⎦ θ3

SOLUTION (7.21)
Fig. (a)
The stiffness matrix is given by Eq.(7.40): m(0,-1)

x
0
[k ]e = [[C ]−1 ]T ( ∫ ∫ [ H ]T [ D][ H ]dxdy )[C ]−1
i(0,1) j(4,1)
y
where,
⎡1 ν 0 ⎤ ⎡ 1 0.3 0 ⎤
⎢ ⎥ ⎢ ⎥
[ D ] = 12(1Et−ν 2 ) ⎢ν 1 ⎥ = λ ⎢0.3 1
3
0 0 ⎥
⎢⎣ 0 0 (1 − ν ) / 2⎥⎦ ⎢⎣ 0 0 0.35⎥⎦
(CONT.)
(7.21 CONT.)

75
and
⎡0 0 0 − 2 0 0 − 6x − 2y 0 ⎤
⎢ ⎥
[ H ] = ⎢0 0 0 0 0 −2 0 − 2x − 6 y⎥
⎢⎣0 0 0 0 − 2 0 0 − 4( x + y ) 0 ⎥⎦
Then,
⎡ 0 0 0 ⎤
⎢ 0 0 0 ⎥
⎢ ⎥
⎢ 0 0 0 ⎥
⎢ ⎥
⎢ −2 0 0 ⎥ ⎡ 1 0.3 0 ⎤
⎢ ⎥
[ H ] [ D][ H ] = ⎢ 0
T
0 − 2 ⎥ λ ⎢0.3 1 0 ⎥[ H ]
⎢ ⎥
⎢ 0 −2 0 ⎥ ⎢⎣ 0 0 0.35⎥⎦
⎢− 6x 0 0 ⎥
⎢ ⎥
⎢− 2 y − 2 x − 4( x + y )⎥
⎢ 0 − 6y ⎥
⎣ 0 ⎦

0 0 0 0 0 0 0 0 0
0 0 0 0 0 0 0 0 0
0 0 0 0 0 0 0 0 0
0 0 0 4 0 1.2 12x 1.2x+4y 3.6y
0 0 0 0 1.4 0 0 2.8(x+y) 0
=λ 0 0 0 1.2 0 4 3.6x 4x+1.2y 12y
2 2
0 0 0 12x 0 3.6x 3.6x 3.6x +12xy 10.8xy
2 2 2 2
0 0 0 1.2x+4y 2.8(x+y) 4x+1.2y 3.6x +12xy 9.6x +13.6xy+9.6y 12xy+3.6y
2 2
0 0 0 3.6y 0 12y 10.8xy 3.6y +12xy 36y

The above matrix will to be integrated term by term over the surface of the plate in the xy plane.
The limits of integration referring to Fig. (a) are as follows : surface is bounded by lines x=0
and x=2y+2 when y varies from -1 to 1. Hence,

∫∫ xdxdy =16/3, ∫∫ ydxdy =4/3, ∫∫ x dxdy =32/3, ∫∫ y dxdy =4/3


2 2

∫∫ xydxdy =8/3, ∫∫ dxdy =4, ∫∫ x dxdy =128/5, ∫∫ x ydxdy =32/5


3 2

∫∫ xy dxdy =32/15, ∫∫ y dxdy =4/5


2 3

It follows that
2 y +2
∫∫ [ H ] [ D][ H ]dxdy = ∫ ∫
1
T
[ H ]T [ D ][ H ]dxdy
−1 0

(CONT.)
(7.21 CONT.)

76
⎡0 0 0 0 0 0 0 0 0 ⎤
⎢0 0 0 0 0 0 0 0 0 ⎥
⎢ ⎥
⎢0 0 0 0 0 0 0 0 0 ⎥
⎢ ⎥
⎢0 0 0 16 0 4.8 64 117333. 4.8 ⎥
= λ ⎢0 0 0 0 5.6 0 0 18.6667 0 ⎥
⎢ ⎥
⎢0 0 0 4.8 0 16 19.2 22.9333 16 ⎥
⎢0 0 0 64 0 19.2 384 704 28.8⎥
⎢ ⎥
⎢0 0 .
0 117333 18.6667 22.9333 70.4 1514667
. 36.8⎥
⎢0 48 ⎥⎦
⎣ 0 0 4.8 0 16 28.8 36.8
−1 −1 T
Upon Multiplying the above with [C ] and [[C ] ] , given in Example 7.9, we obtain [k ]e .

SOLUTION (7.22)
From Eq.(7.43)
θx = ∂∂wy = a3 + a5 x + 2a 6 y + a8 x 2 + 2a9 xy + 3a10 y 2 + a11 x 3 + 3a12 xy 2 (a)
θ y = − ∂∂wx = a2 + 2a4 x + a5 y + 3a 7 x 2 + 2a8 xy + a9 y 2 + 3a11 x 2 y + a12 y 3 (b)
Expressions (7.43), (a), and (b) are combined to yield
⎧a1 ⎫
⎧w⎫ ⎪ ⎪
⎡1 x y x 2
xy y x2 3 2
x y xy y2 3
x y xy ⎤ ⎪M ⎪
3 3

⎪ ⎪ ⎢ ⎥⎪ ⎪
⎨θx ⎬ = 2 xy ⎥ ⎨M ⎬
2
⎢0 0 1 0 x 2y 0 x 2 xy 2 y 2 x3 (c)
⎪θ ⎪ ⎢⎣0 1 0 2 x y 0 3x 2 2 xy y 2 0 3x 2 y y 2 ⎥⎦ ⎪M ⎪
⎩ y⎭ ⎪ ⎪
⎪⎩a12 ⎪⎭
Equations (c) are evaluated at each nodal point employing the given coordinates. Placing the
origin of the coordinates xyz at nodal point m (Fig.7.22), we thus obtain a system of twelve
equations.
These expressions may be written in the following matrix form :
⎧ wi ⎫ ⎡1 0 0 0 0 0 0 0 0 0 0 0 ⎤ ⎧ a1 ⎫
⎪θ ⎪ ⎢
⎪ xi ⎪ ⎢0 0 1 0 0 0 0 0 0 0 0 0 ⎥⎪ a2 ⎪
⎥⎪ ⎪
⎪ θ yi ⎪ ⎢0 1 0 0 0 0 0 0 0 0 0 0 ⎥ ⎪ a3 ⎪
⎪ ⎪ ⎢ ⎥⎪ ⎪
⎪ w j ⎪ ⎢1 b 0 b2 0 0 b3 0 0 0 0 0 ⎥⎪ a4 ⎪
⎪ θxj ⎪ ⎢0 0 1 0 b 0 0 b2 0 0 b3 0 ⎥ ⎪ a5 ⎪
⎪ ⎪ ⎢ ⎥⎪ ⎪
⎪ θ yj ⎪ ⎢0 1 0 2b 0 0 3b 2 0 0 0 0 0 ⎥⎪ a6 ⎪
⎨ ⎬=⎢ ⎨ ⎬
a 3b ⎥ ⎪ a 7 ⎪
(d)
⎪ wm ⎪ ⎢1 b a b2 ab a2 b 3
ab 2 a 2b a3 ab 3

⎪θxm ⎪ ⎢0 0 1 0 b 2b 0 b2 2ab 3a 3 b3 2ab ⎥ ⎪ a8 ⎪
⎪ ⎪ ⎢ ⎪ ⎪
⎪θ ym ⎪ ⎢0 1 0 2b a 0 3b 2 2ab b2 0 3ab 2 a 2 ⎥ ⎪ a9 ⎪

⎪ wn ⎪ ⎢1 0 a 0 0 a2 0 0 0 a3 0 0 ⎥ ⎪a10 ⎪
⎪ ⎪ ⎢ ⎪ ⎪
⎪θxn ⎪ ⎢0 0 1 0 0 2a 0 0 0 3a 2
0 0 ⎥⎥ ⎪a11 ⎪
⎪θ yn ⎪ ⎢0 2
a 2 ⎥⎦ ⎪⎩a12 ⎪⎭
⎩ ⎭ ⎣ 1 0 0 a 0 0 0 a 0 0

77
SOLUTION (7.23)

Proceeding as in Example 7.10, now we have

m n
m θx12
1 n
1
θ y5 p0
4
a=2
i j i
j 1
substitute plate w1
nodal force
and displacements
1

a=2

actual plate

Hence,
{δ } = {w1 , 0, 0, 0, θ y 5 , 0, 0, 0, 0, 0, 0, θx12 }
{Q} = { p 0 / 4, 0, 0, 0, 0, 0, 0, 0, 0, 0, 0, 0}
Thus, from Eqs.(7.47), (7.27), and Table 7.3
1 5 12
⎧ p0 / 4 ⎫ ⎡158.4 321 . − 321. ⎤ ⎧ w1 ⎫
⎪ ⎪ ⎢ ⎥⎪ ⎪
⎨ 0 ⎬=
Et 3
163.8 ⎢ 321. 22.8 0 ⎥ ⎨θ y 5 ⎬ (a)
⎪ 0 ⎪ ⎢⎣− 321 22.8 ⎥⎦ ⎪⎩θx12 ⎪⎭
⎩ ⎭ . 0
From which
w1 = wmax = 0.6021 p0 / Et 3
The “exact” solution is, from Table 5.1 :
w1 = wmax = 0.00406 p0 a 4 / D = 0.70936 p0 / Et 3
( 0.70936− 0.6021)
Error is thus : 0.70936 × 100 = 151
. %

SOLUTION (7.24)

We now have
{Q} = {P 4 , 0, 0, 0, 0, 0, 0, 0, 0, 0, 0, 0}
Referring the solution of Prob. 7.23, we obtain
wmax = 0.6021P / Et 3
where ( wmax ) exact = 0.5063P / Et
3

Error: 0.6021 − 0.5063


0.6021 × 100 = 159
. %

78
CHAPTER 8

SOLUTION (8.1)

With reference to Case B of Table 8.1:


200×109 ( 0.02 )3
H = Dx = 12(1Et−ν 2 ) = = 146.52 kN ⋅ m
3

12 ( 0.91)

200×109 (12×10 −7 )
D y = 12(1Et−ν 2 ) + = 146.52 + = 1346.52 kN ⋅ m
3
E' I
s 0. 2

SOLUTION (8.2)
We obtain location of centroid ( y = ∑ i i
Ay
∑ Ai ), measured from the top surface, as
y = 7.5 mm.
Then,
100 (10)3 10 ( 20 )3
I N . A. = 12 + 12 + 1000( 2.5) 2 + 200(12.5) 2
= 52500 mm4 = 52.5 × 10 −9 m4

Referring to Table 8.1:


210×103 (100)(10)3
Dx = 12[100 −10+10 (10/ 30 )2 ]
= 19365
. × 10 7 N ⋅ mm = 19.365 kN ⋅ m
210×103 ( 52500 )
Dy = 100 = 11025
. × 10 7 N ⋅ mm = 110.25 kN ⋅ m
210×(10)3
G xy = 24 (1.3 ) = 6.731 × 10 6 N ⋅ mm = 6.731 kN ⋅ m
12 ( 6.731×106 )
G= (10)3
= 8.077 × 10 4 N / mm2 = 80.77 GPa
C = 0.246(10)3 (30 − 10)8.077 × 10 4 = 3974
. × 108 N ⋅ mm2 = 397.4 N ⋅ m2
H = 2( 6.731 × 10 6 ) + 3.974 ×10
= 17.436 × 10 6 N ⋅ mm = 17.436 kN ⋅ m
8

100

SOLUTION (8.3)

Case A, Table 8.1:


Dx = 1−νcc2 [ I cx + ( Ecs − 1) I sx ] = 30×109
[30 × 10 −4 + ( 210 ×10
− 1)2 × 10 −4 ]
E E 9

(1− 0.152 ) 30×109

= 12.89(10 4 ) kN ⋅ m

D y = 1−νcc2 [ I cy + ( Ecs − 1) I sy ] = 30×109


[20 × 10 −4 + ( 210 ×10
− 1)1 × 10 −4 ]
E E 9

(1− 0.152 ) 30×109

= 7.9795(10 4 ) kN ⋅ m

1−ν c
G xy = 2 Dx D y = 1− 0.15
2 (10 4 ) (12.89)( 7.9795) = 4.31(10 4 ) kN ⋅ m
H = Dx Dy = (104 ) (12.89)(7.9795) = 10.142(104 ) kN ⋅ m
Dxy = νc Dx D y = 015
. (10 4 ) (12.89)( 7.9795) = 1521
. (10 4 ) kN ⋅ m

79
SOLUTION (8.4)

( a ) From Eq. (5.10), we have


mπx1 nπy1
pmn = 4P
ab sin a sin b
m+ n
−1
Using b = 2a, x1 = a / 2 and y1 = a: pmn = 4P
2a2
( −1) 2

Expression (5.1b) together with Eq. (8.14) and pmn obtained above yields
∞ ∞ m + n −1

w = ∑ ∑ 24aP2 ( −1 ) 2 nπy
m 4π 4 2 2 4 4π 4 sin maπx sin b (1)
Dx + 2 Hm2 n2 π + n Dy
m n a4 a b b4

Here,
H = Dx D y , a1 = a 4 D / Dx , b1 = b4 D / D y
It follows from Eq.(1) that
∞ ∞ m + n −1

w= 2P
π a D
4 2 ∑∑
m n
( −1 ) 2
[( m/ a1 )2 +( n / b1 )2 ]2
sin mπx
a sin
nπy
b (2)

( b ) The moment is given by, from Eqs.(8.5) and (2):


∞ ∞

∑ ∑ ( −1)
m+ n
−1 sin( mπx )sin( nπ / b )
Mx = 2P
π 4a 2 D
2
[( m/ a1 )2 +( n / b1 )2 ]2
[( maπ ) 2 Dx + 015
. ( nbπ )2 Dx D y ] (3)
m n

Numerical values of constants, referring to Table 8.1:


2 a ( 0.2 )3
I cx = 12 = 0.00133a, I cy = 0.5 I cx = 0.000667a
Dx = 21.4×10
( 0.00133a + 0.00555a ) = 1505 × 108 a
9

0.978 .
Dy = 21.4×109
0.978 ( 0.000667a + 0.00278a ) = 7.55 × 10 7 a
Dy
[(1 / a1 ) 2 + (1 / b1 ) 2 ]2 = [ a 2 + ]2 =
Dx 2.083×108
D 4a2 D a4D

Deflection and moment, from Eqs.(2) and (3), are therefore for m=n=1:
w = 9.6 × 10 −6 aP / π 4 , . P / π2
M x = 148

SOLUTION (8.5)

General expression defining the deflection (see: solution of Prob. 8.4), taking the first two terms
(n=1, m=1,3), a=b, and x=y=a/2 reduces to

Dx + 2 H + D y + 81 Dx +18 H + D y
w= 4 Pa 2
π4 ( Dx + 2 H + D y )( 81 Dx +18 H + D y ) = wmax

In case of reinforced concrete, from solution of Prob.8.4:


Dx = 1505
. × 108 a, D y = 7.55 × 10 7 a , H = Dx D y = 10.66 × 10 7 a
We thus have,
wmax = 4 Pa 1462.4
π 4 107 62 , 296.64
= 96.392(10 −12 ) Pa

SOLUTION (8.6)

80
For m=n=1 and x=y=a/2, Eq.(8.16) becomes
w=
16 p0 1
π6 ( Dx + 2 H + D y )/ a 4

Introducing the values of rigidities from Prob.8.1:


w = π 6 0 1.78608 = 9.31794 × 10 −9 p0 a 4
16 p a 4

×106

But wmax = 0.001 m and a=0.5 m


Thus p0 = 172
. MPa

SOLUTION (8.7)

Equation (8.16) for a=b, x=y=a/2, and m=n=1 yields:


wmax =
16 p0 a4
π6 ( Dx + 2 H + D y )

substituting the properties found in solution of Prob.8.2, We obtain


16 p0a 4
wmax = π6
1
19365+ 2 (17436)+110250

or wmax = 9.73 × 10 −5 p0 a 4 / π 6 = 1012


. (10 −7 ) p0 a 4

SOLUTION (8.8)

We have from Eq.(8.23),


p0a 4b4
wmax = 24 b4 Dx +16 a 2b2 H + 24 a 4 D y
(a)
where, with reference to Table 8.1:
200×109 ( 0.02 )3
Dx = H = 12 (1− 0.32 )
= 1465
. × 105
200×109 (12×10 −7 )
D y = 146
. × 10 5 + 0 .2 = 13465
. × 10 6
Thus, using a=2b=4, Eq. (a) results in
wmax = 4.831 × 10 −7 p0

SOLUTION (8.9)

Expression (8.24), together with D1 = (3Dx + 2 H + 3D y ) / 3 and the numerical values found
in solution of Prob.8.8, yields at r=0:
p0a 4 p0 ( 4 )4
wmax = 64 D1 = 64 ( 5.965×105 )
= 6.706 × 10 − 6 p0

SOLUTION (8.10)

Substituting Eq.(8.22) into (8.5), we have


2 y2 Dx 2 D xy 2 y2
p0a 4b4 (1− x 2 − )( − 2 x4 + − )
Mx = a b2 a2
4 ( 3a Dx + 2 a b H +3b D y )
4 2 2
a
4
b2 b4

2 y2 Dy 2 y2 D xy 2 y2
p0a b (1− x 2 −
4 4
)( − + − )
My = a b2 b2
4 ( 3a Dx + 2 a b H +3b4 D y )
4 2 2
b4 a2 a4

2 G p a 2b2 xy
M xy = − 3a 4 Dx + 2xya 20b2 H +3b4 D y

SOLUTION (8.11)

81
See: Solution of Prob. 8.2.
Dx = 19.365 kN ⋅ m Dy = 110.25 kN ⋅ m H = 17.436 kN ⋅ m
Using Eq. (c) of Sec. 8.6, we have
0.003418 P0 a 4 4
wmax = Dx + 0.5714 H ( ba ) 2 + Dy ( ba )2
= 19.3650.003418(100)(1.667)
+110.25( 1 )2 +17.436( 1 )2
1.2 1.2

= 24.4 mm

SOLUTION (8.12)

Refer to Solution of Prob. 8.1. We have


Dx = H = 146.52 kN ⋅ m Dy = 1346.52 kN ⋅ m

The largest deflection, using Eq. (e) of Sec. 8.6:


0.000429 P0 a 4 0.000429(250×10 )(0.5)3 4
wmax = Dx + H ( ba )2 + Dy ( ba )2
= 146.52 +146.52( 1 )2 +1346.52( 1 )2
2 2

= 12.9 mm

SOLUTION (8.13)

From Solution of Prob. 8.3:


Dx = 12.89(104 ) kN ⋅ m Dy = 7.9795(104 ) kN ⋅ m
H = Dx Dy = 10.142(104 ) kN ⋅ m

Thus, Eq. (e) of Sec. 8.6 gives


0.000429 P0 a 4 ×10 )(0.5)
3 4
wmax = Dx + H ( ba )2 + Dy ( ba )2
= [12.890.000429(500
+10.142( 1 ) 2 + 7.9795( 1 )2 ]104
2 2

= 0.08 mm

SOLUTION (8.14)

Let wxx = ∂ 2 w / ∂x 2 , w yy = ∂ 2 w / ∂x 2 , wxy = ∂ 2 w / ∂x∂y.

Strain energy given by Eq.(3.42), upon introducing Eqs.(3.3), (8.4), and ε z = γ xy = γ yz = 0,


becomes then :
t

∫ ∫∫
b a
U= [ z 2 wxx2 E x + z 2 wxx w yy E xy + z 2 w 2yy E y + z 2 wxx w yy E xy
2
1
2
−t 2 0 0

+ 4 z 2 wxy2 G ]dxdydz
or

∫∫
b a Ex t 3 E yt3 E xy t 3
U= 1
2 [wxx2 12 + w 2yy 12 + 2 wxx w yy 12 + 4wxy2 Gt 3
12 ]dxdy
0 0

This expression, based on the notation given by Eqs.(8.6), leads to Eq.(8.26).

SOLUTION (8.15)

82
Strain energy is given by Eq.(8.26):

∫∫
b a
U= 1
2 [ Dx wxx2 + 2 Dxy wxx w yy + D y w 2yy + 4G xy wxy2 ]dxdy (a)
0 0
Assume, by setting 2mπ / a = α m and 2nπ / b = βn ,
∞ ∞
w = ∑ ∑ a mn (1 − cos α m x )(1 − cos βn y ) (b)
m n
which satisfies the boundary conditions (Fig.5.19):
w = 0, ∂w / ∂x = 0 (x=0, x=a); w = 0, ∂w / ∂y = 0 (y=0, y=b)
Work done by p0 equals
∞ ∞
W=∫ ∫ p0 wdxdy = p0 ab∑ ∑ a mn
b a

0 0
m n
We have, from Eq.(b):
∞ ∞
wxx = ∑ ∑ a mnα m2 cos α m x(1 − cos βn y )
m n
∞ ∞
w yy = ∑ ∑ a mn βn2 cos βn x (1 − cos α m x ) (c)
m n
∞ ∞
wxy = ∑ ∑ a mnα m βn sin α m x sin βn y
m n
Taking m=n=1, Eq.(a) becomes then
U = ab ⋅ a112 [ 43 Dxα14 + 43 D y β14 + 12 Dxyα12 β12 + Gxyα12 β12 ]

We have Π = U − W. It follows, from ∂Π / ∂a11 = 0 , that


a11 =
2 p0
3( Dxα14 + D y β14 )+ 2 Dxyα12 β12 + 4 Gxyα12 β12

where α1 = 2π
a , β1 = 2π
b
For square plate (a=b), deflection (b) is therefore
a 4 p0 [1− cos( 2πx / a )][1− cos( 2πy / b )]
w= 8π 4 [3( Dx + D y )+ 2 Dxy + 4 Gxy ]

Hence,
p0a 4
wmax = 2π 4 [3( Dx + D y )+ 2 ( Dxy + 2 Gxy )]

SOLUTION (8.16)

We have
[ D] = t3
12 [T ]T [ D * ' ][T ] = t3
12 [D* ]
where
⎡ 0.75 0.25 0.43 ⎤ ⎡0.75 0.25 − 0.86⎤
⎢ ⎥ ⎢ ⎥
[T ] = ⎢ 0.25 0.75 − 0.43⎥ , [T ]T = ⎢0.25 0.75 0.86 ⎥
⎢⎣− 0.86 0.86 0.5 ⎥⎦ ⎢⎣0.43 − 0.43 0.5 ⎥⎦

(CONT.)
(8.16 CONT.)

83
⎡ 1−130..602 0.1(13.6 )
1− 0.02 0 ⎤ ⎡1388. 139
. 0 ⎤
⎢ 0.2( 6.8 ) ⎥ ⎢ ⎥
[ D * ' ] = ⎢ 1− 0.02 6.8
1− 0.02 0 ⎥= ⎢ 139
. 6.94 0 ⎥
⎢ 0 . ⎥⎦ ⎢⎣ 0 . ⎥⎦
⎣ 0 618 0 618

Thus,
⎡0.75 0.25 − 0.86⎤ ⎡1388 . .
139 0 ⎤ ⎡ 0.75 0.25 0.43 ⎤
109 ( 0.005 )3 ⎢ ⎥⎢ ⎥⎢ ⎥
[ D] = 12 ⎢0.25 0.75 0.86 ⎥ ⎢ 139
. 6.94 0 ⎥ ⎢ 0.25 0.75 − 0.43⎥
⎢⎣0.43 − 0.43 0.5 ⎥⎦ ⎢⎣ 0 . ⎥⎦ ⎢⎣− 0.86 0.86
0 618 0.5 ⎥⎦

⎡ 4171
. − 1948
. 0.776⎤
⎢ ⎥
= 10.417 ⎢− 0.202 9.850 2.207⎥
⎢⎣ 0.775 2.214 4.877⎥⎦

SOLUTION (8.17)

We have n = 2, t1 = t
2 , t 2 = 3t 10 , t3 = 1
10 , E1 = E3 , ν1 = ν3 . Equation(8.36) gives
E t3
Dt = 23 [ 1−ν12 ( t − t ) + 1−ν22 ( t − t33 ) + 1−3ν332 ]
E1 3 3 E2 3
1 2 2

E t3
= 23 [1−ν112 ( t8 − ) + 1−ν122 ( 1000 − 1000 ) + (1−ν121)1000 ]
3
E 27 t 3 E 3
27 t t 3

100

= t3
( 1−ν121 +
99 E 26 E2
1500 1−ν 22
) Q.E.D.

SOLUTION (8.18)

Letting α = 0 o , α = 45o , and α = −45o , *


Eq.(8.53), together with [ D ] yield
⎡30 1 0⎤
⎢ ⎥
[ D ]2 = [ D ]α = 0o
* *
= ⎢ 1 3 0⎥
⎢⎣ 0 0 1⎥⎦
and
⎡9.75 7.75 6.75⎤
⎢ ⎥
[ D ]1 = [ D ]3 = [ D ]α = 45o
* * *
= ⎢7.75 9.75 6.75⎥
⎢⎣6.75 6.75 7.75⎥⎦

Equation (8.56) with t 0 = −6 mm, t1 = −2 mm, t 2 = 2 mm, t3 = 6 mm (Fig.8.12) gives:


3
Dij = 1
3 ∑ (D
k =1
*
ij k ) ( t k3 − t k3−1 )

= ( Dij* )1[( −2)3 − ( −6 )3 ] + 13 ( Dij* )2 [( 2)3 − ( −2)3 ] + 13 ( Dij* )3 [( 6)3 − ( 2)3 ]


1
3

= 16
3 [26( Dij* ) 45o + ( Dij* ) 0o ]

(CONT.)
(8.18 CONT.)

84
⎡1512 1080 936 ⎤
⎢ ⎥
= ⎢1080 1368 936 ⎥ N ⋅ m
⎢⎣ 936 936 1080⎥⎦

Equation (8.55) is thus


⎧25,000⎫ ⎡1512 1080 936 ⎤ ⎧ ∂ w ∂x 2 ⎫
2

⎪ ⎪ 3⎢ ⎥⎪ 2 ⎪
⎨ 0 ⎬ = −10 ⎢1080 1368 936 ⎥ ⎨ ∂ w ∂y 2 ⎬
⎪ 0 ⎪ ⎢⎣ 936 936 1080⎥⎦ ⎪⎩∂ 2w ∂x∂y ⎪⎭
⎩ ⎭
Solving,
⎧ ∂ w ∂x 2 ⎫ ⎧− 43.7⎫
2

⎪ ∂ 2w ⎪ ⎪ ⎪
⎨ ∂y 2 ⎬ = ⎨ 211 . ⎬10 −3 mm−1
⎪∂ 2w ⎪ ⎪ 19.6 ⎪
⎩ ∂x∂y ⎭ ⎩ ⎭

The strains in the laminate, from Eqs.(3.3b), are:


⎧ ε x ⎫ ⎧ 48.3z ⎫
⎪ ⎪ ⎪ ⎪
⎨ ε y ⎬ = ⎨ − 211
. z ⎬10 −3
⎪γ ⎪ ⎪− 19.6 z ⎪
⎩ xy ⎭ ⎩ ⎭

Stress in the laminate 1 and 3 can be calculated by inserting into the foregoing the
appropriate values of z: − 6 ≤ z ≤ −2 or 2 ≤ z ≤ 6. From Eq.(8.51), we have
⎧σ x ⎫ ⎡9.75 7.75 6.75⎤ ⎧ 438. z ⎫
⎪ ⎪ ⎢ ⎥⎪ ⎪
⎨σ y ⎬ = ⎢7.75 9.75 6.75⎥ ⎨ − 211
. z ⎬ MPa
⎪τ ⎪ ⎢ ⎥⎪ ⎪
⎩ xy ⎭α = 45o ⎣6.75 6.75 7.75⎦ ⎩− 19.6 z ⎭
Stress in the 2nd laminate can be calculated by introducing appropriate value of
z ( −2 ≤ z ≤ 2):
⎧σ x ⎫ ⎡30 1 0⎤ ⎧ 438. z ⎫
⎪ ⎪ ⎢ ⎥⎪ ⎪
⎨σ y ⎬ = ⎢ 1 3 0⎥ ⎨ − 211
. z ⎬ MPa
⎪τ ⎪ ⎢ ⎥⎪ ⎪
⎩ xy ⎭α = 0o ⎣ 0 0 1⎦ ⎩− 19.6 z ⎭

SOLUTION (8.19)

We have
t1 = −3 1
3 2
∑ (D
x
Dij = 1
3
*
) ( t k3 − t k3−1 )
ij k
t2 = 2 3
k =1
10 −9 −9 z
= 3 ( Dij* )1[( −3)3 − ( −6)3 ] + 103 ( Dij* )2 [( 2)3 − ( −3)3 ]
−9
+ 103 ( Dij* )3 [( 6)3 − ( 2)3 ]

(CONT.)
(8.19 CONT.)

85
Substituting Eqs.(a) and (e) of Sec.8.10 into this:
⎡10.437 0.437 2.812⎤ ⎡15 15 . 0⎤
189 ⎢ ⎥ 35 ⎢ ⎥
[ D] = 0.437 10.437 2.812⎥ + 15
. 3.75 0⎥
3 ⎢ 3 ⎢
⎢⎣ 2.812 2.812 3397
. ⎥⎦ ⎢⎣ 0 0 5⎥⎦
⎡ 10.417 0.433 − 2.812⎤
208 ⎢ ⎥
+ 0.437 10.437 − 2.812⎥
3 ⎢
⎢⎣− 2.812 − 2.812 3937
. ⎥⎦

After multiplying each term by the numbers shown and adding resulting matrices, we have
⎡1556163. 75.330 − 17.809⎤
⎢ ⎥
[ D ] = ⎢ 75.330 1424.613 − 17.809⎥ N ⋅ m
⎢⎣ − 17.809 − 17.809 526.329 ⎥⎦

SOLUTION (8.20)
⎡10.437 0.437 2.812⎤ ⎡1502.928 62.928 404.928⎤
⎢ ⎥ ⎢ ⎥
(a) [ D ] = 144 ⎢ 0.437 10.437 2.812⎥ = ⎢ 62.928 1502.928 404.928⎥
⎢⎣ 2.812 2.812 3.737 ⎥⎦ ⎢⎣ 404.928 404.928 566.928⎥⎦

(b) D11* = 15 cos 4 30 o + 3.75 sin 4 30 o + 2(15


. + 2 × 5) sin 2 30 o cos 2 30 o = 12.984
*
D22 = 15 sin 4 30 o + 3.75 cos 4 30 o + 2(15
. + 2 × 5) sin 2 30 o cos 2 30 o = 7.359
D12* = (15 + 3.75 − 4 × 5) sin 2 30 o cos 2 30 o + 15
. (sin 4 30 o + cos 4 30 o ) = 0.703
*
D33 = (15 − 15. − 2 × 5)sin 2 30 o cos 2 30 o + 5(sin 4 30 o + cos 4 30 o ) = 4.203
D13* = (15 − 15
. − 2 × 5) sin 30 o cos 3 30 o − (375
. − 15 . − 2 × 5) cos 3 30 o cos 4 30 o
= 1975
.
D23 = (15 − 15
*
. − 2 × 5)sin 3 30 o cos 30 o − (175
. − 15 . − 2 × 5) sin 30 o cos 3 30 o
= 2.895
Thus,
⎡12.984 0.703 1975
. ⎤ ⎡1869.693 101232
. 284.400⎤
⎢ ⎥ ⎢ ⎥
[ D ] = 144 ⎢ 0.703 7.359 2.895⎥ = .
⎢ 101232 1059.696 416.880⎥ N ⋅ m
⎢⎣ 1975
. 2.895 4.203⎥⎦ ⎢⎣ 284.400 416.880 605.232⎥⎦

86
CHAPTER 9

SOLUTION (9.1)

From Table B.3, we have: E = 70 GPa. Hence,


70(109 )(0.01)
D = 12(1Et−ν 2 ) = = 6.41 kN ⋅ m
3

12(0.91)

Equation (9.4) becomes at x = a 2 and y = b 2 :


wmax = =
16 P0 1
π 6D ( 1 + 4 )2 + N ( 1 ) 2
D πa
a2 a2

×10 ) 3
16(40×103 )
= π16(40
6
(6.41×103 ) 5 )2 + 20×103 ( 1 ) 2
( 0.5
6.41×103 π ( 0.5)

= (0.104) 100+11.265 = 1.03 mm

SOLUTIONS (9.2)

Flexural rigidity is
200(109 )(4×10−3 )3
D = 12(1Et−ν 2 ) = = 1.1645 kN ⋅ m
3

12(0.916)

Equation (9.10):
4π 2 (1,164.5)
N cr = 4π 2 D
b2
= (0.2)2
= 1.149 kN
Equation (9.11):
σ cr = 3(1π−νE ) ( bt ) 2 = π 3(0.916)
(200×10 )
2 2 9
4 2
2 ( 200 )
= 287 MPa

SOLUTION (9.3)

We have N x = N y = N , and N xy = 0 . Thus, Eq.(d) in Sec. 9.2 becomes


∞ ∞
∂ 4w
∂x 4
+2 ∂ 4w
∂x 2∂y 2
+ ∂ 4w
∂y 4
− N
D ( ∂ 2w
∂x 2
+ ∂ 2w
∂y 2
)=
16 p0
π 2D ∑∑
m n
1
mn sin maπx sin
nπy
b

Upon following a procedure similar to that described in Example 9.1, we obtain


a mn =
16 p0
2 2 (a)
π 6 Dmn{( m2 + n 2 )2 + ND [( πma )2 +( πnb )2 ]}
a b

Deflection is expressed by
∞ ∞
w = ∑ ∑ a mn sin maπx sin
nπy
b (b)
m n
where a mn is given by Eq.(a). Using a=b, x=y=a/2 :
∞ ∞ m + n −1

wmax = ∑∑ ( −1 )
16 p0 2

π 6 Dmn 2 2 (m,n=1,3,…)
mn{( m2 + n 2 )2 + ND [( πma )2 + ( πnb )2 ]}
m n a b

Equation (3.7) together with Eq.(b) leads to


∞ ∞
σ x = − 1−Ezν 2 ∑∑a
m n
mn [( maπ ) 2 + ν ( nbπ ) 2 ]sin maπx sin
nπy
b

(CONT.)

87
(9.3 CONT.)

Letting z=t/2, x=y=a/2, and a=b :


∞ ∞

∑∑a
m+ n
−1
σ x ,max = − Et
2 (1−ν 2 ) mn ( −1) 2
[( maπ ) 2 + ν ( nbπ ) 2 ]
m n
For m=n=1,
a11 = wmax = a11
16 p0
π 6 D [( ) + D ( πa ) ]
2 2 N 2 2 ,
a 2

and
σ max = Et
2 (1−ν 2 )
a11 ( πa ) 2 (1 + ν )

SOLUTION (9.4)
p0
Substitution of
p( x , y ) = p 0 x
a
x
z
into Eq.(5.3) yields the x
parameter pmn as follows a
N b N

∫∫
b a nπy
pmn = x sin maπx sin
4 p0
a 2b b dxdy y
0 0
nπy b
= [( maπ ) 2 sin maπx − cos maπx ]0a ⋅ [− nbπ cos
4 p0 ax
a 2b mπ b 0 ]
= [( maπ ) 2 sin mπ − cos mπ ][− nbπ cos nπ +
4 p0 2
a b
a 2b mπ nπ ]
= ( m1π sin mπ − cos mπ )(1 − cos nπ )
4 p0
mnπ 2
Then,
pmn = pmn = 0
8 p0
mnπ 2
(m,n=1,3,….), (m,n=2,4,...)

The lateral load is represented by


∞ ∞
p( x , y ) =
8 p0
π2 ∑∑ m n
1
mn sin maπx sin
nπy
b (m,n=1,3,…)

Inserting this into Eq. (9.3), we then obtain


∞ ∞
∂ 4w
∂x 4
+2 ∂ 4w
∂x 2∂y 2
+ ∂ 4w
∂y 4
− N ∂ 2w
D ∂x 2 = Dπ 2
8 p0
∑∑
m n
1
mn sin maπx sin
nπy
b (a)

Boundary conditions are satisfied by taking w in the form of Eq.(5.1b). Introducing Eq.(5.1b) into
Eq.(a) :
∞ ∞

∑ ∑ {a [π 4 ( ma2 + ) + ( maπ ) 2 ] − }sin maπx sin


nπy
=0
2
n2 2 N 8 p0
mn b2 D Dπ 2 mn b
m n
from which
a mn =
8 p0
2 2
Dπ 6
mn [( m2 + n 2 )2 + ND ( πma )2 ]
a b

Deflection is thus
∞ ∞
w( x , y ) =
8 p0
Dπ 6 mn ∑∑ m n a
2

b
1
2
mn [( m2 + n 2 )2 + ND ( πma )2 ]
sin maπx sin
nπy
b

88
SOLUTION (9.5)

Take the deflection w of the form given by Eq.(5.1b).


Let N x = N y = − N , N xy = 0, and p = 0 in Eq. (9.3) :
D∇ 4 w = N ( ∂∂xw2 + ∂∂yw2 ) = 0
2 2

Thus for a=b:


∞ ∞

∑ ∑ [ Dπ ( ma2 + ) − Nπ 2
( m2 + n 2 )]a mn sin maπx sin
nπy
=0
2
4 n2 2
b2 a2 b
m n
The above yields
Dπ 2
N cr = a2
( m2 + n 2 )
For m=n=1, we obtain : N cr = 2π 2 D / a 2 , σ cr = 2π 2 D / a 2 t

SOLUTION (9.6) x
Ny
N xy
Expression (9.1) and
(9.2) now appear y
∂N x ∂N xy Fx
∂x + ∂y + Fx = 0
∂N xy ∂N y
(a) Nx Nx + ⋅⋅⋅
∂x + ∂y + Fy = 0 Fy
N xy + ⋅ ⋅ ⋅
Upon following a procedure
identical to that described in Sec. 9.2, N y + ⋅⋅⋅
and using Eqs.(a) instead of Eqs.(9.1)
and (9.2), we readily obtain Eq. (P9.6).

SOLUTION (9.7)

Flexural rigidity equals


210(109 )(6×10−3 )3
D = 12(1Et−ν 2 ) = = 4.1266 kN ⋅ m
3

12(0.916)

and
r = ba = 150
300
=2
Equation (9.19):
9π 2 (1+ r )
2
Scr = 32 b 2
r 3

9π 2 (4.1266) 1+ 4
= 32(0.15)2 23
= 1.591 kN

SOLUTION (9.8)

We have m = n = 1, a = b, N x = N , N y = 0, and N xy = 0. The deflection that satisfies


the boundary condition is
πy
w = a11 sin πax sin b (a)
Work done by N equals the strain energy due to stretching of the midplane of the plate:

∫∫
a a
Us = 1
2 [ N ( ∂∂wx ) 2 ]dxdy = π 2N
8 a112
0 0
(CONT.)

89
(9.8 CONT.)

Work done by P:
W = Pw x = y = a2 = Pa11
Equation (5.69) becomes
∞ ∞
Ub = π 4 ab
D ∑ ∑ a mn ( ma2 + ) = π 4D
2
2 n2 2
8 b2 2a 2
a112
m n
π 4D π 2N
Hence Π = Ub + Us − W = 2a 2
a112 + 8 a112 − Pa11
and
∂Π
∂a11 = 0: a11 = π 4D
P
2 (b)
a + π 4N

Substitute Eq.(b) into (a) to obtain w.

SOLUTION (9.9)

Assume the deflection surface of the form


∞ ∞
w = ∑ ∑ a mn sin maπx sin
nπy
b (a)
m n
Expression (9.14), for the case under consideration, reduces to
W = 12 N ∫ ∫
a a
[( ∂∂wx ) 2 + ( ∂∂wy ) 2 ]dxdy
0 0
Introducing Eq.(a) into the above,
∞ ∞
W = 12 N ∑ ∑ a mn ∫ ∫
a a nπy nπy
2
[( maπ ) 2 cos 2 mπx
a sin 2 a + ( naπ ) 2 sin 2 mπx
a cos 2 a ]dxdy
0 0
m n
∞ ∞ ∞ ∞
= N ∑ ∑ a mn
2
[( maπ ) 2 a2
8 + ( naπ ) 2 a2
8 ] = N ∑ ∑ a mn
2 π2
8 ( m2 + n 2 )
m n m n
(m,n=1,2,3...)

Strain energy is given by Eq.(5.69):


∞ ∞
U= Dπ 4
8a2 ∑∑am n
2
mn ( m2 + n 2 ) 2

Potential energy is thus, Π = U − W. Hence ∂Π / ∂a mn = 0 gives


D ( m2 + n 2 )π 2
a mn [ a 2 − N] = 0
Since a mn ≠ 0, we thus have
π 2D
N cr = a2
( m2 + n 2 )
Taking, for critical load, m=n=1: N cr = 2π 2 D / a 2

SOLUTION (9.10)

We have N x = N , N y = 0, and N xy = 0.
Choosing a deflection w, given in the from of Eq.(5.1b), the boundary conditions are satisfied.
∞ ∞

∫∫ N ∑ ∑ a mn
a a
∂w 2 m2π 2
Then, W= 1
2 N ( ) dxdy =∂x
ab
8
2
a2
0 0
m n
(CONT.)

90
(9.10 CONT.)

The strain energy U is defined by Eq.(5.69). It follows, from ∂Π / ∂a mn = 0, that


∞ ∞ ∞ ∞
Nab∑ ∑ a m2π 2
− π 2 ab
D ∑ ∑ a mn ( ma2 + ) =0
2
1 n2 2
2 mn a 2 2 b2
m n m n
or (for a mn ≠ 0 ) :
π 2a 2 D
N cr = ( ma2 +
2
n2 2
m2 b2
)
4π 2 D
This is the same result given by Eq.(9.8). Thus, from Eq.(9.10), N cr = b2

SOLUTION (9.11)
1 -1
Boundary conditions
are fulfilled by numbering
the nodes as shown and N N
taking w=0 on boundary. -1 1 1 -1
We have,
N ∂ 2w -1 1 1 -1
∇4w + D ∂x 2 =0
coefficient pattern of the
above is shown in Fig.9.11.
Apply Fig.9.11 at node 1: -1 -1

( − w1 − w1 + 0 + 0) + 2( 0 + 0 + 0 + w1 ) − 8( w1 + w1 + 0 + 0) + 20 w1
+ K9 ( −2w1 + w1 + 0) = 0
or w1 ( 4 − K9 ) = 0
or
K = 36 = 3.648π 2
Thus, N cr = 3.648π 2 aD2
The “exact” solution is, N cr = 4π D / a , given by Eq.(9.10) with a=b.
2 2

SOLUTION (9.12)
3 2 -3
Let α = K / n = K / 16.
2

Boundary conditions are


satisfied by numbering the nodes -3 3 2 -3 3
as shown in the figure. Note that
w=0 along the boundary. -4 4 1 -4 4
On applying Fig.9.11 at the -3 3 2 -3 3
point 1, 2, 3, 4, we obtain:

20w1 − 16w2 − 2w1α = 0 3 2 -3


20w2 − 8w1 + 2w2 − 2w2α = 0

20w3 − 8w2 − 8w4 + 2w1 + ( −2w3 + w2 )α = 0


20w4 − 8w1 − 16w3 − 2w4 + ( −2w4 + w1 )α = 0
(CONT.)

91
(9.12 CONT.)
or
⎡20 − 2α − 16 0 0 ⎤ ⎧ w1 ⎫
⎢ −8 22 − 2α 0 0 ⎥ ⎪⎪w2 ⎪⎪
⎢ ⎥⎨ ⎬ = 0
⎢ 2 −8+α 20 − 2α − 8 ⎥ ⎪ w3 ⎪
⎢ ⎥
⎣− 8 +α 4 − 16 18 − 2α ⎦ ⎪⎩w4 ⎪⎭

Determinant of these equations when set equal to zero gives


[20 − 2α ][( 20 − 2α )( 22 − 2α ) − 8(16 )][( 20 − 2α )(18 − 2α ) − 8(16 )] = 0
The lowest root(from the third bracket of the above) is α = 3821
. . Hence,
K = 16α = 61136. . π
= 6193 2
Q.E.D.

SOLUTION (9.13)
1 -1
We have N
∇4w + ( ∂∂xw2 + ∂ 2w
)=0
2
N
D ∂y 2
(a)
N
Deflection w=0 on the boundary 1 1 -1 -1
and nodes labeled as shown owing
to the symmetry conditions. -1 -1 1 1
Molecule from of Eq.(a) is :
1

2 -8 2 1 -1 1
K
1 -8 20 -8 1 + ( 2) 1 -4 1 =0
n
2 -8 2 1

1
Upon application of the above at the node 1 of the plate shown, we obtain :
20w1 + 16w1 + 2w1 + 2w1 + K
9 ( −4w1 − 2w1 ) = 0
from which
K = 60 or K = 6.08π 2 . (Note that “exact” K=5.30.)
Thus,
N cr = 6.08π 2 D
a2

SOLUTION (9.14)
x
Given, a
πy
w0 = a0 sin πax sin b (a) b N
Equation(9.26) together
with Eq.(a) becomes
y N

a0π 2 πy a0π 2 πy ∂ 2w ∂ 2 w1
∇ 4 w1 = 1
D [− N ( a2
sin πax sin b + b2
sin πax sin b ) + N ( ∂x 21 + ∂y 2
)] (b)
(CONT.)
(9.14 CONT.)

92
Take the solution of w1 in the form
πy
w1 = B sin πax sin b
Introducing this into Eq.(b), we obtain
B = − π 2D
2 Na0
2
(1+ a 2 )2 + 2 N
a2 b

Hence,
α0 πy πy πy
w = w0 + w1 = sin πax sin = a 0 sin πax sin − sin πax sin
2 a0 N
1+α b b π 2D 2 b
(1+ a 2 )2 + 2 N
a2 b

Here,
α= π 2D
2N
(1+ a 2 )2
2

a2 b

For a square (a=b) plate, α = Na 2 / 2π 2 D :


πx
w=
a0
1+α sin 2 a

wmax =
a0
and 1+α

SOLUTION (9.15)
L
With the expression
M0 p0 M0
for the moment at any
distance from the left N
end of the beam N x
P0 L P0 L
M ( x ) = M 0 − 12 p0 x 2 2 z, w 2

+ 12 p0 Lx − Nw
Equation (9.31) becomes as given in Eq.(P9.12a). Then,
p0 x 2
d 2w
− w = − D1 ( − + M0 )
N p0 Lx
dx 2 D 2 2
General solution of this equation is
p0 L3 x p0 L2 x 2 p L4 M 0 L2
w = C1 sinh 2Lλx + C2 cosh 2Lλx + 8 λ2 D
− 8 λ2 D
− 160λ4 D + 4 λ2 D
(a)

Boundary conditions are


dw
dx =0 ( x = 0, x = L2 ) ; w=0 ( x = 0, x = L )
Introducing Eq.(a) into the first three conditions :
p L4 p0 L4
C1 = − 160λ3D , C2 = 16 λ3 D
coth λ
p0 L2 p0 L2 p0 L2
M0 = 4 λ2
− 4λ coth λ = 12 f1 ( λ )
in which,
3( λ − tanh λ )
f1 ( λ ) = λ2 tan λ
Upon substitution of the above constants into Eq.(a), we obtain the deflection w, in the form
given, Eq.(P9.15b).

93
CHAPTER 10

SOLUTION (10.1)

p0 a 4 50 (103 )( 0.25 )4 ×12 (1− 0.09 )


(a) wmax = 64 D = 64× 200×109 ( 5×10 − 3 )3
= 133
. mm
Thus wmax < t
2 : linear theory applies. Equation (c) of Sec. 4.5:
σ max = − 3 4p ( at )2 = − 3( 504×10 ) (50) 2 = −93.75 MPa
3
0

p0 a 4 200(103 )( 0.25 )4 ×12 ( 0.91)


(b) wmax = 64 D = 64× 200×109 ( 5×10− 3 )3
= 5.33 mm
Now wmax > t
2 : 5.33 > 2.5 mm. Use Eq.(10.3):
4
= wmax [1 + 0.65( ) ]: 5.33 = wmax +
p0 a wmax 2 0.65 3
64 D 5 25 wmax
or
3
0.65wmax + 25wmax − 133.25 = 0
Solving, wmax = 385
. mm
Using Eq. (c) of Sec. 10.3:
200 (109 ) 200×103 ( 0.25 )2 ( 0.005 )
N3 = 1− 0.09 24 = 2.5(1018 ); N = 1,357,208.8
and
σ m = 1,3570.005
, 208.8
= 271 MPa
We have σ b = − 4 ( at ) = − 4 (50 ) = −375 MPa
3p 2 03( 200 ) 2

Hence
σ max = σ b + σ m = −375 + 271 = −104 MPa

SOLUTION (10.2)
r u
With reference to the figure,
circumferential strain is
( r + u ) dθ − rdθ dθ
εθ = rdθ = u
r Q.E.D.
Since u=u(r) and w=w(r), proceeding as in
Secs. 9.4 and 10.4, we see that (Fig.9.6):
εr = du
dr + 12 ( dw
dr )
2
Q.E.D.

SOLUTION (10.3)

We have a t = 500 10 = 50
σ yp a 2
(a) Et 2
= 280×106
70 (109 )
(50 )2 = 10
With 10 as ordinate in Fig.10.3b for the curve σ max at center of the plate we read
= 2.4
wmax
t (a)
Corresponding to this value, we have from Fig.10.3a:
p0a 4
Et 4
= 50
From which p0 = 50( 70 × 109 )( 501 ) 4 = 560 kPa
(CONT.)

94
(10.3 CONT.)

(b) pall = 43 (560) = 420 kPa


pall a 4 420 (103 )
and Et 4
= 70 (109 )
(50) 4 = 37.5
by Fig.10.3a,
wmax
= 215
t. or
wmax = 215
. (10) = 215
. mm

SOLUTION (10.4)

Given a/t = 600/12 = 50.


σ yp a 2 280(106 )(502 )
(a)
Et 2
= 70(109 )
= 10
With 10 as ordinate in Fig. 10.4b for the curve σ max at the center of the plate:
wmax
t = 3.6
Corresponding to this value, Fig. 10.4a gives
p0 a 4
Et 4
= 20
or p0 = 20(70 × 109 )( 501 ) 4 = 224 kPa

( b ) We have
pall = 34 (224) = 168 ka
pall a 4 3
and
Et 4
= 168(10
70(109 )
)
(50) 4 = 15
By Fig. 10.4a: wmax t = 3.15
From which
wmax = 3.15(12) = 37.8 mm

SOLUTION (10.5)

We have
σ r = 1−Eν (εr + νεθ ) = 1−Eν [ du
2 dr + 2 ( dr ) + ν r ]
1 dw 2 u
2
Q.E.D.
σθ = E
1−ν 2
(εθ + νεr ) = E
1−ν 2
[ ur + ν du
dr + ν2 ( dw 2
dr ) ]

SOLUTION (10.6)

From Table B.3: E = 105 GPa


Refer to Example 10.2, Hence,
wmax = 0.818b 3
p0b
Et

15(103 )(0.2)
= 0.818(0.2) 3 105(109 )(0.003)
= 3.47 mm

and σ = 0.412 3 p02 Eb 2


t2

(15×103 )2 (105×109 )(0.2)2


= 0.412 3 (0.003) 2
= 19.44 MPa

95
SOLUTION (10.7)

Refer to Solution of Prob. 10.6.


wmax = 0.818b 3
p0b
Et

15(103 )(0.2)
Hence 0.004 = 0.818(0.2) 3 105(109 ) t
, t = 1.96 mm

Then σ = 0.412 3 p02 Eb 2


t2

(15×103 )2 (105×109 )(0.2)2


= 0.412 3 (0.00196)2
= 25.8 kPa

SOLUTION (10.8)

From Table B.3, Refer to Example 10.3.


wmax = 0.704a 3
p0 a
Et

= 0.704(0.25) 3 20(103 )(0.25)


(200×109 )(0.005)
= 3.01 mm

SOLUTION (10.9)

From Eq.(10.5), with reference to Sec.4.2:


∇ 4φ = E{[sin θ cos θ ( 1r dw
dr − d 2w
dr 2
)]2 − (cos 2 θ d 2w
dr 2
+ sin 2 θ ⋅ 1r dw
dr )
× (sin 2 θ + cos 2 θ ⋅ 1r
2
d w dw
dr 2 dr )}
Assuming that x coincide with r, or θ = 0 , the above expression reduces to:
∇ 4φ = E{− ddrw2
2
1 dw
r dr }
That is, ∇ φ = − L( w, w )
4 E
2 Q.E.D.
Substitute N r = ( t / r )( dφ / dr ) and N θ = t ( d
2
φ / dr )2
into, Eq.(9.3):
dφ d 2 w d 2w d φ
2
∇4w = + Dt [ 1r cos 2 θ + sin 2 θ
p
D dr dr 2 dr 2 dr 2
dφ d 2φ 1 dw
dr sin θ + dr 2
+ r12 dr dw cos 2 θ ]
2
r dr

Taking x axis coincide with r (or θ = 0 ), the above yields


dφ d 2 w d 2φ
∇4 w = + Dt [ 1r + 1r +
p dw
D dr dr 2 dr 2 dr ] Q.E.D.

SOLUTION (10.10)

Equation (10.7b) becomes



Um = ∫ ∫ (ε + εθ2 + 2νεr εθ )rdrdθ
Et 2
2 (1−ν ) 0
2 r

= πEt
∫ (ε + εθ2 + 2νεr εθ )rdr
2
1−ν 2 r Q.E.D.
Substituting Eq.(P10.2) into the above,
Um = πEt
∫ [( du 2
) + 14 ( dw
dr ) + dr ) + ( r ) + 2ν r ( dr ) + ν r ( dr ) ]rdr
4 du 2 u 2 u du u dw 2
1−ν 2 dr dr ( dw
Q.E.D.

96
CHAPTER 11

SOLUTION (11.1)

Temperature distribution is, 20 54


T ( z ) = 39 + 15 z
0.003 = 39 + 5000 z x
From Eqs.(11.7):
z 20 24 6 mm
t

M * = αE ∫ t T ( z ) zdz = αE (5000 ) z3
0.003
= 14616
2 3
. N
− 2 − 0.003

and
t

N * = α E ∫ t T ( z )dz = α E (39)(0.003)2 = 380 kN / m N xy = 0


2

− 2

Inasmuch as u=v=0, Eqs.(11.6) lead to


N x = N y = − 1N−ν = − 380
0.7 = 542.9 kN / m
*

Also
∇ 2 M * = 0 and w=0. Hence, Eqs.(11.6):
M x = M y = − 1M−ν = − 1460..716 = −208.8 N
*

M xy = 0

SOLUTION (11.2)

From Solution of Prob. 11.1:

M * = 146.16 N N * = 380 kN ⋅ m N x = N y = 542.9 kN m


M x = M y = −208.8 N M xy = 0 t = 6 mm, ΔT = 54 − 24 = 30 0C
ν = 0.3, E = 70 GPa α = 23.2 × 10−6 o C

Use Eqs. (11.8), at the lower surface ( z = t 2) :


σ x = 1t ( N x + 1N−ν ) + t6 ( M x + 1M−ν ) − Eα1−( νΔT )
*
2

9
)(23.2×10−6 )(30)
= [542.9 + 0.97 ] + (6×106 −3 )2 [−208.8 + 146.16
0.97 ] −
3 70(10
10 380
6×10−3 0.97

= 155.8(106 ) − 9.69(106 ) − 50.23(106 ) = 95.9 MPa

σ y = σ x = 95.9 MPa

Apply Eqs. (11.8), at the neutral surface ( z = 0) :


σ x = 1t ( N x + 1N−ν ) − 0 − Eα1−( νΔT )
*

Hence,
σ x = σ y = 155.8 − 0 − 50.23(106 ) = 105.6 MPa

97
SOLUTION (11.3)

Substituting Eq.(11.19) into (11.6) and setting a=b:


∞ ∞
Mx = 16 M *
∑∑ ( m a )2 +ν ( n a )2
sin maπx sin
nπy
− 1M−ν
*

(1−ν )π 4 mn [( m a )2 +( n a )2 a
m n
where
t

M * = αE ∫ t ( ΔT ) zdz =
2 αE ( ΔT ) t 2
4
− 2

Similarly, other components of the bending moment and twisting moment are found as follows:
∞ ∞
My = 16 M *
∑∑ ν ( m a )2 + ( n a )2
sin maπx sin
nπy
− 1M−ν
*

(1−ν )π 4 mn [( m a )2 +( n a )2 ] a
m n
∞ ∞
M xy = − 16 Mπ 4a(12−ν ) ∑ ∑ mn[( m a )12 +( n a )2 ] cos maπx cos
*
nπy
a
m n

SOLUTION (11.4)

On applying Eq.(11.7):
M * = α ∫ ( ΔT ) zdz = AαE ∫ z 4 dz = 1
80 AαEt 5
Using a=b and x = y = a
2 , Eq.(11.19),
∞ ∞ ∞ ∞ m + n −1

wmax = 16 M *
(1−ν ) Dπ 4 ∑∑
m n
sin( mπ 2 )sin( nπ 2 )
mn [( m a )2 +( n a )2 ]
= 16 M *a 2
(1−ν ) Dπ 4 ∑∑ m n
( −1 ) 2
mn [ m2 + n 2 ]

For m=n=1:
wmax = 8 M *a 2
(1−ν ) Dπ 4
= 11.557(10−3 )α Aa 2t 2
Substituting Eq.(11.19) into Eq.(11.6),
∞ ∞
M x = − (116−νM)π 2 ∑ ∑[ ( −1)sin( mπx a )sin( nπy b )
+
ν ( −1)sin( mπx a )sin( nπy b )
] − 1M−ν
* *

a 2 mn [( m a )2 + ( n b )2 ] b2 mn [( m a )2 +( n b )2 ]
m n

At x = y = a
2 , and m=n=1 (and a=b):

M x ,max = 8M *
(1 + ν ) − 1M−ν = 0.693(10−3 )α Et 5 A
*

(1−ν )π 2

SOLUTION (11.5)

Using Table B.3: E = 79 GPa and α = 11.7(10−6 ) o C .


Refer to Solution of Prob. 11.4.
wmax = 11.557(10−3 )α Aa 2t 2
= 11.557(10−3 )(11.7 ×10−6 )( A)(120 × 10−3 ) 2 (5 ×10−3 ) 2
= 48.68(10−15 ) A
Similarly,
M x ,max = 0.693(10−3 )α Et 3 A
= 0.693(10−3 )(11.7 ×10−6 )(79 ×109 )(5 ×10−3 )3 ( A)
= 80.07(10−9 ) A

SOLUTION (11.6)

98
We have
t

M * = αE ∫ t ( Az 2 + Bz ) zdz = αEBt 3
2
1
12 (a)
− 2

( a ) Equation (11.19) with m=n=1 and a=b:


πy
w= 8a2 M *
(1−ν ) Dπ 4
sin πax sin a (b)
At x = y = a
2 :
12 (1−ν 2 ) αEBt 3
wmax = 8a 2 M *
(1−ν ) Dπ 4
= 8a2
(1−ν )π 4 Et 3 12
8 (1+ν )
= π4
a 2αB = 01068
. a 2αB

From Eq.(b), we obtain


πy ∂ 2w πy
∂w
= cos πax sin = − (1−8νM) Dπ 2 sin πax sin = ∂ 2w
*
8 aM *
∂x (1−ν ) Dπ 3 a , ∂x 2 a ∂y 2
∂w πy
∂y = 8 aM *
(1−ν ) Dπ 3
sin πax cos a

Equation (11.6):
πy
M x = − D( ∂∂xw2 + ν ∂ 2w
) − 1M−ν = [ 8(1π+2ν ) sin πax sin − 1]
2 *
M*
∂y 2 1−ν a

At x = y = , M * = αEBt 12 :
2
a
2
8 (1+ν )
M x ,max = M*
1−ν [ π2
− 1] = 0.0064αEBt 3

SOLUTION (11.7)

Stress:
2 EαT1z
σ = σ x = σ y = − Eα1−( ΔνT ) = − (1−ν ) t (a)
EαT1
At z = − t
2 : σ max = 1−ν . EαT1
= 15

Moment:
t t

M = Mx = M y = ∫ σ z dz = − (12 E−ανT)t ∫ z 2 dz
2 2
1

− t2 − t2
EαT t 2
= − 6(1−1ν ) = − 14 EαT1t 2
Strain:
Since clamped plate has no deflection, w=0. Use Eqs. (11.6) to (11.8),
ε x = ε y = E1 [σ − νσ ] + α ( ΔT ) = 0

SOLUTION (11.8)

Refer to Example 11.1.

wmax = α2at (T1 − T2 )


2

−6 2
= 80(10 )(0.22)
2(8×10−3 )
(20) = 4.84 mm

SOLUTION (11.9)

99
With reference to Sec.5.12, for m=n=1:
2πy
w = a11 (1 − cos 2aπx )(1 − cos a ) (a)
Work done equals
W=∫ ∫ p( x , y )w dx dy = a11 ∫ ∫
a a a a 2πy
p( x , y )(1 − cos 2aπx )(1 − cos a )dx dy
0 0 0 0
Strain energy, from Eqs.(5.65) and (a):

∫∫
a a 2
U= ( ∂∂xw2 + ∂ 2w 2
) dx dy = 24π 4 D
2 a11
D
2
0 0 ∂y 2 a2

Potential energy is Π = U − W. From ∂Π / ∂a11 = 0 , we then obtain

∫∫
a a 2πy
a11 = a2
48 Dπ 4
p * ( x , y ) ⋅ (1 − cos 2aπx )(1 − cos a )dx dy
0 0
It follows that
[∫ ∫
a a 2πy
w= a2
48 Dπ 4
p * ( x , y ) ⋅ (1 − cos 2aπx )(1 − cos a )dxdy ]
0 0
2πy
× [(1 − cos 2aπx )(1 − cos a )]

SOLUTION (11.10)

The center deflection wc


of the original plate is obtained
by adding the center deflections C
* b=a x
of a plate loaded by p (Fig.a) and O
of a plate subject to f(y) as a
shown in Fig.(b).
y (a)

f(x)
f(y) f(y)
C C C
M*
1−ν = f(y) x = x + x
1 2

y y y
(b) (c) (d)

We have,
t

M * = αE ∫ t ( ΔT ) zdz = AαEt 5 y 2
2
1
80
− 2

Equation (11.11) is thus,


p * = − ∇1−Mν = − 40AE(1α−tν )
2 * 5

From Sec.5.7, we obtain for plate of Fig.(a):


p*a
( wc ) p* = 0.00192 D = −0.000576 Aαt 2 (1 + ν )a 4
Plate of Fig.(b) may be replaced by the plate of Figs.(c) and (d), as shown. Here,
(CONT.)
(11.10 CONT.)

100

f ( y ) = ∑ M n sin
b


nπy nπy
Mn =
2

b with f ( y ) sin b dy
−b2
n =1
For the plate of Fig.(c), boundary conditions are:
∂ 2 w1
w1 = 0, ∂y 2
=0 ( y = ± b2 ) (a)
w1 = 0 ( x = 0, x = a ) (b)
∂ 2 w1
−D ∂x 2
= f ( y) ( x = 0, x = a ) (c)
Let

w1 = ∑ { An cosh nπbx + Bn sinh nπbx + Cn x cosh nπbx + Dn x sinh
nπy nπy
b }cosh b
n =1
We see that conditions (a) are satisfied by the assumed w1 . Conditions (b) result in
An = 0
Bn sinh nπba + Cn a cosh nπba + Dn a sinh nπba = 0 (1)
The last conditions (c) yield
nπy
( Cn + Dn ) cosh b = − nπbD f ( y ) (2)
Bnα sinh αa + Cn (α a cosh αa + α cosh αa + α sinh αa )
2 2

+ Dn (α 2 a sinh αa + α cosh αa + α sinh αa )] = − D1 f ( y ) (3)


where,
α= nπa
b = nπ ,
f ( y ) = M * / (1 − ν ) = AαEt 5 y 2 / 80(1 − ν )
Solving Eqs.(1) to (3), we obtain Cn , Bn , and Dn . Thus, deflection w1 of plate in Fig.(c) is
known.
To account for fixed edges at y = ± b / 2, we set:
∂w1 ∂w1
∂y = ∂y y = ±b / 2 (d)

Here w2 is the deflection of plate in Fig.(d). For plate of Fig.(d), from Eq.(a) of Sec. 5.6 and
Eq.(c) of Sec.5.7, we write:

f ( x ) = ∑ M m sin maπx
m=1
and at y=b/2,

∂w2
∂y = a
2πD ∑ m
Mm
m [tanh α m (α m tanh α m − 1) − α m ]sin maπx

Hence condition (d) leads to M m . Having M m , Eq.(5.40) gives



w2 = a
2πD ∑ m
sin( mπx / a )
m cosh α m M m ( 2b tanh α m cosh
mπy
a − y sinh
mπy
a )

Next, we evaluate center (x=a/2, y=0) deflections for plates of Figs. (c) and (d): w1c and
w2 c . Then ( ( wc ) M * = w1c + w2 c .
Center deflection of original plate is thus,
wc = ( wc ) p* + ( wc ) M * = −0.000576 Aαt 2 (1 + ν )a 4 + w1c + w2 c

SOLUTION (11.11)

101
Fixed edge solid plate (b=0). We have c1 = c2 = 0 and Eq.(11.28) becomes
w = c3 r + c 4 + w p
2
(a)
Boundary conditions
w = 0, dw / dr = 0 (r=a)
give then
c3 = − 21a w (1)
p , c4 = a2 w (1)
p − wp
(0)
Q.E.D.
Simply supported solid plate (b=0). We have again deflection given by Eq.(a). Boundary
conditions,
w = 0, D( ∂∂rw2 + νr ∂w
+ 1M−ν = 0
2 *

∂r (r=a)
result in
c3 = − 2(11+ν ) [ (1M−ν ) + w (p2 ) + νa w (1) c4 = − c3a 2 − w (p0 )
*

p ], Q.E.D.

SOLUTION (11.12)

We have from Eq.(11.7),


t

M * = αE ∫ Ar 3 z 2 dz = αEAt 3r 3 = Kr 3 , K= αEat 3
2
1 1
12 where 12
− t2
Then,
p* = − ∇1−Mν = − 1−1ν ( ∂∂rM2 + 1r ∂M *
) = − 19−rν K
2 * 2 *

∂r

Formula (11.29) leads to


w p = − D (19 K−ν ) ∫ 1r ∫ r ∫ 1r ∫ r 2 drdrdrdr = − 25(1Kr−ν )⋅D = − 300
αEAt r 5 3 5

D (1−ν )

With reference to Case B of Table 11.3:


M*
c3 = − 2(11+ν ) [ (1−νa) D + w (p2 ) + νa w (1)
p ]
where,
αEAt r
p = − 600 D (1−ν ) , w(p2 ) = − 15αEAt
3 4 3 3
w (1) r
D (1−ν )

Thus,
νKa 3
c3 = − 2(11+ν ) [ (1−Kaν ) D − 1.25Ka
(1−ν ) D − ]=
3 3
0.1 Ka 3
5(1−ν ) D (1+ν ) D
Similarly,
*
c4 = a2
+ w(p2 ) + νa w(1)
p ] − w p = 10 (1+ν ) D +
5
Ma (0) Ka Ka 5
[
2 (1+ν ) (1−ν ) D 25(1−ν ) D
Thus,
w = − 10(1Ka+ν ) D ( r 2 − a 2 ) + −
3
Ka 5 Kr 5
25(1−ν ) D 25(1−ν ) D (a)
At the center (r=0):
wc = Ka 2 1
[
10 D 1+ν + 1
25(1−ν ) ]

Note: As a check it can be shown that w given by Eq.(a) satisfies w = 0 and M r = 0 at r=a.
Substituting the derivatives of w into Eq.(3.48), or alternately using Eqs.(11.8) and (11.25),
we obtain at the plate center (r=0):
σ r = − 1−Ezν ( ddrw + νa ) = − 125zKa = 15 AEαa 3 z
2 3
dw
2 2 dr t3

SOLUTION (11.13)

102
From the solution of Prob.11.12:
αEAt r
w p = − 300 (1−ν ) = − 25 D (1−ν ) K , c1 = c2 = 0.
3 5 5
r

Referring to the case A of Table 11.3:


c3 = Ka 3
10 (1−ν ) D , c4 = Ka 5
10 (1−ν ) D + Ka 5
25(1−ν ) D

Thus, w= Ka 3
10 (1−ν ) D (r 2 − a 2 ) + Ka 5
25(1−ν ) D − Kr 5
25(1−ν ) D (a)
At the center (r=0),
wc = 10(1Ka−ν ) D (1 + ) = 2.6 25(1Kr−ν ) D
5 5
1
25
Substituting the w into Eq.(4.11), or alternately employing Eq.(11.8) together with (11.25), we
obtain the stress at the center of plate:
σ r = − 1−Ezν ( ddrw + νa ) = − 12t 3z Ka 3 1+ν
2
dw
2 2 dr 5 1−ν )

SOLUTION (11.14)

Expression (11.7),
t

M * = αE ∫ t Br 2 z 6 dz = αEBt 7 r 2 = K1r 2 , K1 = αEBt 7 .


2
1 1
448 where 448
− 2

p =− ∇2 M *
= − 1−1ν ( ∂∂rM2 + 1r ∂M *
) = − 1−ν1
* 2 * 4K
Then, 1−ν ∂r
Equation (11.30), setting b=a, becomes
w p = − 16(1−1ν ) D [r 4 − 5α 4 − 4a 2 ( a 2 + 2r 2 ) ln ar + 4a 2 r 2 ]
K

and then we find


p = − 16 (1−ν ) D [4r − 16a r ln a −
K 4a 4
w (1) 1 3 2 r
r ]
With reference to Table 11.3 (Case C), for b=a and a=3a:
c1 = −172 p + 1925
. aw (1) . w (p0 )
where, for r=3a:
3.37 a 3 K1 1.782 a 4 K1
p = −
w (1) (1−ν ) D , w (p0 ) = w p = − (1−ν ) D
Hence,
5.796 a 4 K1 3.430 a 4 K1 2.366 a 4 K1
c1 = (1−ν ) D − (1−ν ) D = (1−ν ) D
Similarly,
0.847 w(p1 ) 0.779(p0 ) 1.466 a 2 K1
c2 = − a + a2
= (1−ν ) D
4.324 a 2 K1 2.409 a 2 K1 1.915 a 2 K1
c3 = − (1−ν ) D + (1−ν ) D =− (1−ν ) D

c4 = 0
It can now be shown that w = wh + w p satisfy boundary conditions. The deflection w is
thus, at r=2a:
K1a 4
w= (1−ν ) D [2.366 ln( 2) + 1466
. ln( 2) − 1913
. (3)]
K a4
− 16(1−1 ν ) D [16 − 5 − 4 ln( 2) − 32 ln( 2) + 16]
K a4 K a4 K a4
or w = −0.040 (1−1 ν ) − 0128
. (1−ν ) = −0168
1
. 1
(1−ν ) D

= −0.045(1 + ν )αBt 4 a 4

103
CHAPTER 12

SOLUTION (12.1)

Use Eq.(12.3) with φ = π / 2, ps


Ps ⋅π ( a sin π )2
Nφ = =−
ps a
2πa sin 2 φ 2
r0 = a sin φ
and φ
σφ =
ps a
Q.E.D. a r1 = r2 = a
2t

Alternatively, substitute Eqs.(12.16) O


into Eq.(c) Sec.12.5,
N φ = − a sin1 2 φ [ ps a 2 ∫ sin φ cos φdφ + c ] = 1
sin 2 φ
[ 14 ps cos 2φ − ac ]
At φ = π / 2: N φ = − psπa 2 / 2πa = − ps a / 2. Thus,
− =− − ∴ =
ps a ps a c c ps a
2 4 a a 4 .
ps a 1− cos 2φ
∴ σφ =

Hence, N φ = − = =
ps a ps a
4 sin 2 φ 2 t 2t Q.E.D.
Then, from Eq.(12.3), N θ = − N φ − p z a:

Nθ = − ps a cos 2 φ = − 12 ps a ( 2 cos 2 φ − 1)
ps a
2
and
σθ = − cos 2φ
ps a
2t Q.E.D.

SOLUTION (12.2)

( a ) Use Eqs. (P12.1):


σ φ = − p2ta = − 2(30
3
2(10 )(9)
s
×10 )
= 0.3 MPa −3

At φ = 0, σ φ = σ θ ,max

( b ) Apply Eq. (f) of Sec. 12.3:


−3
σ cr = k Etr = 0.25 (110×10 )(30×10 9
)
9
= 91.7 MPa
Thus, there is no possibility of buckling, as σ φ = σ θ ,max < σ cr .

SOLUTION (12.3)

Let p represent the pressure differential.


At crown,
pa 2 0.152 p
σ = σ φ = σθ = 2 bt or σ= 2 ( 0.12 )( 0.006 ) = 15.625 p
From the above, p = σ / 15.625 = 14 × 10 / 15.625 = 896 kPa
6

At equator,
σφ = = 12.5 p, σθ =
(1 − 2ab2 ) = 3.28 p
pa pa 2
2t t
Clearly, these equations result in larger values for p .

104
SOLUTION (12.4)

Refer to Solution of Prob. 12.3. We now have a uniform stress:


σ = σφ = σθ = pa
2t or σ= 0.15 p
2(0.006) = 12.5 p
This gives
28(106 )
p = 12.5
σ
= 12.5 = 2.24 MPa

SOLUTION (12.5)

Expression (12.22) and Hooke’s law give


γ tan α νy
εθ = E1 (σθ − νσ φ ) = E1 { cos α [ y ( h − y ) − (h −
2y
2 3 )]}
( a ) Hence, εθ = 0 when :
νy
y ( h − y ) − 2 ( h − 23 y ) = 0 or y 2 ( 23 ν − 2) + hy ( 2 − ν ) = 0
From the above, for y ≠ 0 ,
h ( 2 −ν )
y = 2−( 2ν / 3)
Taking, ν = 0.3:
3( 2− 0.3 )
y = 2−( 0.6/ 3) = 2.815 m
( b ) To ascertain at which hoop strain is maximum, we set dεθ / dy = 0. In so doing ,

2 y − h + ν2 ( h − )=0
4y
3
or
h ( 2−ν ) 3( 2 − 0.3 )
y= 4 (1−ν / 3 ) = 4 (1− 0.1) = 1407
. m

SOLUTION (12.6)
k l
x
(a) x = a sin φ a
y = a cos φ y φ
Vkmnl = πx 2 y m n
h = a-y
VmOn = 13 πh 2 (3a − h ) O x
γ y
F = −γπx 2 y − 3 π ( a − y ) 2 ( 2a + y )
= − 13 γπ [3x 2 y + 2a 3 + y 3 − 3a 2 y ]
= − 13 γπa 3 [3 sin 2 φ cos φ + 2 − cos 3 φ − 3 cos φ ]
= − 13 γπa 3 [3 cos φ (1 − cos 2 φ ) + 2 − cos 3 φ − 3 cos φ ]
= − 23 γπa 3 [1 − cos 3 φ ]
Thus,
γa 2 1− cos3 φ
N φ = − 2πa sin
F
2
φ
= 3 sin 2 φ
γa 2 cos3 φ
= 3 [ 1−cos1 2 φ − 1−cos2 φ ]
γa 2 cos 2 φ cos3 φ γa 2 cos2 φ (1− cos φ )
= 3 [1 + 1− cos2 φ − 1− cos2 φ ] = 3 [1 + (1+ cos φ )(1− cos φ ) ]

(CONT.)

105
(12.6 CONT.)
γa 2 cos 2 φ
σ φ = tφ
N
or Nφ = 3 [1 + 1+ cos φ ], Q.E.D.
Equation (12.3), with p z = γy = −γa cos φ , gives then
γa 2
cos φ
2
Nθ = 3 [3 cos φ − 1 − 1+ cos φ ]
or
γa 2
Nθ = [2 cos φ − 1+ cos σθ =
1 Nθ
3 φ ], t Q.E.D.

γa 2 γa 2 cos2 φ
(b) σθ = 3t [2 cos φ − 1+ cos
1
φ ], [1 + 1+ cos φ ] σφ = 3t

From Eq.(d) in Sec.12.7, with r1 = r2 = a:

dφ − v cot φ = E [σ φ ( r1 + νr2 ) − σ θ ( r2 + νr1 )]


dv 1

a 3γ (1+ν ) cos 2 φ
= 3 Et [1 + 1+ cos φ − 2 cos φ + 1+ cos
1
φ]
a 3γ (1+ν ) 1+ cos 2 φ a 3γ (1+ν ) 2 − cos φ − cos 2 φ
= 3 Et [1 − 2 cos φ + 1+ cos φ ]= 3 Et [ 1+ cos φ ]
or
a 3γ (1+ν )
dv
dφ − v cot φ = 3 Et [ 2
1+ cos φ − (cos φ )] = f (φ )
Then, from Eq.(e) in Sec.12.7:
v = [∫
f (φ )
sin φ dφ + c ]sin φ
= {∫ ( 3Et sin φ [ 1+cos
a 3γ (1+ν )
φ − cos φ ]) dφ + c}sin φ
2

a 3γ (1+ν ) sin φ
= 3 Et [
1+ cos φ − sin φ ln(1 + cos φ )] + c sin φ
a 3γ (1+ν )
Since, v = 0 at φ = 90 o , the foregoing gives c = − 3 Et . Therefore
a γ (1+ν ) sin φ
− sin φ ln(1 + cos φ ) − 1]
3
v= 3 Et [
1+ cos φ (a)
By Eq.(c) of Sec.12.7 and Eq.(12.25b),
1
E (σθ − νσ φ ) = a1 ( v cot φ − w )
from which
w = v cot φ − Ea (σθ − νσ φ )
Thus, substituting Eq.(a):
a 3γ (1+ν )cot φ sin φ
w= 3 Et [ 1+cos φ − sin φ ln(1 + cos φ ) − 1]
γa 2 γa 2 cos2 φ
− Ea [ 3t ( 2 cos φ − 1+cos
1
φ)−ν 3t (1 + 1+cos φ )]
or
γa 3 sin φ
w= 3 Et {(1 + ν ) cot φ [1+ cos φ − sin φ ln(1 + cos φ ) − 1]
cos2 φ
− [2 cos φ − 1+cos
1
φ − ν (1 + 1+ cos φ )]}

Note that at φ = 90 o :
γa 3 γa 3
v = 0, w = 3 Et . ) = 0.433
(13 Et
Similarly, at φ = 0:
1.3a 3γ γa 3 γa 3 γa 3
v= 3 Et ( −1) = −0.433 Et , w= 3 Et [−2 − 12 − 0.3( 23 )] = −0.983 Et

106
SOLUTION (12.7)

We write
p = 100 + γx (1)
( a ) At x = 2 , Eq.(1) gives
h

p = 100 + 10( 7.5) = 175 kPa


t= = = 10 mm
pa 175( 8 )
From Eq.(12.10), σ all 140 (103 )

( b ) At x = 5 h 8 , Eq.(1) becomes
p = 100 + 10( 85 × 15) = 193.75 kPa
and
t= = 1107
193.75( 8 )
140 (103 )
. mm
( c ) We have, from Eq.(c) of Sec.4.5, with p = 100 kPa:
2
t top = [ ]2 =[ ] 2 = 185.2 mm
1 1
3 pa 3(100)( 64 )
4 σ all 4 (140×103 )

SOLUTION (12.8)

We have F Nφ
r1 = 172 mm , r2 = 82.5 mm
F = πri2 p , p = −0.09 MPa
Equation (12.1b):
π ( 0.081)2 ( 90 , 000 )
N φ = − 2πrF2 (1) = 2π ( 82.5 )(1) = 3579
. kN / m
Then
σφ =

t = 3, 579
0.003 = 1193
. MPa
Equation(12.1a):
σθ
0.0825 + 1193
.
0.172 = 0.09
0.003 , σθ = 1903
. MPa

SOLUTION (12.9)

2b = 12 (3 + 2), b = 125
. m, 2a = 12 (3 − 2), a = 0.25 m , Inner radius, ro = 2
2 = 1 m.
From Eq.(12.18),
pa ( ro + b ) 1.6×106 ( 0.25 )(1+1.25 )
t= 2 roσ all = 2×1× 200 (106 )
= 2.25 mm
Also
1.6 (106 )( 0.25 )
t= = = 1 mm
pa
2σ all 2 ( 200×106 )

SOLUTION (12.10)
Maximum stress occurs at the bottom of the tank. Applying Eqs.(12.24) with φ = 180 o :
γa 2
[5 + 12−( (−−11) ) ] = γa 2
2
Nφ = 6
γa 2
[1 − 6( −1) − 12−( (−−11) ) ] = −γa 2
2
Nθ = 6
Thus
γa 2 9.81(103 )(102 )
σφ = t ; 125(10 6 ) = t , t = 7.85 mm

107
SOLUTION (12.11)

Use Eqs. (12.23) with φ = 90o :


Nφ = γ 6a (1 − 12cos γa
2 2

+ cos90 ) = 6
2
90

2
Nθ = γ 6a (5 − 6 cos 90 + 12cos
+ cos90 ) =
5γ a 2
2
90
6
Therefore
σ φ = γ6at ; 125(106 ) = 9.81(106t )(10)
2 6 2

Solving t = 1.31 mm

SOLUTION (12.12)

p = − pz = γ ( H + h − y ) , ro = y tan α , sin φ = cos α


The second of Eqs.(12.8),
N θ = γ ( H + h − y ) y cos
tan α
α Q.E.D.
We also have
F = −γ ( H + h − y )πro2 − γπro2 = −πγy 2 ( H + h − 23 y ) tan 2 α
y
3
The first of Eqs.(12.8) is thus
γ tan α
N s = 2 ( H + h − 23 y ) y cos α Q.E.D.

SOLUTION (12.13)

We write, referring to Fig.P12.13 :


r0 = s cot φ , r = s cot φ , pz = 0 , py = 0 , F = 2π
s0
s r0 pe
Expressions (12.8) yield,
2πr0 ( s0 / s ) pe
Ns = − = − s⋅sin ∴ σ s = − st 0sineφ
s p
0 e s p
2πr0 sin φ φ Q.E.D.

Nθ = ∴ σθ = 0
=0
pz r0
and sin φ Q.E.D.
Owing to the symmetry in loading τ sθ = 0.

SOLUTION (12.14)

Equation (12.8):
γ ( h1 − y ) y tan α
σθ = =
pr0
t sin φ t cos α
a b
γ ( h1 − y ) y tan α φ r2
= t cos α Q.E.D. r0
c d
where, a
y h1
r0 = y tan α , φ = π
− α , sin φ = cos α =
r0
2 r2 f g
y tan α i h2
r2 = =
r0
cos α cos α α e
We can write ( referring to a Math Table ):
F = Fabcd + Fced − F feg + F fig
= −[γ ( h1 − y )(πr22 cos 2 α ) + 13 πr22 cos 2 α ⋅ yγ − 13 πa 2 cos 2 α ⋅ h2γ
+ γπ ( 23 a 3 − a 3 sin α + 13 a 3 sin α )] (a)

(CONT.)

108
(12.14 CONT.)

The first of Eqs.(12.8) gives


N s = − 2π ⋅ y tanFα ⋅cos α = − 2πyFsin α (b)
Upon introducing Eq.(a) into Eq.(b) we obtain N s . Then arranging terms properly from
N s / t we find σ s , as given by Eq.(P12.14).

SOLUTION (12.15)

V = Vkmnl + VmOn
k l
= π ( a sin φ ) 2 x + π3 ( H − x ) 2 (3a − H + x )
= π3 [3x( a sin φ ) 2 + ( H − x ) 2 (3a − H + x )] a
F = −γV
γπ x
=− 3 [3x ( a sin φ ) 2 + ( H − x ) 2 (3a − H + x )] H
From Eq.(12.3): α
N φ = − 2πa sin
F
2
φ φ
a
γ m n
= 6 a sin 2 φ
[3x ( a sin φ ) 2 + ( H − x )2 (3a − H + x )]
and O
γ h=H-x
σφ = [3xa 2 sin 2 φ + ( H − x ) 2 (3a − H + x )]

t = 6 a sin 2 φ

We have p = − p z = γx . Hence N φ + N θ = − pz a = γax .


Thus
N θ = γax − N φ
γ
= γax − 6 a sin 2 φ
[3x ( a sin φ ) 2 + ( H − x ) 2 (3a − H + x )]
γ
= 6 a sin 2 φ
[3xa 2 sin 2 φ − ( H − x ) 2 (3a − H + x )]
Hence
γ
σθ = = [3xa 2 sin 2 φ − ( H − x ) 2 (3a − H + x )]

t 6 at sin 2 φ

SOLUTION (12.16)
3
[1+( dy dx )2 ]
r1 =
2

d2y
, t=5 mm
dx 2

Referring to the sketch : 50


sin φ = 4
17 r2
r2 = 50( 17
4 ) = 5154
. mm φ
At y=100, we have x=50, and 1
4
d2y
= = 14 , =
dy 50 1
dx 12.5 dx 2 12.5
3
[1+16 ] 100
r1 = = 876.2 mm
2
Hence 1
12 . 5

Equation(12.1b), with F = −πp0 x :


2
y
πpo x 2
Nφ = = = = 0.026 po
po x po ( 0.05 ) x
2πx sin φ 2 sin φ 2 ( 4 17 )

(CONT.)

109
(12.16 CONT.)

+ = po : po = 0.8762o +
Nθ 0.026 p Nθ
Then r1 r2 0.05154

Solving, N θ = 0.05 po . Thus

σθ = σφ =

= 10 po = 50 MPa = 5.2 po = 26 MPa

t t

SOLUTION (12.17)

With reference to Fig.12.15:


r0 = s ⋅ sin α , r = s ⋅ tan α ,
2 sin φ = cos α
( s /cos α )− s cos α
cos φ = y / r2 = s⋅tan α = 1− cos α
tan α ⋅cos α = sin 2 α
tan α ⋅cos α , p z = p cos φ
The second of Eq.(12.8) is therefore,
N θ = − p z r2 = − ps ⋅ sin α ⋅ tan α Q.E.D.
Weight of the cone equals
F = ∫ p ⋅ 2πr2 sin φ ⋅ ds = πp cos φ ( L2 − s 2 ) + c
L

s
As no force act at top c=0. The first of Eqs. (12.8) becomes
πp cos φ ( L2 − s 2 ) p ( L2 − s 2 )
Ns = − 2π ⋅s sin α ⋅sin φ =− 2 s⋅cos α Q.E.D.

SOLUTION (12.18)

Equations (12.23) lead to


γa 2 2 cos2 φ γa 2 2 cos 2 φ
σφ = 6t (1 − 1+ cos φ ) = σ1 , σθ = 6t (5 − 6 cos φ + 1+ cos φ ) = σ 2
It follows that
1 γa 2 cos 2 φ 2 cos 2 φ
2
τ max = 12 (σθ − σ φ ) = 2 6t (5 − 6 cos φ + 1+ cos φ −1+ 1+ cos φ )
γa 2 2 cos 2 φ γa 2 2+ 2 cos φ −3 cos φ −3 cos 2 φ + 2 cos 2 φ
= 6t ( 2 − 3 cos φ + 1+ cos φ ) = 6t ( 1+ cos φ )
γa 2
1+ cos φ +1 γa
)(1 − cos φ ) = φ )(1 − cos φ )
2
= 6t ( 1+ cos φ 6t (1 + 1+cos
1
Q.E.D.

SOLUTION (12.19)

Redo first part of Example 12.6 with


p = − p z = −γ ( h + a − a cos φ )
to readily obtain the results given by Eqs.(P12.19).

SOLUTION (12.20)

Substituting Eqs.(12.23) and r1 = r2 = a into Eq.(d) in Sec. 12.7


1 γa 2 cos2 φ 2 cos2 φ
2
f (φ ) = Et 6 [a(1 + ν )(1 − 1+cos φ ) − a(1 + ν )(5 − 6 cos φ + 1+ cos φ )]
γa (1+ν )
3
2 cos φ
2
2 cos φ
2
= 6 Et [1 − 1+ cos φ − 5 + 6 cos φ − 1+cos φ ]
γa 3 (1+ν ) 4 cos 2 φ γa 3 (1+ν ) cos2 φ + cos φ − 2
= 6 Et [6 cos φ − 1+ cos φ − 4] = 6 Et [ 1+ cos φ ]
γa (1+ν )
3
= 6 Et [cos φ − 1+cos
2
φ]

(CONT.)

110
(12.20 CONT.)

Equation (e) of Sec. 12.7 is thus,

∫ (cot φ −
γa 3 (1+ν ) 2 cos ecφ
v = {[ 3 Et 1+ cos φ )dφ ] + c}sin φ (a)
We integrate the above to obtain v . Then we choose c such that v = 0 at φ = φ0 .
Then apply Eq.(a) of Sec. 12.7 to determine
w = v cot φ − Ea (σθ − νσ φ )
where σθ and σφ are given by Eqs.(12.23).

SOLUTION (12.21)

We have : pr = −γx , p x = 0, N xθ = 0, f 2 (θ ) = 0. Equation (12.31) yields,


γbx
N θ = − pr = − pr b = γbx, σθ = t Q.E.D.
Equation (12.30c),
dN x / dx = 0 ∴ N x = c1 or σx = c Q.E.D.
where c = c1 / t
Refer to Fig. P12.19. We have φ = 30 for a=10 m and b=5 m. Stresses in the bottom part,
o

from Eq.(P12.19):
γa 2
σθ = 6t [3 + 5 − 6( 23 ) + 2 1+ 43 ] = 0.601γ
(3 )
2
a2
t

σφ = − γ6at [3 + 1 − 2 1+( ) ] = −0.533γ


2 3
4 a2
3 t
2

Therefore
0.533(14×103 )(10)2
100(10 6 ) = t , t = 7.46 mm
Upper part (see Eqs. P12.21):
γbx
σθ = t where x = h + a (1 − cos 30 ) = 10 + 134
. = 1134
. m o

Thus
14(103 )5(11.34 )
100(10 6 ) = t , t = 7.94 mm

SOLUTION (12.22)

Loading may be expressed by


pr = −γa sin θ , pθ = p x = 0
The first of Eqs.(12.31) is thus,
N θ = γa 2 sin θ Q.E.D.
The second of Eqs.(12.31) becomes
N xθ = −γax cos θ + f1 (θ )
From symmetry N xθ = 0 at x=0.

This gives f1 (θ ) = 0. Hence,


N xθ = −γax cos θ Q.E.D.
The third of Eqs.(12.31) reduces to
N x = − 12 γx 2 sin θ + f 2 (θ )
(CONT.)

111
(12.22 CONT.)

No extension along a longitudinal fiber (for any θ ) to occur :


L

∫ ( N x − νN θ )dx = 0
2
1
Et
−L 2

Substituting into the above N x and N θ we obtain,


f 2 (θ ) = γ ( 24 sin θ + νa 2 sin θ )
2
L

Then,
N x = − 12 γx 2 sin θ + 1
24 γL2 sin θ + νN θ
= 12 γ ( 12 − x 2 ) sin θ + νN θ
2
L
Q.E.D.

SOLUTION (12.23)

We have : N θ = − pr a , pθ = p sin θ = p x = 0, pr = p cos θ .


Expression (12.31),
N xθ = − ∫ ( pθ + 1 ∂N θ
a ∂θ )dx + f1 (θ ) , N x = −∫ 1 ∂N xθ
a ∂θ dx + f 2 (θ )
Since no torque is applied at ends, N xθ = f1 (θ ) = 0. Therefore
N θ = − pa cos θ ∴ σθ = − cos θ
pa
t Q.E.D.
2 px sin θ
N xθ = −2 px sin θ + f1 (θ ) ∴ τ xθ = − t Q.E.D.
Then,
N x = −∫
px 2
cos θ dx + f 2 (θ ) = − cos θ + f 2 (θ )
2 px
a a

pL2
The condition that N x = 0 at x = ± L / 2 leads to f 2 (θ ) = 4a cos θ .
Hence,
σ x = − 4 at ( L2 − x 2 ) cos θ
p
Q.E.D.

SOLUTION (12.24)

We have pr = p cos θ , pθ = p sin θ , p x = 0 , and r = a .Thus


N θ = − pr r = − pa cos θ (a)

N xθ = − ∫ ( pθ + 1 ∂Nθ
a ∂θ )dx + f1 (θ ) = −2 px sin θ + f1 (θ ) (b)

N x = −∫ 1 ∂N xθ ∂f1 (θ )
a ∂θ dx + f 2 (θ ) = a1 [ px 2 cos θ − ∂θ x ] + f 2 (θ ) (c)
L

f1 (θ ) = 0 and ∫ ( N x − νN θ )dx = 0 .
2
Boundary conditions :
−L 2

. Substitute in the latter Eqs. (a) and (c), integrate, and solve for f 2 (θ ) :
2
f 2 (θ ) = − 12 a cos θ − νpa cos θ
pL

Solution is thus
p cos θ
σθ = − cos θ σx = (12 x 2 − L2 − 12νa 2 )
pa
t 12 at

τ xθ = − sin θ
2 px
t

112
SOLUTION (12.25)

For the shell under consideration :


p r = − p, r = a, pθ = p x = 0 L L
2 2
Equations (12.31):
N θ = − pr r = pa σθ =
pa
or t O x
N xθ = − ∫ ( pθ + 1 ∂Nθ
r ∂θ )dx + f1 (θ )
N xθ = f1 (θ )

Since no torque is applied at ends, N xθ = f1 (θ ) = 0. Thus,


N x = − ∫ ( px + 1 ∂N xθ
r ∂θ )dx + f 2 (θ ) or N x = f 2 (θ ).
The condition
L L

∫ ( N x − νN θ )dx = ∫ L [ f 2 (θ ) − νpa ]dx = 0


2 2

−L 2 − 2

yields, after integration, f 2 (θ ) = νpa. It follows that


νpa
N x = νpa and σx = t

SOLUTION (12.26)

The loading is pθ = p sin θ , pr = p cos θ , and p x = 0 . Equations(12.31), with r=a:


N θ = − pa cos θ Q.E.D.
and
N xθ = − ∫ ( p sin θ + p sin θ )dx + f1 (θ ) = −2 xp sin θ + f1 (θ )
N x = −∫ 1 ∂N xθ
dx + f 2 (θ ) = cos θ − + f 2 (θ )
x2 p x df1
a ∂θ a a dθ

Since no torque is applied at ends, N xθ = f1 (θ ) = 0.


Boundary conditions : N x = 0 at x = ± L / 2 lead to
f 2 (θ ) = − cos θ
L2 p
4a
Hence,
N xθ = −2 xp sin θ
Q.E.D
N x = − 4 a ( L2 − 4 x 2 ) cos θ
p

SOLUTION (12.27)

From Problem 12.26 : N θ = − pa cos θ . Then


σθ = − 1t pa cos θ Q.E.D.
Equations (12.31) lead to
x2 p
N xθ = −2 xp sin θ + f1 (θ ) , N x = a cos θ − ax + f 2 (θ )
df1

From symmetry N xθ = 0 at x=0. Thus f1 (θ ) = 0. Hence,

cos θ + f 2 (θ )
x2 p
Nx = a

(CONT.)

113
(12.27 CONT.)

Inasmuch as ends are not free to move axially,


L L

0 = ∫ L ( N x − νN θ )dx = ∫ L [ cos θ + f 2 (θ ) + νpa cos θ ]dx


2 2 x2 p
a
− 2 − 2

or, after integration,


f 2 (θ ) = − p( 12L a + νa ) cos θ
2

The stress resultants are thus,


N x = p( xa − 12L a − νa ) cos θ , N xθ = −2 xp sin θ
2 2
Q.E.D.
Stresses are obtained by dividing above equations by t.

SOLUTION (12.28)

Since pr = 0, the first of Eqs.(12.31) gives N θ = 0.


Remaining equations of (12.31) become
df1 (θ )
N xθ = f1 (θ ), N x = − ax dθ + f 2 (θ )

The condition that N x = 0 at x=0 gives f 2 (θ ) = 0.


Thus, in line with the result of Example 12.9, we take
df1 (θ )
N x = − ax dθ = A1 cos θ

Integrating the above


f1 (θ ) = − ∫ cos θdθ = − sin θ + A2
aA1 aA1
x x

Clearly, from symmetry : N xθ = 0 at θ=0 and θ = π . Hence, A2 = 0.

The vertical component of shear resultant integrated over the annular cross section must be
equal to − P :
π π
∫ ∫
2 a 2 A1
( N xθ / t ) sin θ ⋅ tadθ = − P2 − sin 2 θdθ = − 2πxa = − P
2
or x
0 0

Solving A1 = px / πa 2 . The stresses are therefore,


σθ = 0, τ xθ = − πPat sin θ , σ x = πPx a 2t
cos θ Q.E.D.

SOLUTION (12.29)

The quantities N x , N y , N xy = N yx are forces per unit length of the element ABCD
through which they are transmitted (Fig.12.19). The lengths of the sides AB=CD and AC=BD are:
dy dx
cos θ and cos φ . The horizontal x-directed components of the total forces are thus
) cos φ = N x dy , ) cos φ = N yx dx
dy dx
(N x cos θ ( N xy cos φ (a)

Similarly, the y-components are


) cos θ = N y dx , ) cos θ = N xy dy
dx dy
(N y cos φ ( N xy cos θ (b)
Equations (a) and (b) give
cos θ cos φ
Nx = Nx cos φ , Ny = Ny cos θ , N xy = N xy = N yx Q.E.D.

114
SOLUTION (12.30)

The tangential, axial, and radial stresses are given by


σ1 = pr
t = 24 p σ2 = pr
2t = 12 p σ3 = 0

From Table B.3, σ yp = 345 MPa . Introducing the foregoing into Eqs. (2.34b)
and (2.35), we obtain the critical pressures as follows:

The maximum shearing stress theory:


24 p = 1
2.5 (345 ×106 ) , p = 5.75 MPa

The maximum energy of distortion theory:


p(242 − 24 × 12 + 122 )1 2 = 1
2.5 (345 × 106 ) p = 6.64 MPa

The permissible value of the internal pressure is therefore limited to 5.75 MPa .

SOLUTION (12.31)

For r0 = b − a, Eqs.(12.18) give


σ φ = σ1 = = 2172 σθ = σ 2 =
pa 2 b − a pa pa
2 t b− a . 2t , 2t

We then have from Eq.(2.35),


σ yp
(σ12 − σ1σ 2 + σ 22 ) 2 =
200 (106 )
. 2 − 2176 + 1) =
1 pa
n , 2t ( 2176 . n
or
400×106 ( t ) 400×106 ( 0.001)
n= 1.886 pa = 1.886×1.5×106 ( 0.06 )
= 2.36

115
CHAPTER 13

SOLUTION (13.1)

We have E = 70 GPa (by Table B.3). Use Eqs. (13.14), (13.24), and (b) of
Sec. 13.7.
β 4 = 3(1a−tν ) = (0.1)3(0.91) = 57.485 m −1
2

−3 2
2 2
(5×10 2
)
Pa 2 β 3 2
wmax = 2 Et = 150(10 )(0.1) (57.485)
2(70×109 )(5×10−3 )
= 0.123 mm

σ x ,max = 3P
2βt2
= 3(150×103 )
2(57.485)(5×10−3 )2
= 156.6 MPa

σ θ ,max = 2
(− at + β32νt 2 )
3
= 150(10 )(57.485)
2 [− 100
5 + (57.485)2 (5×10−3 ) 2 ]
3(0.3)

= −39.26 MPa

SOLUTION (13.2)

Refer to Solution of Prob. 13.1.


Pa 2 β
wmax = 2 Et

Substitute the numerical values:


0.1(10−3 ) = 150(10
3 2
)( a )(57.485)
2(70×109 )(5×10−3 )
, a = 90.1 mm
Thus,
d = 180.2 mm

SOLUTION (13.3)

We have, from Eq.(b) of Sec.13.7:


σ1 = 3P
2βt2
, σ 2 = − P2βta + 23νβ tP 2 (a)
where,
β 4 = 3(1a−tν ) = (0.06)3(0.91) β = 117.34 m −1
2

2 2
(2×10 2 −3 2
)
;
Hence
σ1 = 3P
2βt2
= 3P
2(117.34)(2×10−3 )2
= 3196 P

σ2 = 2 ( − at + 3ν
β 2t 2
)
= P (117.34)
2 [− 602 + (117.34)3(0.3)
2
(2×10−3 )2
]
= 95.9 P − 1760 P = −801P
Equation (2.35) with n=1:
σ12 − σ1σ 2 + σ 22 = σ yp2
P 2 [(3196) 2 − (3196)(−801) + (−801) 2 ] = (210 × 106 ) 2
or
P = 57.3 kN / m

116
SOLUTION (13.4)

Refer to solution of Prob.13.3. Now we have


σ1 − σ 2 = σ yp : P[(3196) − (−801)] = 210(106 )
Solving P = 52.5 kN / m

SOLUTION (13.5)

With reference to Fig. (a) p c


and Example 13.1 :
O

b pdx x
w0 = e − βx f1 ( βx )dx
0 8β D
3

−∫
c
f1 ( βx )dx
p
z
0 8β D
3

L
After integrating the above expression, we obtain b
pa 2 − βc − βb
w0 = 2 Et [e cos( βc ) − e cos( βb)] Fig. (a)

SOLUTION (13.6)

Maximum shear stress theory, Eq.(2.34) with n=1, referring to Eqs.(b) of Sec. 13.7:

σ1 − σ 2 = σ yp ; 3P
2 βt 2
− 2 ( − at + 3ν
β 2t 2
) = 200 × 10 6 (a)
where,
a = 0.25 m, t = 0.005 m, ν = 0.3, β = ( 0.253××00.91.005 ) = 36.3568
1
4
2 2

Expression (a) becomes,


3P
2 ( 36.3568 )( 0.005 )2
− 36.3568
2
P
[− 00..005
25
+ ( 36.35680).29( 0.005 )2 ] = 200 × 10 6
Solving
P = 96.89 kN / m

SOLUTION (13.7)
3(1−ν 2 ) 3(1− 19 )
β4 = a 2t 2
= ( 0.4 )2 (10×10 − 3 )2
or β = 20.2 m−1
Apply Eqs.(b) of Sec.13.7:
σ1 = 3P
2 βt 2
= 3P
2 ( 20.2 )( 0.01)2
= 742.57 P

σ2 = 2 ( − at + 3ν
β 2t 2
)
= [− 400
10 +
P( 20.2 ) 3( 13 )
2 ( 20.2 )2 ( 0.01)2
]
= −404 P + 247.5 P = −156.47 P

(a) 742.57 P = σ u : 742.57 P = 340(10 6 )


or P = 457.9 kN / m

σ1 σ ( −156.47 P )
(b) σu − σuc2 = 1: 742.57 P
340 − 620 = 10 6
Solving P = 410.4 kN / m

117
SOLUTION (13.8)
From Eq.(13.23), ΔM x = f 3 ( βx ). Thus,
pdx

Mx = ∫ f 3 ( βx )dx + ∫
b p c p
f 3 ( βx )dx = [ f 2 ( βb ) + f 2 ( βc )]
p
4β 4β 8β 2
0 0
For c=b :
M max = f 2 ( βb) = M x ,max
p
4β 2

Hence, with N x = 0:
σ x ,max = f 2 ( βb )
6 M x ,max
+ =
Nx 3 p
t2 t 2 β 2t 2

We also have,
6 ( νM x ,max ) 3 νp
σθ ,max = − Ew
a + t2
= − pa[1 − f 4 ( βb )] + 2 β 2t 2 f 2 ( βb )
and, since N xθ = M xθ = 0:
τ xθ = + =0
N xθ 6 M xθ
t t2

SOLUTION (13.9)

Refer to Solution of Prob. 13.8.


β 4 = 3(1a−tν ) = (0.3)3(0.91) , β = 26.238 m −1
3

−3 2
2 2
(8×10 2
)
and
β b = 26.238(0.061) = 1.6
Table 13.1:
f 2 ( β b) = 0.202
Thus
σ x ,max = 3 p
2 β 2t 2
f 2 ( β b)
10(106 )
= 3
2 (26.238)2 (8×10−3 )2 (0.202) = 68.8 MPa

SOLUTION (13.10)
0.05 m
P P
We have
3(1− 0.09 ) 0.025 m
β = [ 0.375 ] = 20.9906 m−1
1
4
2 2
( 0.01)
At x=0.025:
βx = 20.9906 × 0.025 = 0.52476 rad .
= 30.066 o
Referring to Table 13.1, or by direct computation :
f1 ( βx ) = e −0.52476 (cos 30.066 + sin 30.066) = 0.80852
f 3 ( βx ) = e −0.52476 (cos 30.066 + sin 30.066) = 0.21564
Equation (13.23) gives
w= P
8 β 3D
f 1 ( βx ) = P ( 0.80852 )
8( 20.9906 )3 D
= 0.0000109 DP

( a ) Radial contraction is thus


2w = 2( 0.0000109 DP ) = 0.0000218 DP

(CONT.)

118
(13.10 CONT.)

( b ) Applying Eqs.(13.5),
σx = 3P
2 βt 2
f 3 ( βx ) =
3 P ( 0.21564 )
2 ( 20.9906 )( 0.01)2
= 154.0975P
Resultant axial stress is, 2σ x = 308195
. P.
Similarly,
Pβ a
σθ = − 2t f 1 ( βx ) + 3νP
2 βt 2
f 3 ( βx )
=− ( 0.80852) +
P ( 20.9906 ) 0.375 3( 0.3) P ( 0.21564 )
2( 0.01) 2( 20.9906 )( 0.01)2

= −318.212 P + 46.2292 P = −271982


. P
Resultant hoop stress is, 2σθ = −543965
. P

SOLUTION (13.11)

From the first of Eqs.(13.25),


Qφ = ( N φ r0 ) − N θ cos θ + r1 p y
1 d r1
r0 dφ r0

The second of Eqs.(13.25) becomes


N φ r0 + N θ r1 sin φ + ( N φ r0 )] − [ r10 N θ cos θ + r1 p y ]
r
d
dφ [ r10 d

d

+ p z r1r0 = 0 (a)
The third of Eqs.(13.25) takes the form,
d
dφ ( M φ r0 ) − M θ r1 cos φ − r1 d
dφ ( N φ r0 ) + r12 N θ cos θ − r12 r0 p y p z = 0 (b)
In Eqs.(a) and (b), we have r0 = r2 sin φ .

SOLUTION (13.12)

The third of Eqs.(13.26) leads to


1 d
s ds ( M s s ) + 1s Mθ = Qs
Substituting the above into the second of Eqs.(13.26), we obtain
Nθ + d
ds [ dsd ( M s s ) + Mθ ]cot φ = − pz s cot φ (a)
Expression (a) and the first of expressions(13.26) now represent the governing equations of a
conical shell.

SOLUTION (13.13)

From Example 13.2,


pt 2 2.3
Mθ = M φ = 12 0.7 cos φ = 0.274 pt 2 cos φ

We have: p y = p sin φ , p z = p cos φ , p x = 0 , r0 = a sin φ . Expressions (13.27) become,


d
dφ ( N φ sin φ ) − N θ cos φ − Qφ sin φ = pa sin 2 φ (1)
N φ sin φ + N θ sin φ + d
dφ ( Qφ sin φ ) = − pa sin φ cos φ (2)
d
dφ ( 0.274 pt cos φ sin φ ) − 0.274 pt cos φ − Qφ a sin φ = 0
2 2 2
(3)
Using Eq.(3), we obtain
Qφ = − a1 0.274 pt 2 sin φ (a)
(CONT.)

119
(13.13 CONT.)
Equation (2) appears
0.274 pt 2 sin 2 φ
N φ sin φ + N θ sin φ + d
dφ (− a ) = − pa sin φ cos φ (b)
Differentiating both sides of the above with respect to φ,
d
dφ ( N φ sin φ ) + N θ cos φ − ( 0.274 pt ) cos 2φ = − pa cos 2φ
2
a
2
( 2' )
Subtract Eq. ( 2' ) from Eq.(1) :
0.548 pt 2 cos 2φ
− 2 N θ cos φ − Qφ sin φ + a = pa sin 2 φ + pa cos 2φ
or
0.548 pt 2 cos 2φ
− 2 N θ cos φ + a1 0.274 pt 2 sin 2 φ + a = pa(sin 2 φ + cos 2φ )
Solving
sin 2 φ 0.274 pt 2 cos 2φ
N θ = a1 0137 + a − (1 + cos 2φ )
pa
. pt 2 cos φ cos φ 4 cos φ (c)
Equation (c) gives for φ = 0 :
0.274 pt 2
Nθ = −
pa
a 2 (d)
σθ = 0.274 p( at ) −
pa
Thus, 2t
Hence,
σ θ , gen .
= + 1 = −0.548( at ) 2 + 1
0.274 p ( t / a )
σ mem. − pa / 2 t

We see that for t/a << 1 results differ very little. That is, σθ , gen. ≈ σ mem. .
Now substitute Eq.(d) into Eq.(b) and set φ = 0, to obtain:
2
Nφ = − −
0.137 pt pa
a 2

Then, σ φ = −0137
. p( at ) − 12 p( at )
It follows that,
σ φ , gen . −0.137 p ( t / a ) − pa / 2 t
σ mem. = − pa / 2 t +
= 0.274( at ) 2 + 1
− pa / 2 t

We observe, for t/a << 1, that σ φ , gen. ≈ σ mem. .

SOLUTION (13.14)

λ4 = 3(1 − ν 2 )( at )2 = 3(1 − 361 )(30)2 or λ = 7158


.
We also have α = 35 and
o

Pa 2 (1−ν ) 10 (103 )( 2.25 )2 (1− 1 6 )


4 λ2
= 4 ( 7.158 )2
= 205.85
φ λ (α − φ ) e λ ( φ −α ) sin λ (α − φ ) cos λ (α − φ )
35o 0 1 0 1
30 o 0.625 rad 0.5353 0.5851 0.8110
25o 1.25 rad 0.2865 0.9490 0.3153
20 o 1.874 rad 0.1535 0.9544 -0.2986
15o 2.50 rad 0.0821 0.5985 -0.8011
10 o 3.123 rad 0.0440 0.0186 -0.9998
5o 3.75 rad 0.0235 -0.5716 -0.8206
(CONT.)

120
(13.14 CONT.)
Pa 2 (1−ν )
Equation (13.45) gives M φ = 4 λ2
e λ ( φ −α ) [sin λ (α − φ ) − cos λ (α − φ )]
or
φ Mφ
35o 205.85(1) [0-1] = -205.85 N
30 o 205.85(0.5353)[ 0.5851 - 0.8110 ] = -24.89 N
25o 205.85(0.2865)[ 0.9490 - 0.3153 ] = 37.37 N
20 o 205.85(0.1535)[ 0.9544 + 0.2996] = 39.60 N
15o 205.85(0.0821)[ 0.5985 + 0.8011] = 23.65 N
10 o 205.85(0.044 )[ 0.0186 + 0.9998] = 9.224 N
5o 205.85(0.0235)[-0.5716 + 0.8206] = 1.20 N

Mφ ( N )

50

0 (φ )
35o 30 o 25 o 20 o 15o 10 o 5o
-50

-100

-150

-200

-250

SOLUTION (13.15)

Refer to solution of Prob.13.14. The stress σ φ ,max and σθ ,max occur at the fixed edges. Use
Eq.(13.5):
σθ = σφ = τθφ = 0
Nφ 6 Mφ
+ +
Nθ 6 Mθ
t t2 t t2
Here, from Eqs.(13.45):
M φ = −205.85 N
Mθ = νM φ = −34.31 N
10 (103 )( 2.25 )
Nφ = − =− = −1125
pa
2 2 . kN / m
3
Nθ = [( 56 )(1)(1 + 0) − 1] = −185
10 (10 )( 2.25 )
2 . kN / m
The compressive stresses are thus
6( 34.31)106
σθ = 1850
0.075 + 5625 = 24,666.7 + 36,597.3 = 6126
. kPa
σφ = 110.,075
6
+ 5625 = 150,000 + 219,573 = 369.6 kPa
250 6 ( 205.85 )10

121
CHAPTER 14

SOLUTION (14.1)

( a ) Expression (14.1) becomes


e − βx
w= 2 β 3D
( − Q1 cos βx ) = 0, Q1 ≠ 0, βx ≠ 0 ∴ cos βx = 0
We thus have
x= π
= Lmin β = [ ( 0.083( 0)(.91 ] = 82.97266 m−1
) 1
4
2β where 2 2
0.003 )
π
Hence Lmin = 2 ( 82.97266 ) = 0.01893 m = 18.93 mm

( b ) At the edge (x=0),


2 Q1βa 2
w = − 2 β 31D = −
Q
Et

The edge load is therefore,


−5
= − ( −7.52×(1082.97266
)( 200×10 )( 0.003 ) 9
Q1 = − 2wEt
βa 2 )( 0.082 )
= 42.37 kN / m

SOLUTION (14.2)

We have pressure vessel


pipe

β = [ ( 0.53( 0)(.91 ] = 18178 m −1


) 1
2
0.01 ) 2 . 4

x
z
π π p
(a) Lmin = β = 18.178 = 173 mm

( b ) Refer to Example 14.1 and


Table 13.1 (or Eqs.13.22).
At the fixed end (x=0), Lmin
3( 3.6×106 )
σx = − =
3p
β 2t 2
= 327 MPa(18.1782 )( 0.012 )

σθ = −νσ x = −981
. MPa
Clearly, the above obtained values are the largest axial and hoop stresses in the pipe owing
to the bending . They become negligible at Lmin distance away from the fixed end. Membrane
solution gives,
σθ = pa / t = 3.6( 0.5) / 0.01 = 180 MPa

SOLUTION (14.3)

At the simply supported edge M1 = 0. Pipe properties are :


200×109 ( 0.003 )2
β =[ 3( 0.91)
] 4 = 104.953 m−1 , D= = 494.505 N ⋅ m
1

( 0.052 )( 0.0032 ) 10.92

Then,
f 4 ( βL ) = f 4 (5.25) = 0.00268, f 2 ( βL ) = f 2 (5.25) = −0.005
By the use of Eq.(14.4), we obtain
pa 2
w = − 2 β13D Q1 f 4 ( βL ) where Q1 = −2β 3 D Et

(CONT.)

122
(14.3 CONT.)

Hence,
f 4 ( βL ) =
pa 2
w= Et
4×106 × 0.052
200×109 ( 0.003 )
× ( 0.00268 ) = 0.04472 × 10 −6 m.
Tangential stress equals,
σθ = +
Nθ 6 Mθ
t t2
−6
200×10 ( 0.04472×10 9
= − Ew
a = − = −178,888 Pa
Nθ )
where, t 0.05

Mθ = νM x = −νD d 2w
dx 2
=− 0.3
β [Q1 f 2 ( βL )]
4×106 ( 0.05 )2
Mθ = −0.3[2 × 104.9532 × 494.505 200×109 ( 0.003) ]( −0.005) = 0.27253 N
Therefore,
σθ = −178,888 + 6((−00.003
.27235 )
)
= −347,555 Pa = −0.3475 MPa 2

Membrane solution is
4×106 ( 0.05 )
σθ = pa
t = 0.003 = 66.67 MPa

SOLUTION (14.4)

3(1−ν 2 )
β4 = a 2t 2
= 3( 0.91)
(1)( 20×10 − 3 )2
= 6,825; β = 9.09 m−1
βL βL βL βL
Since L = 15. > π 9.09 : f1 ( 2 ), f 2 ( 2 ), f 3 ( 2 ), and f 4 ( 2 ) are all very small quantities,
we can use expressions derived in Example 14.1.
The maximum deflection occurs at x = L 2 . From Eq.(14.8), with f 3 = f 4 = 0:
pa 2
wmax = − = − 200( 20 ) = −1 mm
4 (1)
Et
Formula (f) of Example 14.1 yields at x=0:
σθ = − Ew
a −ν = 0− 6ν p
[ f1 ( 0) − 2 f 2 ( 0)]
6 Mx
t2 t 2 2β 2
4(106 )
= − ( 0.02 )2 [1 − 0] = −1019
6 ( 0.3 )
2 ( 9.09 )2
. MPa
While at x = L
2 :
200(10 )( −0.001)
σθ ,max = − Ew
9

a = − 1 = 200 MPa

SOLUTION (14.5)

We have E = 70 GPa (Table B.3). Refer to Solution of Prob. 14.4.

At x=L/2:
pa 2 2
wmax = − Et = − 70(20)
5(1)
= −3.57 mm
At x = 0:
σ θ = − Ew
a −ν
6M x
t2
= 0 − 6tν2 p
2β 2
[ f1 (0) − f 2 (0)]
5(106 )
= − (0.02)
6(0.3)
2
2(9.09)2
[1 − 0] = 136.2 MPa
At x=L/2:
σ θ ,max = − Ew 70(10 )( −0.00357) 9

a =− 1 = 250 MPa

123
SOLUTION (14.6)

General expression for direction is


w = C1 sin βx sinh βx + C2 sin βx cosh βx
pa 2
+ C3 cos βx sinh βx + C4 cos βx cosh βx − Et (1)
Equation(1) must be an even function of x. Thus,
C2 = C3 = 0 (2)
Boundary conditions are
w = 0, d 2w
dx 2
( x = L / 2)
=0 (3)
Introducing Eqs.(1) and (2) into Eq.(3), and setting α = βL / 2:
pa 2
− Et + C1 sin α sinh α + C4 cos α cosh α = 0
C1 cos α cosh α − C4 sin α sinh α = 0

Solving
pa 2 sin α sinh α pa 2 2 sin α sinh α
C1 = Et sin 2 α sinh 2 α + cos 2 α cosh 2 α = Et cos 2α + cosh 2α
pa 2 cos α cosh α pa 2 2 cos α cosh α
C4 = Et sin 2 α sinh 2 α + cos 2 α cosh 2 α = Et cos 2α + cosh 2α (4)
Noting that
Et
a2
= 4 Dβ 2 = 64α 4 D
L4
(5)
displacement may be written as follows :

2α + cosh 2α sin β x sinh β x − cos 2α + cosh 2α cos β x cosh β x ) (6)


4
w = − 64pLDα 4 (1 − cos2sin α sinh α 2cos α cosh α

It follows, at x=0, that


pL4
wmax = − 64 Dα 4 (1 − 2 cos α cosh α
cos 2α + cosh 2α ) Q.E.D
.
SOLUTION (14.7)

For long cylinders α → large. The second term in the parenthesis of Eq.(P14.6) becomes small.
Thus Eq.(P14.6) appears,
pL4 pa 2
wmax = − 64 Dα 4 = =
p
Dβ 4 Et Q.E.D.
This shows that, for long cylinders, the effect of the end supports upon the deflection at
midlength is negligible.

SOLUTION (14.8)

From Eq.(14.10),
Q1 = − P2 , M1 = h2 ( βL )
p
2β 2
where,
β = [ ( 0.63( 0)(.91 ] = 16.595 m−1 , βL = 21573
) 1
2
0.01 ) 2
4
. .
Therefore,
h1 ( βL ) = h2 ( βL ) = h3 ( βL ) ≈ 1
Equation (14.9) yields
2 (1.4×106 )
P= = = = 168.72 kN / m.
p 2p
(1− 12 ) β β 16.595

(CONT.)

124
(14.8 CONT.)

We have,
M1 = = 1.4×106
= 2542 N
p
2β 2 2 (16.595 )2

Q1 = − P2 = − −0.16873×10
= −8.4365 × 10 4 N / m
6

2
Then, at x=0:
wmax = − 2 β13D ( βM1 + Q1 )
M x ,max = − D ddxw2 = ( 2βM1 + 2Q1 ) = −2542 N
2
1
2β (as expected)
Axial stress equals
σ x ,max = = −152.52 MPa
6 M x ,max
t2
Tangential stress is given by, at x=0 :
σθ ,max =
Nθ ,max 6 Mθ ,max
t + t2
where,
Mθ ,max = 0.3 M x ,max = −762.584 Pa
12 (1−ν 2 )
= − Ew
a = − a ( βM1 + Q1 ) = −8.3995 × 10 7 Pa
Nθ ,max E
t 2 β 3 Et 3
Hence,
σθ ,max = −8.3995 × 10 7 + 6( −( 0762 .584 )
.01)
= −129.75 MPa. 2

SOLUTION (14.9)

Decrease in radius of pipe, from Eq.(13.23), is : P / 8β 3 D. Increase in radius of ring equals :


Pa 2 / AE. Therefore,
1
8 β 3D
= AE
a2

Solving
3 3
8 a 2β 3D 2 a 2t 3 3 4 (1−ν )
2
A= = = 156
4

E 3(1−ν 2 ) 3 3 . at 3 Q.E.D.
a 2t 2

SOLUTION (14.10)

Decrease in radius of pipe is, from Eq. (13.23): P / 8β 3 D. Increase in the radius of the
ring is : Pa 2 / AE. Thus,
AE = δ
+ Pa
2
P
8 β 3D

or taking ν = 0.3 and substituting the value of β :


0.643 EP ( at ) 2 + Pa 2

3
AE (a)
Here P is to be solved in terms of given parameters E, a, t, A, and δ.
Maximum bending moments is then,
M max = = P4 [ 3(1a−tν 2 ) ] 4
2 2 1
P
4β (b)
Hence the maximum bending stress:
σ max = =
6 M max 3P 2 2 1

t2 2t 2
[ 3(1a−tν 2 ) ] 4 (c)

125
SOLUTION (14.11)

In case of long shells h1 ( βL ) = h2 ( βL ) = h3 ( βL ) ≈ 1. From Eq.(14.11), with


β = 16.595 m −1
(see: solution of Prob.14.8):
P= = 1.4×106
= 1.4×106
= 160,956 N / m
p
t
+(1− 12 ) β + 2
0 . 01 16 . 595 0.4+8.298
A 0 . 025

And
Q1 = − P2 = −80,478 N / m

Also, using Eq.(14.12),


M1 = 1
2β 2
(p− Pt
A )= 1
2 (16.595 )2
. −
(14 0.160956× 0.01
0.025 )10 6 = 2425 N
At x=0:
w = − 2 β13D ( βM1 + Q1 )
M x ,max = − β1 ( βM1 + Q1 ) = 1
16.595 (16.595 × 2425 − 80478) = −2425 N
σ x ,max = = − 6( 0×.2425 = −145.5 MPa
6 M x ,max
t2 01)2

We have,
Mθ ,max = 0.3 M x ,max = −727.5 N .
N θ ,max = − Ewt
a =
Et 12 ( 0.91)
a 2 β 3 Et 3 ( βM1 + Q1 )
6× 0.91( −40235 )
= 4570.164 ( 0.6 ) ⋅ 10 4 = −0.801 × 10 6 N / m
Hence,
σθ ,max = − 0.801×10
+ 6(( −0.727 = −123.75 MPa.
.5 ) 6

0.01 2
01)

SOLUTION (14.12)

Reaction forces P per unit circumferential length of cylinder are produced between the plate
and the cylinder. Plate is taken to be in biaxial state of stress, σ r = σθ = P / t p . The radial
displacement at the circumference of the hole in the plate is thus
δ p = Ea (σ r − νσθ ) = (1 − ν )
Pa
1
Et p (a)
But Eq.(14.6) with M1 = 0 and Q1 = − P 2 yields for cylinder:
βa 2 P
wc = Et h1 ( βL ) (b)
Equating Eqs.(a) and (b):
tp 1−ν 1−ν at 1−ν
t = βah1 ( βL ) = 4
1
3(1−ν 2 ) ⋅a h1 ( βL )
= 4
3(1−ν 2 )
t 1
a h1 ( βL )

This, substituting h1 ( βL ) from Eq.(14.7), result in Eq.(P14.12).

SOLUTION (14.13)

Given : p = 9.81 × 3.7 = 36.3 kPa. With reference to Sec. 14.4, then have
σθ ,max = 1t γha = 01.2 36.3 × 2.7 = 480 kPa
and
σ x ,max = t
3
γha = 171
. σθ ,max = 171
. ( 480) = 821 kPa

126
SOLUTION (14.14)

Expressions (14.2) and (14.3) give,


wmax = − 2 β13D ( βM + Q ) (1)
dw
dx = 1
2β 2D
( 2βM + Q ) (2)
where,
Et23
β4 = D=
Et2
4a2D
, 12(1−ν 2 )
, Q = shear force at mn , M = moment at mn
Particular solution is
γ ( h− x ) γ ( h− x )a 2
w1 = − 4 β 4 D1
=− Et1 (3)
The above yield at x = h1 ,
γ ( h − h1 ) a 2 γa 2
w1 = − =
dw1
Et1 , dx Et1 (4)

The continuity of deflection and slope at mn will be maintained by setting Eqs. (1) and (2)
equal to Eqs.(4) with reversed sign:
γa 2 ( h − h1 ) γa 2
− 1
2 β 3D
( βM + Q ) = Et1 , 1
2β 2D
( 2 βM + Q ) = − Et1

or
2γa 2 ( h − h1 ) β 3 D 2γa 2 β 2 D
− ( βM + Q ) = Et1 , ( 2 βM + Q ) = − Et1 (5)
Adding these equations,
2γa 2 β 2 D
βM = Et1 [( h − h1 )β − 1]
from which
2γa 2 βD
M= Et1 [( h − h1 ) − 1]
Equation (5) gives then
2 a 2γβ 2 D
Q=− Et1 [2β ( h − h1 ) − 1]

SOLUTION (14.15)

Thermal deflection is δ = aα ( ΔT ) = 0.3(17 × 10 −6 )100 = 51


. × 10 −4 m. Expressions
(14.2) and (14.3) give,
− 1
2 β 3D
( βM1 + Q1 ) = 51
. × 10 −4 , 1
2β 2D
( 2βM1 + Q1 ) = 0
from which
M1 = 2aα ( ΔT )β 2 D , Q1 = −4aα ( ΔT )β 3 D
120×109 ( 5×10−3 )
β = [ ( 0.3 3)(( 05.×9110) . m −1 ,
] 4 = 3319 D= = 1374 × 103 N ⋅ m .
1
Here, 2 −3 2
) 12( 0.91) .
Thus,
. × 10 −4 )(3319
M1 = 2(51 . ) 2 (1374
. × 103 ) = 1543
. × 103
. × 10 −4 )(3319
Q1 = −4(51 . )3 (1374
. × 103 ) = −10245
. × 105
Deflection and moment are given by Eqs.(14.16). We see that, maximum moment occurs at
x=0. Hence,
M x ,max = 2( 0.3)(17 × 10 −6 )(3319
. )2 (1374
. × 103 )(100)[ f1 ( 0 ) − f 2 ( 0 )]
= 1544
. × 103 N
(CONT.)

127
(14.15 CONT.)

Then,
6 (1.544×103 )
σ x ,max = + = 0+ = 370.6 MPa
Nx 6 M x ,max
t t2 ( 5×10− 3 )2
Hoop stress equals
σθ ,max = − Ew
a − νσ x = 0 − 0.3( 370.6 ) = −1112
. MPa

SOLUTION (14.16)

a = 0.6 m, t = 0.012 m, ν = 0.3, α = 117


We have . (10 −6 ) , ΔT = 50 o C , and
E = 200 GPa. Thus
3(1−ν 2 ) 3( 0.91)106
β=4 a 2t 2
=4 0.36 (144) . m −1
= 1515
200 (12)3
D= Et 3
12 (1−ν 2 )
= 12 ( 0.91) = 31,648 N ⋅ m
βx = 1515
. ( 0.04) = 0.6, f1 ( 0.6) = 0.763, f 2 ( 0.6) = 0.31
f 3 ( 0.6) = 0143
. , f 4 ( 0.6) = 0.453.

Applying Eqs.(14.16):
w = −0.6(117. )(10 −6 )50[( 0143
. − 2 × 0.453) + 1] = −0.083(10 −3 ) mm
. × 10 −6 )(1515
M x = 2( 0.6)(117 . ) 2 (31,648)(50)[0.763 − 0.31] = 2.31 kN ⋅ m

Then, using Eq.(f) of Example 14.1:


σx = 6 Mx
t2
= 6( 2, 310 )
( 0.012 )2
= 96.25 MPa
200 (10)9 ( −0.083×10 −6 )
σθ = Ew
a − νσ x = 0.6 − 0.3(96.25 × 10 6 )
= −0.028(10 6 ) − 28.875(10 6 ) = −28.9 MPa

SOLUTION (14.17)

By applying Eq.(14.19),
Eα ( ΔT )
σθ ,max = [ 3(1 − ν ) + (1 − ν 2 ) 2 ]
1

2 3 (1−ν )

100×109 (19×10 −6 )( 200 )


= 2.425 [1197
. + 0.954] = 339.5 MPa

SOLUTION (14.18)

Upon substituting the given data into Eq.(14.24) :


εθ = 0.002× 200×109 ×11.7×10 −6 ×134+ 0.006×103×109 ×18.9×10 −6 ×134
0.002× 200×109 + 0.006×103×109
. × 10 −3
= 2153
−3
Radial Δr = 2153
expansion is thus, . × 10
2 × 0.398 = 0.857 mm.
Stresses are now obtained as follows :
⎧σ s1 ⎫ ⎡2135 − 117
. (134)⎤ ⎧167.2⎫
⎨ ⎬= =⎨
200×103
⎢2153 − 117 ⎬ MPa
⎩σ s 2 ⎭
1− 0.3
⎣ . (134)⎥⎦ ⎩167.2⎭

and
(CONT.)

128
(14.18 CONT.)
⎧σ s 2 ⎫ 103×103 ⎡2135 − 18.9(134)⎤ ⎧55.8⎫
⎨ ⎬ = 1− 0.3 ⎢ ⎥ = −⎨ ⎬ MPa
⎩σ s3 ⎭ ⎣2153 − 18.9(134)⎦ ⎩55.8⎭
It is seen that N s + N b = 12 ( 2 × 167.2 )( 0.005) − 12 ( 2 × 55.8 )( 0.015) ≈ 0.

SOLUTION (14.19)

Discontinuity stress, from Eq.(14.36), is


σθ ,max = 1032
. t = 1032
ap
. 0.01 = 6192
0.6×1
. MPa
Membrane stress equals
σθ = pa
t = 60 MPa

SOLUTION (14.20)

Expressions (14.38) lead to


0.6(1×10 )
σ x ,max = 2172
6

2 t = 2172 2( 0.01) = 6516


ap
. . . MPa
σθ ,max = 1128
. t = 67.68 MPa.
ap

By applying Eq.(12.13), at the edge (φ = π / 2 ):


0.6 (1×106 )
σ φ = − t (1+cos φ ) = − =− = −60 MPa , σθ = 60 MPa
ap ap
t 0.01

SOLUTION (14.21)

The constants are


3( 0.91)( 0.6 )2
ρ= = 1, k =[ ] 4 = 9.957 , β = 16.595
tc 1
th ( 0.01)2
Applying Eqs.(14.41),
( 9.957 )3 + 2 (1.7 )9.957+ 2 (1.7 )(1.3 ) 106 × 0.6
Q1 = − 2×9.9572 +[ 0.7+1.3]9.957+ 0.91 4 = −702 × 103 N / m
2 ( 9.957 )3 + 0.7 ( 9.957 )2 + 2×1.7 (1.3 ) 106 × 0.6
M1 = 2×9.9572 +[ 0.7+1.3]9.957+ 0.91 8×9.957
= 42.246 × 103 N

Expression for bending moment is thus,


M x = M1 f1 ( βx ) + β1 Q1 f 2 ( βx ) = 42.246 × 103 f1 ( βx ) − 42.302 × 103 f 2 ( βx )
Maximum M x occurs at x=0 and is equal to M1 . Therefore,
6 ( 42.246×103 )
σ x ,max = ( 0.01)2
= 2535 MPa

Deflection is given by Eq.(14.4),


w = − 2 β13D [βM1 f 3 ( βx ) + Q1 f 4 ( βx )]
. × 10 −3 f 4 ( βx ) − 4188
= 4193 . × 10 −3 f 3 ( βx )
Hoop stress is expressed by
6νM x
σθ = − −
ap Ew
t a t2
(a)

(CONT.)
(14.21 CONT.)

129
Upon substitution of the M x and w obtained above and and the given numerical values we
obtain, from Eq.(a):
σθ = 60 × 10 6 + e − βx [−762.275 × 10 6 cos( βx ) − 1395
. × 109 sin( βx )] (b)
To ascertain maximum value of σθ we proceed in usual manner.
Then,
dσ θ
dx = 0 = e − βx β [2.157 × 109 sin( βx ) − 632.677 × 10 6 cos( βx )]
or
0 = 2157
. × 109 sin( βx ) − 632.677 × 10 6 cos( βx )
or
tan( βx ) = 293.282 × 10 −3 ∴ βx = 16.34 o = 0.285 rad
Hence, Eq.(b) becomes
σθ ,max = 60 × 10 6 + e −0.285 [−762.275 × 10 6 cos 16.34 o − 1395
. × 10 9 sin 16.34 o ]
= −785 MPa
To check the correctness of the results, we refer to Sec.14.10. We observe that
θh ' = 810 θh " = 018315
6 3

×18315×1.3 = 1134 ×1.3 = 1065


( 0. 6 ) .6× 42246
. , .
θc = 2( 275.394 )18315 [2 × 16.595 × 42246 − 702000] = 0.069
1

and
θc = θh '−θh " = 0.069, as required by Eq.(14.39).
Similarly, we calculate
δc ' = 153
. × 10 −4 , δh = 1475 . × 10 −4 , δc " = 0.056 × 10 −4
Thus, approximately, δc ' = δh + δc " , which is required by Eq.(14.40).

SOLUTION (14.22)

( a ) Membrane stress in the shell is given by σ = pa / 2t. Thickness is therefore


2×106 ( 0.5 )
t= = = 0.005 m = 5 mm
pr
2σ 2 (100×106 )

( b ) From Table 14.2,


2×106 ( 0.5 )
t= = = 0.00627 m = 6.27 mm
pr
2σe − 0. 2 p 2 (100×106 ) 0.8− 0.2 ( 2×106 )

130
CHAPTER 15

SOLUTION (15.1)

Upon substitution of the given numerical values,


2γaL2 t 2 ⋅12 (1−ν 2 ) 2γ ( 50 )( 252 )⋅12 (1− 0.32 ) γ
Zt
2000 = 2000 Et 3π 2
= 2000 E ( 0.5 )π 2
= 691517
. E

With reference to Table 15.1 we then have, from Eqs.(15.11) :


wmax = c10 + c11 − c30 − c31 + c50 + c51
= 2000
Zt
( −1212 − 1183 + 6.742 + 6.704 − 0.526 − 0.525)
γ
or wmax = −1647612
. E cm.
Elastic law (15.10) gives,
Nx = Et
1−ν 2
[ ∂∂ux + ν ( a1 ∂v
∂θ − wa )]
N x ,max = Et
1−ν 2
{a mn mπ
L ( − sin m2π ) + ν [ an bmn sin m2π − a1 sin m2π ]}
= Et
1−ν 2
{[a10 πL ( −1) + a11 πL ( −1) + a30 πL (1) + a31 πL (1) + a50 πL ( −1) + a51 πL ( −1)]
+ ν [b10 ( 0) + b11 a1 (1) + b30 ( 0) + b31 a1 ( −1) + b50 ( 0) + b51 a1 (1)
− c10 a1 (1) + c11 a1 (1) + c30 a1 ( −1) + c31 a1 ( −1) + c50 a1 (1) + c51 a1 (1)]}
= 1−Etν 2 {[ πL ( − a10 − a11 + 3a30 + 3a31 − 5a50 − 5a51 ] + ν [ a1 ( b11 − b31 + b51 )
− a1 ( c10 + c11 − c30 − c31 + c50 + c51 )]}
Introducing the given data,
69.1517 ( 0.5 )γ
π
N x,max = { 25
1− 0.32
( −106.4729) + 0.3[− 66.30442
50 − ( − 238250.605 )]}
= 19.68γ N / cm.
Similarly, M x = − D( ∂∂xw2 + ν ∂ 2w
2

a 2 ∂θ 2
)
M x ,max = − 12(1Et−ν 2 ) [cmn ( mLπ ) 2 ( − sin m2π ) + cmn ( n 2 ) sin m2π ]
3

= − 12(1Et−ν 2 ) {[− c10 ( πL ) 2 − c11 ( πL ) 2 + c30 ( 3Lπ ) 2 + c31 ( 3Lπ ) 2 − c50 ( 5Lπ ) 2 − c51 (5 πL ) 2 ]
3

ν
+ a2
[c10 ( 0) + c11 + c30 ( 0) − c31 + c50 ( 0) + c51 ]}
=− Et 3
12(1−ν 2 )
{[( L ) ( − c10 − c11 ) + ( 3Lπ ) 2 ( c30 + c31 ) + ( 5Lπ ) 2 ( − c50 − c51 )]
π 2

ν
+ a2
( c11 − c31 + c51 )}
Introducing given numerical values into the above,
69.1517 ( 0.5 )3 γ π 2
M x,max = − 12 (1− 0.32 )
{[( 25 ) ( 2395) + ( 325π ) 2 ( −13.446) + ( 525π ) 2 (1051
. )]
+ ( 500.3)2 ( −1176.821)} = −28.865γ N
Circumferential stress resultants, from Eqs.(15.10), are :
N θ ,max = 1−00.5.32 ( 46.327 − 4.014)691517
. γ = 1607.7γ N / cm.
M θ ,max = ( 0.5 )3
12 (1− 0.32 )
( −10.8973 − 0.4707 )691517
. γ = −8.999γ N
Therefore,
γ 6 ( −28.865γ )
σ x ,max = 19.680..182
5 + ( 0.5 ) = −653.4γ N / cm2 2

1607.7γ 6( −8.999γ )
σθ ,max = 0.5 + ( 0.5 )2
= 2999.4γ N / cm2

131
SOLUTION (15.2)

Use the results found in Solution of Prob.15.1. Now we have


2γaL2t 2 ⋅12 (1−ν 2 ) 2γ ( 50 )( 25 )2 ⋅12 (1− 0.32 ) γ
Zt
2 , 000 = 2 , 000 Et 3π 2
= 2 , 000 E (1)π 2
= 34.5759 E
Equation (15.11) on the basis of Table 15.1:
γ
wmax = c10 + c11 − c30 − c31 + c50 + c51 = −82380.7 E cm
Equations (15.10) yield:
N x ,max = Et
1−ν 2
{a mπ
mn L ( − sin m2π ) + ν [ an bmn sin m2π − a1 sin m2π ]}
34.5759 (1)γ π
= { ( −106.4729) + 0.3[− 66.21802
0.91 25 50 − ( − 238250.605 )]}
= 19.68 N / cm
Similarly,
34.5759 (1)3 γ π 2
M x,max = − 12 ( 0.91) {( 25 . ) + ( 325π ) 2 ( −13.446)
) ( 2395
. ) + ( 500.3)2 ( −1176.821)} = −114.566γ N
+ ( 525π ) 2 (1051
N θ ,max = 1.0
0.91 ( 46.372 − 4.014 )34.5759γ = 1,609.4γ N / cm
( −10.8973 − 0.4707 )(34.5759γ ) = −35.994γ N
(1)3
M θ ,max = 12 ( 0.91)

Stresses are therefore


γ 6 ( −114.566γ )
σ x ,max = 191.68
.0 + (1.0 )
= −667.7γ N / cm2
2

1, 609.4γ 6 ( −35.994γ )
σθ ,max = 1.0 + (1.0 )2
= 1,393.4γ N / cm2

SOLUTION (15.3)

From Table B.3, E = 105 GPa . Flexural rigidity is thus


3
D = 12(1Et−ν 2 ) = 12(1−1 3) = 1, 640.6 N ⋅ m
3 105(5)

Use Eqs. (d) and (e) of Example 15.3. Vertical reduction at the center:
3
4(103 )(0.2)3
wv = 0.149 2PaDL = 0.149 2(1,640.6)(0.4) = 3.63 mm
Horizontal increase at the center:
3 3
wh = 0.137 2PaDL = 0.137 2(1,640.6)(0.4) = 3.34 mm
3
4(10 )(0.2)

SOLUTION (15.4)

From Table B.3, E = 200 GPa . Hence,


3
D = 12(1Et−ν 2 ) = 12(1 = 2, 289.4 N ⋅ m
3 200(5)
− 0.32 )

Apply Eqs. (d) and (e) of Example 15.3. Vertical reduction at the center:
3 3 3
wv = 0.149 2PaDL = 0.149 2(2,289.4)(0.4)
4(10 )(0.2)
= 2.6 mm
Horizontal increase at the center:
3 3
wh = 0.137 2PaDL = 0.137 2(2,2894.4)(0.4) = 2.39 mm
3
4(10 )(0.2)

132
SOLUTION (15.5)

The corresponding characteristic equation of Eq.(15.31):


α 8 + λ8 = 0 or α = 8 − 1λ
Applying De Moivre’s formula, the roots of 8 − 1 are ( see, for example,
Calculus and Analytic Geometry by Thomas, Addison Wesley ):
± ( 0.923879 ± i0.382683), ± ( 0.382683 ± i0.923879)
We have
α1, 2,3, 4 = ±(γ ± iβ ) α 5 , 6 , 7 , 8 = ± ( β ± iγ )
where i = − 1 and
γ = 0.923879λ β = 0.382683λ
Thus, the solution of Eq.(15.31)
W = A1eα1θ + ⋅ ⋅ ⋅ ⋅ ⋅ + A8 eα8θ
is expressed in the form given by Eq.(P15.5).

SOLUTION (15.6)

We have
A11 = ( A11* ) k ( t k − t k −1 ) = ( A11* ) o ( t1 − t o ) + ( A11* ) i ( t 2 − t1 ) + ( A11* ) o ( t3 − t 2 )
= E y ( − 2i + 2t ) + E x ( 2i + 2i ) + E y ( 2t − 2i ) = 2 E y t o + E x t i
t t t t

A22 , A12 , and A33 may be determined similarly. The D’s are already obtained in Sec. 8.9.

SOLUTION (15.7)
Given: t o = −5 mm , t1 = −1 mm , t 2 = 5 mm , α = 30 o
From Example 8.7:
⎡15 15 . 0⎤ ⎡12.984 0.703 1925
. ⎤
⎢ ⎥ 9 ⎢ ⎥
[ D ]2 = ⎢15
*
. 3.75 0⎥10 , [ D ]1 = ⎢ 0.703 7.359 2.895⎥109
*
(a)
⎢⎣ 0 0 5⎥⎦ ⎢⎣ 1975
. 2.895 4.203⎥⎦

Formula (15.55) is thus


Aij = ( Dij* )1 ( t1 − t o ) + ( Dij* ) 2 ( t 2 − t1 )
= ( Dij* )1 [−1 − ( −5)]10 6 + ( Dij* ) 2 [5 − ( −1)]10 6
= [4( Dij* )1 + 6( Dij* ) 2 ]10 6 (b)
Substituting Eqs.(a) into (b):
⎡15 15 . 0⎤ ⎡12.984 0.703 1925
. ⎤
⎢ ⎥ 6 ⎢ ⎥
[ A] = 4(10 )⎢15. 3.75 0⎥ + 6(10 )⎢ 0.703 7.359 2.895⎥
6

⎢⎣ 0 0 5⎥⎦ ⎢⎣ 1975
. 2.895 4.203⎥⎦
Adding the matrices we obtain:
⎡137.904 10.218 1185
. ⎤
⎢ ⎥
[ A] = ⎢ 10.218 51154
. 17.37 ⎥ MN ⋅ m
⎢⎣ 1185
. 17.37 45.218⎥⎦

133
SOLUTION (15.8)

For L/a < 5 : Using Eq.(15.69),


− 16
1 1
σ cr = 0.605[1 − 0.901(1 − e )] 200×101( 0.025 ) = 2,135 MPa.
9
0 . 025

For L/a > 5 : Using Eq.(15.59)


σ cr = 0.605 ⋅ 200×101( 0.025 ) = 3,025 MPa.
9

No failure in buckling. Since σ cr > σ yp and material would yield first.

SOLUTION (15.9)

Application of Eq.(15.69) yields,


− 16
1 0.6
σ cr = 0.605[1 − 0.901(1 − e 0 . 012
)] = 16412
. MPa.
We thus have,
Pcr = 2πrtσ cr = 2π ( 0.6)( 0.012)(16412
. ) = 74,246 kN .

SOLUTION (15.10)

( a ) For L/a < 5, use Eq.(15.69) :


− 16
1 0.4
0.605[1 − 0.901(1 − e )] 210×a10 t =
9
t 450000
2πat
or
( 0.099 + 0.901e −0.0395/ t )t 2 = 5.6371 × 10 −7
By trial and error,
t = 0.00121 m = 121
. mm

( b ) For L/a > 5, we employ Eq.(15.59) :


0.605 210×a10 t =
9
450000
2πat
from which
t = [ 2π ( 210450000 ] 2 = 7.508 × 10 −4 m = 0.751 mm
1

×109 )( 0.605 )
Note that, for yielding to impend :
400 × 10 6 [2π ( 0.4)t ] = 450,000
Solving
t = 4.476 × 10 −4 m = 0.448 mm
Thus, the thickness for the long pipe is limited to 0.751 mm. However, if L/a < 5, t is limited to
1.21 mm.

134
K10792

6000 Broken Sound Parkway, NW


Suite 300, Boca Raton, FL 33487
270 Madison Avenue
an informa business New York, NY 10016
www.taylorandfrancisgroup.com 2 Park Square, Milton Park
Abingdon, Oxon OX14 4RN, UK

You might also like